Subscribe to ForumIAS

10000 Questions for Prelims 2020 | Prelims 2020 Marathon

Hi,

With hardly 120 days left for Prelims 2020, I urge all community members and aspirants for 2020 to invest their energies into Prelims preparation. As we all know, practice is the key; let's pool in our diversified resources and ensure we solve enough MCQs. 

5.6k views

48) During British rule, the formation of Nari Bahinis or women brigades is associated with the

a. Tebhaga Movement

b. Quit India Movement

c. Civil Disobedience Movement

d. Mappila Revolt

3.2k views
@prelims2020 No that's not correct... 


3k views
@upsc2020 Correct Answer is A. 



The Tebhaga movement erupted in 1946 in Bengal on the eve of the withdrawal of the British. Although the tide of Tebhaga receded as fast as it rose, the uprising stands out as one of the most important political events in twentieth century Bengal. Among the unique features of the movement is the large-scale participation of women on par with men. The landless and poor peasant women formed fighting troops called nari bahini and took a front rank role in defending the gains of the movement and in countering the repression of the state.


5.5k views

Kapdagonda shawls, recently in news are associated with the tribe

A. Dongria Kondh

B. Bru

C. Chenchu 

D. Bhuyans


3k views
@Patrick_jane That's Correct. 


The Dongarias named the shawl ‘Kapdagonda’ in their Kui language. They weave three types of shawls — Kapdagonda, Sialiganda and Ullindiganda. 

Kalahandi and Rayagada district administrations have been pressing for GI tag to the Dongaria Shawl since 2018. 

3k views

Kapdagonda shawls, recently in news are associated with the tribe

A. Dongria Kondh

B. Bru

C. Chenchu 

D. Bhuyans


A)Dongria kondh,pvtg of odisha uses this kapdagonda shawls.

That's Correct. 

The Dongarias named the shawl ‘Kapdagonda’ in their Kui language. They weave three types of shawls — Kapdagonda, Sialiganda and Ullindiganda. 


Kalahandi and Rayagada district administrations have been pressing for GI tag to the Dongaria Shawl since 2018. 

3k views

50. 

With reference to GOI Act 1935, consider the following statements: 

1. Entry into the Federation was compulsory for the Provinces but voluntary for the Princely States.

2. Indian legislature whether Federal 

or provisional was authorised to amend the 1935 Act. 

Which of the statement above is/are correct? 

A. 1 only 

B. 2 only 

C. Both 1 and 2

D. Neither 1 nor 2

3k views

#35)Consider the following historical places: 

1. Ajanta Caves 

2. Bagh caves 

3. Sittanvasal 

Which of the above places is/are also known for mural paintings? 

(a) 1 only 

(b) 1 and 2 only 

(c) 1, 2 and 3 

(d) None of the above

C

3k views

#42)According to Buddhist tradition, Maha Sammata was

A. The first monarch of the world

B. A compassionate Bodhisattva who renounced his life for the Sangha

C. A character of Gautam Buddha’s previous life

D. None of the above

A

3k views

#39)Consider the following statements regarding mountain and valley breeze: 

1. Valley breeze flows during the day from valley towards the mountains. 

2. Mountain breeze flows during the night from mountains towards valley. 

Which of the statements given above is/are correct? 

(a) 1 only 

(b) 2 only 

(c) Both 1 and 2 

(d) Neither 1 nor 2

Is it D ? 

3k views

#38)Consider the following statements about the Jotedars in the 18th century Bengal: 

1. They were subordinate to the Zamindars in terms of influence in the villages. 

2. They controlled local trade as well as moneylending. 

3. They were called haoladars, gantidars or mandals. 

Which of the statements given above is/are correct? 

(a) 1 only

(b) 2 and 3 only 

(c) 3 only

(d) 1, 2 and 3

The rise of jotedars weakened the authority of Zamindars... 

Francis Buchanan’s survey of the Dinajpur district in North Bengal we have a vivid description of this class of rich peasants known as jotedars. By the early nineteenth century, jotedars had acquired vast areas of land – sometimes as much as several thousand acres. They controlled local trade as well as moneylending, exercising immense power over the poorer cultivators of the region. A large part of their land was cultivated through sharecroppers (adhiyars or bargadars) who brought their own ploughs, laboured in the field, and handed over half the produce to the jotedars after the harvest.

Within the villages, the power of jotedars was more effective than that of zamindars. Unlike zamindars who often lived in urban areas, jotedars were located in the villages and exercised direct control over a considerable section of poor villagers. They fiercely resisted efforts by zamindars to increase the jama of the village, prevented zamindari officials from executing their duties, mobilised ryots who were dependent on them, and deliberately delayed payments of revenue to the zamindar. In fact, when the estates of the zamindars were auctioned for failure to make revenue payment, jotedars were often amongst the purchasers.


3k views

50. 


With reference to GOI Act 1935, consider the following statements: 


1. Entry into the Federation was compulsory for the Provinces but voluntary for the Princely States.


2. Indian legislature whether Federal 


or provisional was authorised to amend the 1935 Act. 


Which of the statement above is/are correct? 


A. 1 only 


B. 2 only 


C. Both 1 and 2


D. Neither 1 nor 2

3k views

51. 

Consider the following statements.

1. In election of a President, an MP can give his vote to more than one candidate.

2. All the disputes regarding elections of President lies with election commission of india.

Which of the statement above is/are correct? 


A. 1 only 


B. 2 only 


C. Both 1 and 2


D. Neither 1 nor 2


3k views

50. 


With reference to GOI Act 1935, consider the following statements: 


1. Entry into the Federation was compulsory for the Provinces but voluntary for the Princely States.


2. Indian legislature whether Federal 


or provisional was authorised to amend the 1935 Act. 


Which of the statement above is/are correct? 


A. 1 only 


B. 2 only 


C. Both 1 and 2


D. Neither 1 nor 2

A

Only British could amend. Indians had no power 

Correct Answer. 

3k views

51. 

Consider the following statements.

1. In election of a President, an MP can give his vote to more than one candidate.

2. All the disputes regarding elections of President lies with election commission of india.

Which of the statement above is/are correct? 


A. 1 only 


B. 2 only 


C. Both 1 and 2


D. Neither 1 nor 2


D

Incorrect. 

Lets wait for others to answer it out. 

2.6k views

51. 

Consider the following statements.

1. In election of a President, an MP can give his vote to more than one candidate.

2. All the disputes regarding elections of President lies with election commission of india.

Which of the statement above is/are correct? 


A. 1 only 


B. 2 only 


C. Both 1 and 2


D. Neither 1 nor 2


D

Incorrect. 

Lets wait for others to answer it out. 

Correct Answer is A. 


Jurisdiction lies with Supreme Court. 

2.6k views

Isn't it that National emergency on account of external aggression amounts to War.

Also, national emergency can be declared on account of 'war and external aggression.'

Thus, option B should be correct?

Even I find option 1&2 tricky here

2.6k views

Consider the following statements. 

1. Ideal locations for oil palm trees are within eight degrees latitude north and south of the equator. 

2. Oil palm has very high productivity when compared to other oilseeds like mustard.

Which of the statement above is/are correct? 


A. 1 only 


B. 2 only 


C. Both 1 and 2


D. Neither 1 nor 2

2.6k views

Consider the following statements. 

1. Ideal locations for oil palm trees are within eight degrees latitude north and south of the equator. 

2. Oil palm has very high productivity when compared to other oilseeds like mustard.

Which of the statement above is/are correct? 


A. 1 only 


B. 2 only 


C. Both 1 and 2


D. Neither 1 nor 2

C.

That's correct

2.6k views

Which among the following motions are introduced to draw the attention of a minister to a matter of urgent public importance?

1. Calling Attention Motion

2. Adjournment Motion

3. Privilege Motion

Select the correct answer using the code given below.

A. 1 and 2 only

B. 2 and 3 only

C. 1 and 3 only

D. 1 only


2.6k views

Consider the following statements:

1. An individual needs to have 6 years of continuous stay in the country to obtain citizenship by naturalisation.

2. When an Indian citizen voluntarily acquires the citizenship of another country, his Indian citizenshipa automaticallyterminates.

Which of the statements given above is/are implied by the Constitution?

A. 1 only

B. 2 only

C. Both 1 and 2

D. Neither 1 nor 2


2.6k views

Which among the following motions are introduced to draw the attention of a minister to a matter of urgent public importance?

1. Calling Attention Motion

2. Adjournment Motion

3. Privilege Motion

Select the correct answer using the code given below.

A. 1 and 2 only

B. 2 and 3 only

C. 1 and 3 only

D. 1 only


A

Calling Attention Motion It is introduced in the Parliament by am memberto call the attention of a minister to a matter of urgent publici importance and to seek an authoritative statement from him on that matter.


Adjournment Motion It is introduced in the Parliament to draw attentiono ofthe House to a definite matter of urgent public importance, and needs thes supportof 50 members to be admitted.

Privilege Motion It is concerned with the breach of parliamentaryprivileges by a minister. It is moved by a member when he feels that aminister has committed a breach of privilege of the House or one or more ofi itsmembers by withholding facts of a case or by giving wrong or distortedfacts. Its purpose is to censure the concerned minister.



2.6k views

Consider the following statements regarding "earthquake swarms"


1. In 'earthquake swarms', numerous high intensity earthquakes occur locally over an extended period.

2. In earthquake swarms, there is a clear sequence of foreshocks, main quakes and aftershocks.

Which of the above written statements is/are not true?


A.  

1 only


B.  

2 only


C.  

Both 1 and 2


D.  

Neither 1 nor 2



2.6k views

Which of the following was/were not an outcome of Battle of Buxar?

1. Nawab of Awadh was expelled from the rule of Awadh

2. Grant of Diwani and Nizamat functions in Bengal, Bihar and Orissa to East India Company.

Select the correct answer using the codes given below.

a. 1 only

b. 2 only

c. Both 1 and 2

d. Neither 1 or 2

2.6k views

With reference to ground level ozone (GLO), consider the following statements:

1. GLO reach unhealthy levels on cold days.

2. They can be transported long distances by wind.

Which of the statements given above is/are correct?

a) 1 only

b) 2 only

c) Both 1 and 2

d) Neither 1 nor 2

2.6k views

#61)Consider the following statements related to the Ramosi Peasant Force:

1. It was organized by Anant Lakshman Kanhere.

2. It aimed to rid the country of the British by instigating an armed revolt.

Which of the above statements is/are correct? 

(a) 1 only 

(b) 2 only 

(c) Both 1 and 2 

(d) Neither 1 nor 2


It was formed by Vasudev Balwant Phadke.

2.5k views

#62)In which of the following state does Vedanthangal Bird Sanctuary Located?

(a)Tamil Nadu 

(b) Karnataka 

(c) Kerala 

(d) Telangana 

A: Tamil Nadu


Vedanthangal Bird Sanctuary is one of the smallest bird sanctuaries of India that is situated in Tamil Nadu. The sanctuary is home to approx. 40,000 birds out of which 26 belong to a rare species.

Some famous species include Cormorants, Pintails, Pond Heron, Paddy Bird, Painted Stork, Shovellers, Terns, White Ibis with migratory birds like Grey Pelican Australia, Grey Heron Bangladesh, Open-billed Stork Bangladesh make their shelter in Vedanthangal Bird Sanctuary. 

2.5k views

#63) Which of the following Tribe believes Diwali is a Symbol of Mourning?

a) Tharu

b) Santhal

c) Saharia

d) Bhoksas

@Patrick_jane No idea. Was this is news?

2.5k views

#64)The term Abwabs refers to which of the following.

a) Mid level Military officials in the Delhi sultanate.

b) Taxes or fines levied on subjects by native chiefs of odisha.

c) Gifts given to Native chiefs by Nizam.

d) Octroi tax levied in the region of Awadh.

A?

2.5k views

#65)Consider the following statements 

1) C R Das is also called as Deenbadhu.

2)C F Andrew's also called as Deshbandu.

Which of the above statements is/are correct? 

(a) 1 only 

(b) 2 only

(c) Both 1 and 2

(d) Neither 1 nor 2


D

Reverse is true. 

2.5k views

With reference to Zero Budget Natural Farming, consider the following statements.

1. It was developed as an alternative to the Green Revolution’s methods. 

2.The method encourages intensive irrigation and deep ploughing.

Which of the above statements is/are correct? 


(a) 1 only 


(b) 2 only


(c) Both 1 and 2


(d) Neither 1 nor 2

2.5k views

The government recently approved setting up of Vardhavan port in the state of 

A. Tamil Nadu

B. Gujarat

C. Maharashtra

D. Andhra Pradesh

2.5k views

Polycrack technology is related to 

A. Mining

B. Waste to Energy

C. Artificial intelligence

D. Irrigation

2.3k views

#69)Consider the following statements regarding ‘Potti Sriramulu’

1. He was a social activist worked for emancipation of dalits

2. Participated in individual satyagraha including salt satyagraha

3. His struggle led to the formation of Andhra Pradesh with its capital Kurnool

Select the correct answer using the code given below

A. 1 only

B. 1 and 3 only

C. 1, 2 and 3 

D. 1 and 2 only

D?

2.3k views

#71)The Seva Sadon was founded by

A. Naoroji Furdonji

B. S.S. Bangal

C. J.B. Wacha

D. Behramii M Malabari

D

2.3k views

With reference to World Bank's Pandemic Emergency Financing Facility (PEF), consider the following.

1. It was set up after the Ebola outbreak in West Africa. 

2. It is financed by voluntary contribution from developed countries. 

Select the correct answer. 

A. 1 only 

B. 2 only

C. 1 and 2 Both

D. Neither 1 nor 2


2.3k views

White Spot Syndrome Virus, recently in news, is known to affect


A. Pigs


B. Birds 


C. Marine Life 


D. Dogs

2.3k views

White Spot Syndrome Virus, recently in news, is known to affect


A. Pigs


B. Birds 


C. Marine Life 


D. Dogs

C

2.3k views

With reference to Pangolins, consider the following statements. 

1. The international trade of Pangolins is banned. 

2. They are listed as critically endangered under IUCN. 

Select the correct answer. 


A. 1 only 


B. 2 only


C. 1 and 2 Both


D. Neither 1 nor 2

2.3k views

Which among the following country is NOT a member of Central Asia Business Council? 

A. Turkmenistan

B. Kazakhstan

C. Uzbekistan

D. Afghanistan

2.3k views

Which among the following is a possible impact of Removal of Dividend Distribution Tax? 

1. Reduction of cost of doing business

2. Foreign investors in tax Havens

3. Increase the in hand profits of PSU companies.

Select the correct answer using the code below. 

A. 1 and 2 only

B. 2 and 3 only 

C. 1 and 3 only 

D. 1, 2 and 3 



2.3k views

Which among the following country is NOT a member of Central Asia Business Council? 

A. Turkmenistan

B. Kazakhstan

C. Uzbekistan

D. Afghanistan

D. Tajakistan is a member. 

That's correct. 

Ministry of External Affairs along with FICCI (Federation of Indian Chambers of Commerce and Industry) recently launched the India-Central Asia Business Council. They were joined by 5 countries namely Tajikistan, Uzbekistan, Turkmenistan, Kazakhstan and Kyrgyz Republic. The Council will act as a common platform for the businesses of Central Asia and India.

2.3k views

Deposit Insurance and Credit Guarantee Corporation (DICGC) is a wholly-owned subsidiary of

A. SEBI 

B. RBI 

C. LIC 

D. None of the above

2.3k views

With reference to Super Critical Carbon Dioxide, consider the following statements.

1. The technology can help to generate clean energy from power plants.

2. The technology will help in making the power plants cheaper with lower operating costs.

Which of the statements given above is/are correct?

a. 1 only

b. 2 only

c. Both 1 and 2

d. Neither 1 nor 2

2.2k views

Which among the following is/are indoor air pollutants?

1. Radon

2. Volatile organic compounds

3. Asbestos

Select the correct answer using the code given below.

a. 1 and 2 only

b. 2 and 3 only

c. 3 only

d. 1 and 3 only

2.2k views

Which among the following may be an effect of the policy of Helicopter money?

A.        It may lead to appreciation of domestic currency.

B.     It may lead to increased consumer spending. 

C.     It may lead to lower economic growth.

D.     It could have a deflationary impact on economy. 

2.2k views

Deposit Insurance and Credit Guarantee Corporation (DICGC) is a wholly-owned subsidiary of

A. SEBI 

B. RBI 

C. LIC 

D. None of the above

B. 

That's correct. 

Deposit Insurance and Credit Guarantee Corporation (DICGC), wholly owned subsidiary of RBI, provides insurance for bank deposits.

All commercial banks (public, private & rural) including branches of foreign banks functioning in India and cooperative banks are insured by the DICGC.

2.2k views

C

C for which question ? Request you to please answer by replying to the question there...


2.2k views


Polycrack technology is related to 

A. Mining

B. Waste to Energy

C. Artificial intelligence

D. Irrigation


B is the correct answer. 

Indian Railways has commissioned country’s first governmental waste to energy plant at Bhubaneswar.The national transporter has put in place the governmental Waste to Energy plant in Mancheswar Carriage Repair Workshop which falls under East Coast Railway. The waste to energy plant uses a patented technology called POLYCRACK, is first-of-its-kind in Indian Railways and fourth in India.

It is world’s first patented heterogeneous catalytic process which converts multiple feed stocks into hydrocarbon liquid fuels, gas, carbon and water.


@Puppy_123

2.2k views

#16)Which of the following provision does not necessarily make elections democratic? 

(a) Elections to be held at regular intervals

(b) Political Equality and Universal Adult Franchise

(c) Free to contest elections

(d) Provide real choice between candidates

c

Should it not be D? It is a vague statement

2.2k views

With reference to Super Critical Carbon Dioxide, consider the following statements.

1. The technology can help to generate clean energy from power plants.

2. The technology will help in making the power plants cheaper with lower operating costs.

Which of the statements given above is/are correct?

a. 1 only

b. 2 only

c. Both 1 and 2

d. Neither 1 nor 2

C


That's correct. 

2.2k views

Which among the following is the major component of Aviation Fuel?

a. Diesel

b. Natural Gas

c. Petrol

d. Kerosene

2.2k views

Which among the following may be an effect of the policy of Helicopter money?

A.        It may lead to appreciation of domestic currency.

B.     It may lead to increased consumer spending. 

C.     It may lead to lower economic growth.

D.     It could have a deflationary impact on economy. 

B?

That is correct. 

Helicopter money involves the central bank or central government supplying large amounts of money to the public, as if the money was being distributed or scattered from a helicopter. One of the main benefits of helicopter money is that the policy theoretically generates demand, which comes from the ability to increase spending. One of the primary risks associated with helicopter money is that the policy may lead to a significant currency devaluation in the international foreign exchange markets. The currency devaluation would be primarily attributed to the creation of more money.


2.2k views
@prelims2020 Radon. Comes from Soil. 


@anshul_rai_sharma It think@prelims2020 is right. Radon is an indoor pollutant and all would be the right answer. 

2.2k views

 Which among the following factors is/are necessary for the formation of a delta?

1.        A river must have large sediment load.

2.        Velocity of river must be high to deposit sediment at river mouth.

3.        Deposition by rivers should be more than removal by tides. 

Select the correct answer using the code given below.

A.        1 and 3 only

B.       1 and 2 only

C.    1 only

D.    1, 2 and 3

2.2k views

 Which among the following factors is/are necessary for the formation of a delta?

1.        A river must have large sediment load.

2.        Velocity of river must be high to deposit sediment at river mouth.

3.        Deposition by rivers should be more than removal by tides. 

Select the correct answer using the code given below.

A.        1 and 3 only

B.       1 and 2 only

C.    1 only

D.    1, 2 and 3

A? 

That's correct. 


For the formation of a delta, following conditions are necessary:

1.        A river must have a large sediment load.

2.        The velocity of a river must be sufficiently low to allow most of the load to be deposited in the river’s mouth.

3.        The river’s sediment load must be deposited faster than it can be removed by the action of tides and currents. 

2.2k views

Consider the following statements about organ transplantation:

1.        Human body tissue cannot be used for transplantation.

2.        Organs can't be used for transplantation when there is instant death due to road accident. 

Which of the statements given above is/are correct? 

A.  1 only

B.  2 only 

C.  Both 1 and 2

D. Neither 1 nor 2

2.2k views

"The name of the dance form is derived from the name of a village. It consists of two forms - the traditional musical dance-drama and the solo dance. Techniques like with a pitcher full of water on the head exist. “

The above excerpt best describes which of the following dance forms?

A.    Bharatnatyam

B.        Mohiniyattam

C.        Manipuri

D.        Kuchipudi


2.2k views

“George left his job to pursue a training course. After completion of training, he did not get the job as per his preference and at the expected wage.”

With reference to the data given above, which type of unemployment situation is depicted?

A. Disguised unemployment

B. Structural unemployment

C. Frictional unemployment

D. Seasonal unemployment

2.2k views

Consider the following statements about organ transplantation:

1.        Human body tissue cannot be used for transplantation.

2.        Organs can't be used for transplantation when there is instant death due to road accident. 

Which of the statements given above is/are correct? 

A.  1 only

B.  2 only 

C.  Both 1 and 2

D. Neither 1 nor 2

B

That is correct. 

In addition to organs, one can also donate tissue. Tissues is composed of layers of cells that function together to serve a specific purpose. Cornea is one of the most commonly transplanted tissues each year. 

In India, a large number of people die due to road traffic accidents. People that die due to these circumstances are unable to donate their organs. As organs need to be transplanted as soon as possible following the donor’s death. Organs come from people who are certified as dead while on a ventilator in a hospital intensive care unit. A deceased donor is kept on a ventilator after he or she has been declared brain dead until the organs can be harvested.

2.2k views

"The name of the dance form is derived from the name of a village. It consists of two forms - the traditional musical dance-drama and the solo dance. Techniques like with a pitcher full of water on the head exist. “

The above excerpt best describes which of the following dance forms?

A.    Bharatnatyam

B.        Mohiniyattam

C.        Manipuri

D.        Kuchipudi


D?

Correct. 

Kuchipudi is the name of a village in the Krishna district of Andhra Pradesh. It is about 35 km. from Vijayawada. To show the dexterity of the dancers in footwork and their control and balance over their bodies, techniques like dancing on the rim of a brass plate and with a pitcher full of water on the head was introduced.

2.2k views

“George left his job to pursue a training course. After completion of training, he did not get the job as per his preference and at the expected wage.”

With reference to the data given above, which type of unemployment situation is depicted?

A. Disguised unemployment

B. Structural unemployment

C. Frictional unemployment

D. Seasonal unemployment

A

Disguised Unemployment is correct. 

2.2k views

With reference to the financial sector, the term “Unicorn” frequently seen in news refers to:

A.        Shares making run against the market

B.       Single profit making company of a sector

C.      Start-up Company valued over 1 billion dollars.

D.       Debt free profit making company valued over 10 billion dollars

2.2k views

1 ? What ?

2.2k views

b

Please reply to the question by quoting it, as it is difficult to comprehend you are giving answer to which question.

2.2k views

Which among the following factors determine the climate of India?

1. Longitude

2. Altitude

3. Continentality

Select the correct answer using the code given below.

A. 1 and 2 only

B. 2 and 3 only

C. 1 and 3 only

D. 1, 2 and 3

2.2k views

Which of the police reforms have been instructed by the Supreme Court in Prakash Singh v/s Union Of India, case?

1. Establishment of Police Complaint Authority.

2. Separation of law making and investigation function of police.

3. Creation of Police establishment board.

Select the correct answer from the codes given below:

A. 1 and 2 only

B. 2 and 3 only

C. 1 and 3 only

D. 1, 2 and 3

2.2k views

Which one of the following organisations issues the “Inclusive Development Index”?

A.. World Bank

B. IMF

C. World Economic Forum

D. Asian Development Bank


2.3k views

Which one of the following organisations issues the “Inclusive Development Index”?

A.. World Bank

B. IMF

C. World Economic Forum

D. Asian Development Bank


C.. WEF

That's correct

2.3k views

Which of the police reforms have been instructed by the Supreme Court in Prakash Singh v/s Union Of India, case?

1. Establishment of Police Complaint Authority.

2. Separation of law making and investigation function of police.

3. Creation of Police establishment board.

Select the correct answer from the codes given below:

A. 1 and 2 only

B. 2 and 3 only

C. 1 and 3 only

D. 1, 2 and 3

D

Correct. 

In Prakash Singh Vs Union of India (2006), SC delivered a judgement instructing the central and state governments to comply with a set of seven directives that laid down practical mechanism to kick-start police reforms.

The directives include,

1.Constitute a State Security Commission (SSC) to ensure that the state government does not exercise unwarranted influence or pressure.

2.Ensure that the DGP is appointed through the merit-based transparent process with a minimum tenure of 2 years.

3.Even police officers on operational duties are also provided minimum tenure of 2 years.

4.Separation of Investigative and Law and Order functions of the police.

5.Set up a Police Establishment Board to decide on transfers, postings, promotions and other service related matters.

6.Set up Police Complaints Authority at state and district level to inquire into public complaints against police officers.

7.Set up National Security Commission at the Union level to prepare a panel for selection and placement of Chiefs of the Central Police Organizations.

2.3k views

Which among the following factors determine the climate of India?

1. Longitude

2. Altitude

3. Continentality

Select the correct answer using the code given below.

A. 1 and 2 only

B. 2 and 3 only

C. 1 and 3 only

D. 1, 2 and 3

B

Correct. Latitude has effect, not longitude. 

2.3k views

Dry, warm summers, predominant orchard farming and near absence of cattle rearing.” The above statement best

describes which of the following regions?

A. African Savannah

B. Mediterranean Region

C. Central Asian Steppes

D. Siberian Tundra

2.3k views

School Health Ambassador Initiative has been launched under the scheme:

A. Pradhan Mantri Swasthya Suraksha Yojana

B. National Health Mission

C. Rasthriya Bal Swasthya Karyakram

D. Ayushman Bharat

2.3k views

Which among the following sectors can use Technical Textiles?

1. Medicine

2. Agriculture

3. Construction

Select the correct answer using the code given below.


A. 1 and 2 only

B. 2 and 3 only

C. 1 and 3 only

D. 1, 2 and 3


2.3k views

Consider the following statements about organ transplantation:

1.        Human body tissue cannot be used for transplantation.

2.        Organs can't be used for transplantation when there is instant death due to road accident. 

Which of the statements given above is/are correct? 

A.  1 only

B.  2 only 

C.  Both 1 and 2

D. Neither 1 nor 2

B

That is correct. 

In addition to organs, one can also donate tissue. Tissues is composed of layers of cells that function together to serve a specific purpose. Cornea is one of the most commonly transplanted tissues each year. 

In India, a large number of people die due to road traffic accidents. People that die due to these circumstances are unable to donate their organs. As organs need to be transplanted as soon as possible following the donor’s death. Organs come from people who are certified as dead while on a ventilator in a hospital intensive care unit. A deceased donor is kept on a ventilator after he or she has been declared brain dead until the organs can be harvested.

shouldn't the answer be D?

as eyes can be harvested for corneal transplantation even after instant death.


Cornea is a tissue, not an organ. Cornea is transplanted, not the entire eye. 

2.3k views

“George left his job to pursue a training course. After completion of training, he did not get the job as per his preference and at the expected wage.”

With reference to the data given above, which type of unemployment situation is depicted?

A. Disguised unemployment

B. Structural unemployment

C. Frictional unemployment

D. Seasonal unemployment

A

Disguised Unemployment is correct. 

i think answer should be C.frictional unemployment

because the unemployment is between switching the jobs.and he did not accept the job.i.e.he is still unemployed.

disguised unemployment implies he is visibly employed,but actually unemployed as there are lower wages.(that would mean he has already accepted the job,which in this case,he hasn't.)

is it possible?

There is a caveat of training - that enhanced his training skills. Herein, disguised unemployment refers to the adoption of inferior jobs with a relatively lower productivity. 

Frictional unemployment is the result of employment transitions within an economy. It is due to voluntary movement by workers. 

2.3k views

Which among the following sectors can use Technical Textiles?

1. Medicine

2. Agriculture

3. Construction

Select the correct answer using the code given below.


A. 1 and 2 only

B. 2 and 3 only

C. 1 and 3 only

D. 1, 2 and 3


C?

No. It is D

Technical textiles are material and products manufactured primarily for their technical properties and functional requirements rather than for aesthetic characteristics. The scope of use of technical textiles encompasses a wide range of applications such as agro-textiles, medical textiles, geo-textiles, protection-textiles, industrial-textiles, sports-textiles and many other usages. Use of technical textiles have benefits of increased productivity in agriculture, horticulture and aquaculture fields, better protection of military, para-military, police and security forces, stronger and sturdier transportation infrastructure for highways, railways, ports and airports and in improving hygiene and healthcare of general public. In India, technical textiles hold immense growth opportunities both for the industry as well as across various applications.

2.3k views

School Health Ambassador Initiative has been launched under the scheme:

A. Pradhan Mantri Swasthya Suraksha Yojana

B. National Health Mission

C. Rasthriya Bal Swasthya Karyakram

D. Ayushman Bharat

D?

Correct

2.3k views

Dry, warm summers, predominant orchard farming and near absence of cattle rearing.” The above statement best

describes which of the following regions?

A. African Savannah

B. Mediterranean Region

C. Central Asian Steppes

D. Siberian Tundra

B?

Correct.

2.3k views

Consider the following statements about the 13th Conference of Parties (COP) of the Convention on the conservation of migratory species.

1. India will chair the presidency of CMS f0r three years from 2020. 

2. The mascot for CMS COP13 is ‘Gibi – The Great Indian Bustard

3. The Great Indian Bustard is a critically endangered species 

Which among the above statements is/are correct?

a. 1 and 2 only

b. 2 and 3 only

c. 1 and 3 only

d. 1, 2 and 3

2.3k views

The terms Nirukta, Jyotisha and Kalpa refers to

A. Vedangas

B. Upanishads

C. Tantras

D. Mahabhashya

2.3k views

Consider the following pairs:

Women ruler          Territory

1. Raziya Sultana    : Delhi 

2. Rudramdevi         : Kashmir

3. Didda                   : Deccan

Which of the pairs given above is/are correctly matched?

A. 1 and 2 only

B. 3 only

C. 1 only

D. 1, 2 and 3

2.3k views

Consider the following pairs:

Women ruler          Territory

1. Raziya Sultana    : Delhi 

2. Rudramdevi         : Kashmir

3. Didda                   : Deccan

Which of the pairs given above is/are correctly matched?

A. 1 and 2 only

B. 3 only

C. 1 only

D. 1, 2 and 3

I think its C.

Rudrama devi is ruler of warangal/kakatiya Empire.

Raziya sultan of Delhi sultanate ruled Delhi.

Didda no idea?

Answer is correct.

Queen Didda was the ruler of Kashmir from 958 CE to 1003 CE. Most information pertaining to her is derived from the Rajatarangini, a twelfth-century Sanskrit treatise on Kashmir by Kalhana.

2.3k views

Consider the following statements about the 13th Conference of Parties (COP) of the Convention on the conservation of migratory species.

1. India will chair the presidency of CMS f0r three years from 2020. 

2. The mascot for CMS COP13 is ‘Gibi – The Great Indian Bustard

3. The Great Indian Bustard is a critically endangered species 

Which among the above statements is/are correct?

a. 1 and 2 only

b. 2 and 3 only

c. 1 and 3 only

d. 1, 2 and 3

I think its D

The term of members of CMS is 3yrs,India is current chair.

CMS is also known as Bonn convention which is the only legal binding agreement for protection of Migratory species n its habitats.

Great Indian Bustard (Gibi) is the mascot for 13th COP of CMS held in Gujarat.

Gibi is in Critical endangered category of IUCN.

Schedule 1 of wpa 1972

Project Godawan of Rajasthan for protection n conservation of Gibi.



Correct..

2.3k views

The terms Nirukta, Jyotisha and Kalpa refers to

A. Vedangas

B. Upanishads

C. Tantras

D. Mahabhashya

I think its A.

The Nirukta,jyothisha and kalpa are part of Vedangas which form as Limbs of 4 vedas.

Correct 

2.3k views

Which one of the following is the correct sequence of ecosystems in the order of decreasing net productivity?

A. Coastal Marsh, Temperate Deciduous Forest, Open Ocean, Desert

B. Temperate Deciduous Forest, Coastal Marsh, Desert, Open Ocean

C. Temperate Deciduous Forest, Coastal Marsh, Open Ocean, Desert

D. Coastal Marsh, Open Ocean, Temperate Deciduous Forest, Desert

2.3k views

Which one of the following is the correct sequence of ecosystems in the order of decreasing net productivity?

A. Coastal Marsh, Temperate Deciduous Forest, Open Ocean, Desert

B. Temperate Deciduous Forest, Coastal Marsh, Desert, Open Ocean

C. Temperate Deciduous Forest, Coastal Marsh, Open Ocean, Desert

D. Coastal Marsh, Open Ocean, Temperate Deciduous Forest, Desert

A.. I Think. 

Correct

These are representative values for the net productivity of a variety of ecosystems — both natural

and managed. These values are only approximations and are subject to fluctuations because of variations in

temperature, fertility, and availability of water.

Estimated Net Productivity of Certain

Ecosystems (in kilocalories/m2/year)

15,000 Tropical rain forest

12,000 Coastal marsh

5,000 Temperate deciduous forest

2,500 Ocean close to shore

2,000 Tall-grass prairie

800 Open ocean

500 Desert

2.3k views

The process of curing inflation by reducing money supply is called

A. Cost-push inflation

B. Demand-pull inflation 

C.Disinflation 

D. Reflation

2.3k views

Which of the following factors pose threat to the wetlands?

1. Urbanization

2. Agriculture

3. Aquaculture

4. Water transport

Select the correct answer using the code

given below.

A. 1 and 3 only

B. 2 and 4 only

C.1, 3 and 4 only

D. 1, 2, 3 and 4

2.3k views

All of the goods which are scarce and limited in supply are called

A. Luxury goods

B. Expensive goods 

C. Capital goods

D. Economic goods

2.2k views

All of the goods which are scarce and limited in supply are called

A. Luxury goods

B. Expensive goods 

C. Capital goods

D. Economic goods

D?

Correct. 

Economic good is a good or service that has a benefit (utility) to society. Also, economic goods have a degree of scarcity and therefore an opportunity cost.

2.2k views

Which of the following factors pose threat to the wetlands?

1. Urbanization

2. Agriculture

3. Aquaculture

4. Water transport

Select the correct answer using the code

given below.

A. 1 and 3 only

B. 2 and 4 only

C.1, 3 and 4 only

D. 1, 2, 3 and 4

D?

Correct

2.2k views

The process of curing inflation by reducing money supply is called

A. Cost-push inflation

B. Demand-pull inflation 

C.Disinflation 

D. Reflation

C?

Correct. Disinflation is a temporary slowing of the pace of price inflation. It is used to describe instances when the inflation rate has reduced marginally over the short term.

2.2k views

All of the goods which are scarce and limited in supply are called

A. Luxury goods

B. Expensive goods 

C. Capital goods

D. Economic goods

D?

A?

Incorrect.

Answer is D - Economic Goods. 

Economic goods are those which have a price and their supply is less in relation to their demand or is scarce. The production of such goods requires scarce resources having alternative uses. For example, land is scarce and is capable of producing rice or sugarcane. If a farmer wants to produce rice he will have to forgo the production of sugarcane. The price of rice equals the production of sugarcane forgone by the farmer. Thus economic goods relate to the problem of economizing scarce resources for the satisfaction of human wants. 

Luxury goods, also called superior goods, are products with a demand that is directly related to consumer income exponentially. In other words, when consumer income increases, they purchase more of these goods and vice versa.

2.2k views

The process of curing inflation by reducing money supply is called

A. Cost-push inflation

B. Demand-pull inflation 

C.Disinflation 

D. Reflation

C?

C?

Correct.

2.2k views

With respect to HOPE SPACECRAFT,choose incorrect statements

1.it is being sent by saudi arabia with help of the USA

2."hope" name suggests inspiration for the arab world.

3.it will put a rover on the surface and observer in the orbit to study mars


A.1 and 2 only

B. 2 and 3 only

C. 1 and 3 only

D.all of the above.

C.

2.2k views

Consider the following pairs.

  Peninsula                   Country

1. Chukchi                     Japan

2. Yucatan                      Mexico 

3. Jutland                       Denmark

Which of the pairs given above are correctly matched?

a. 1 and 2 only

b. 2 and 3 only

c. 1 and 3 only

d. 1, 2 and 3


2.2k views

With reference to culture, Chahar Bait refers to

a. a theatre form of Himachal Pradesh.

b. a folk music form from Afghanistan

c. the style of gardens in Mughal palaces

d. a traditional animal sport of Mizoram

2.2k views

With reference to cultural history of India, Karuvi refers to

a. temple dance forms

b. musical instruments

c. bronze sculptures

d. burial practices

2.2k views

Which among the following sites are prominent Neolithic Settlements?

1. Attirampakkam

2. Maski

3. Gufkral

Select the correct answer using the code given below.

a. 1 and 3 only

b. 1 and 2 only

c. 2 and 3 only

d. 1, 2 and 3

2.2k views

With reference to cultural history of India, Karuvi refers to

a. temple dance forms

b. musical instruments

c. bronze sculptures

d. burial practices

 c? No idea .Guessing

Incorrect. Answer is B

Musical instruments are the tangible and material representation of music which is an auditory art. A study of these helps in tracing the evolution of music and also explains many aspects of the material culture of the group of people to which these instruments belong. For instance, the hair used for making the bow, the wood or clay used for making the drum, or the hide of animals used in the instruments, all these tell us about the flora and fauna of a particular region.

The Tamil word for instrument-Karuvi is found in Sangam literature of the 2nd to 6th century A.D., the literal meaning of which is "tool". This is extended to mean instrument in the context of music.

2.4k views

Which among the following sites are prominent Neolithic Settlements?

1. Attirampakkam

2. Maski

3. Gufkral

Select the correct answer using the code given below.

a. 1 and 3 only

b. 1 and 2 only

c. 2 and 3 only

d. 1, 2 and 3

D ?

Incorrect. Answer is C

The Neolithic sites were spread over almost all the regions of Indian subcontinent. In the northwestern region Mehrgarh is a classic site in the Kachi plains of Baluchistan. The important sites in Kashmir Valley include Burzahom and Gufkral. In South India the Neolithic settlements were discovered along the rivers Bhima, Krishna, Tungabhadra and Kaveri. Some important sites are Sanganakallu, Brahmagiri, Maski, Piklihal, Hallur in Karnataka; Utnur, Nagarjunakonda, Budihal in Andhra Pradesh; and Paiyampalli in Tamil Nadu. These sites have yielded dwelling pits alongwith the evidence of cultivation of cereals and domestication of animals. 

In Tamil Nadu, an important Paleolithic settlement site is Attirampakkam in Chingleput region. 

2.4k views

With reference to Indian culture, Sadir refers to the

a. dance practiced by Devdasis

b. mural paintings done in temples of Kerala

c. ancient theatre form of Meghalaya

d. couplets composed by sufi saints

2.4k views

With reference to tribal movements, Kherwar movement is associated with

a. Gonds

b. Santhals

c. Mundas

d. Ahoms

2.4k views

With reference to the economic philosophy of Ambedkar, consider the following statements.

1. Ambedkar visualized a larger role of state and the constitutional methods to bring about inclusive

economic growth.

2. He believed that the caste system is responsible for immobility of labour and capital in

the country. 

3. He believed in state ownership of agricultural land and key industries .

Which of the statements given above are correct?

a. 1 and 2 only

b. 1 and 3 only

c. 1 and 3 only

d. 1, 2 and 3


2.4k views

 Recently fortification of drinking water has been used in prevention of deficiencies in children. Water can be fortified with

1. Vitamins

2. Iron

3. Zinc

Select the correct answer using the code given below.

a. 1 and 2 only

b. 2 and 3 only

c. 1 and 3 only

d. 1, 2 and 3

2.4k views

With reference to culture, Chahar Bait refers to

a. a theatre form of Himachal Pradesh.

b. a folk music form from Afghanistan

c. the style of gardens in Mughal palaces

d. a traditional animal sport of Mizoram

C ?

Incorrect. That's Chahar Bagh which are Mughal Gardens.

Correct answer is B. 

Chahar (four) bait (verse), as its name suggests, is a musical composition in which verses of four lines each are sung. The popular folk music form originated in Afghanistan. Iit has deep connections with the khayal form of classical Hindustani music. Called dedh phadakka, chahar bait was popularised in India by Afghani soldiers who used it as an interlude during war.  It has roots in the war poetry of Arabia, where it was read out in battlefields to motivate soldiers.

2.4k views

For which question?

2.4k views

Chindu Bhagavatam/Yakshaganam is traditional theater art form that belongs to which of the following region.

a)Karnataka 

b)Lakshwadeep 

c)Telangana 

d)Tamil Nadu

A

2.4k views

Recently in news "Dasuopu Glacier" is located at 

a)Greenland

b)Antarctica 

c)Himalayas

d)Alps

C

2.4k views

 Recently fortification of drinking water has been used in prevention of deficiencies in children. Water can be fortified with

1. Vitamins

2. Iron

3. Zinc

Select the correct answer using the code given below.

a. 1 and 2 only

b. 2 and 3 only

c. 1 and 3 only

d. 1, 2 and 3

D

2.4k views

With reference to tribal movements, Kherwar movement is associated with

a. Gonds

b. Santhals

c. Mundas

d. Ahoms

B

2.4k views

With reference to Indian culture, Sadir refers to the

a. dance practiced by Devdasis

b. mural paintings done in temples of Kerala

c. ancient theatre form of Meghalaya

d. couplets composed by sufi saints

A?

Correct. Sadir Natyam – a solo dance form performed for centuries by devadasis in temples and eventually in the royal courts of South India, especially in Tamil Nadu.

2.3k views

Chindu Bhagavatam/Yakshaganam is traditional theater art form that belongs to which of the following region.

a)Karnataka 

b)Lakshwadeep 

c)Telangana 

d)Tamil Nadu

A

A?

Correct. 

2.3k views

Chindu Bhagavatam/Yakshaganam is traditional theater art form that belongs to which of the following region.

a)Karnataka 

b)Lakshwadeep 

c)Telangana 

d)Tamil Nadu

A

A?

Correct. 

No crct answer isc

  • It is a theatre art form popular in north Telangana. Chindu is derived from the artistes’ caste Chindu Madiga, a sub-caste of Madiga among SCs.
  • The art form dates back to second century BC.
  • Chindu Bhagavatam is also called Chindu Yakshaganam as it is similar to Yakshaganam, a traditional theatre form particular to Karnataka State.
  • Most of the stories narrated in the art form are from ‘Bhagavatam’.

Good trap. 

2.3k views

Alley Cropping refers to 

a. planting rows of trees at wide spacings with a companion crop

b. allowing the stubbles of the original crop to grow again after harvesting

c. repetitive growing of the same crop on the same land.

d. more than two crops of different heights cultivated simultaneously on the same field

2.3k views

Subhash Chandra Bose was associated with the Indian National Congress for a long period of time. During his tenure, which among the following steps were undertaken by him?

1. He set up the National Planning Committee to develop a national plan for India. 

2. He introduced the Anti Zamindari bill in central legislature. 

3. He represented Indian National Congress in the Third Round Table Conference. 

Select the correct answer using the code given below.

a. 1 only

b. 1 and 2 only

c. 2 and 3 only

d. 1 and 3 only

2.3k views

Subhash Chandra Bose was associated with the Indian National Congress for a long period of time. During his tenure, which among the following steps were undertaken by him?

1. He set up the National Planning Committee to develop a national plan for India. 

2. He introduced the Anti Zamindari bill in central legislature. 

3. He represented Indian National Congress in the Third Round Table Conference. 

Select the correct answer using the code given below.

a. 1 only

b. 1 and 2 only

c. 2 and 3 only

d. 1 and 3 only

A??

Correct.

2.3k views

Alley Cropping refers to 

a. planting rows of trees at wide spacings with a companion crop

b. allowing the stubbles of the original crop to grow again after harvesting

c. repetitive growing of the same crop on the same land.

d. more than two crops of different heights cultivated simultaneously on the same field

A

Correct

2.3k views

With reference to Consumer Protection Act 2019, consider the following: 

1. It does not protect consumers from E Commerce. 

2. Celebrities are accountable for promoting misleading acts. 

Which is correct? 

A. 1 only 

B..2 only 

C..1 and 2 both 

D. Neither 1 nor 2

2.3k views
@samudragupt Yes, E-Commerce is included in the definition of market. I think statement 2 is also correct. Posting a relevant screenshot from prs.@upsc2020 pls clarify in case this is wrong.



B is the correct answer.

2.3k views

"It is found in lentic ecosystem, flowers of this plant are capable of thermoregulation and leaves are hydrophobic in nature". The statement best describes which flower? 

a.   Water Hyacinth

b.        Orchids

c.        Sacred Lotus

d.        Sea Buckthorm

2.3k views

When RBI hikes the rates, what impact it may have on the consumer?

1.        Home loans may be obtained at reduced rates.

2.        EMIs on previous loans may increase. 

Which of the statements given above is/are correct?

a.      1 only

b.        2 only

c.        Both 1 and 2

d.        Neither 1 nor 2


2.3k views

Which among the following animals are associated with Harappan culture?

1.        Elephant

2.        Rhinoceros

3.        Horse

4.        Buffaloes

Select the correct answer using the code given below.

a.        1, 2 and 4 only

b.        2, 3 and 4 only

c.        2 and 3 only

d.        1, 2, 3 and 4 

2.3k views

Consider the following statements with reference to Copernicus earth observation programme:

1. It has been launched by NASA.

2. European Space Agency and ISRO are associate partners for developing the project. 

Which of the statements given above is/are correct? 

a.  1 only

b.  2 only 

c.  Both 1 and 2

d. Neither 1 nor 2


2.3k views

Which among the following animals are associated with Harappan culture?

1.        Elephant

2.        Rhinoceros

3.        Horse

4.        Buffaloes

Select the correct answer using the code given below.

a.        1, 2 and 4 only

b.        2, 3 and 4 only

c.        2 and 3 only

d.        1, 2, 3 and 4 

Harappan fellas were well acquainted with Elephant and Rihno. They had also domesticated buffalo. Honestly, I am still doubtful in Horse. To my mind, ans should be 'A'. Can anyone pls deal with this horse theory? #Haathi Ghoda Paalki, jai Kanhaiya laal ki.

Correct Answer isD

Information about animals which are associated with HArappan era is through seals, arachaeological findings like toys, skeletal remains etc. PAshupati seal of one horned male deity is surrounded by four animals viz elephant, tiger, rhinoceros and a buffalo.  The discovery of crafts includes bronze rhinoceros, elephant, two-wheeled charriot with a rider and a buffalo. Skeletal remains from the sites suggest the presence of domesticated and wild animals in these cultures. The important domesticated animals were cattle, sheep, goat, dog, pig, horse, etc. The wild animals included black buck, antelope, nilgai, barasinga, sambar, cheetah, wild buffalo and one-horn rhino. The bones of fish, water fowl, turtle and rodents were also discovered.

Source: http://download.nos.org/srsec315new/History%20Book_L03.pdf

2.3k views

Consider the following statements with reference to Copernicus earth observation programme:

1. It has been launched by NASA.

2. European Space Agency and ISRO are associate partners for developing the project. 

Which of the statements given above is/are correct? 

a.  1 only

b.  2 only 

c.  Both 1 and 2

d. Neither 1 nor 2


This space programme is managed by ESA. Nasa and Isro it seems don't have any role. The prime objective is to provide high quality imagery of earth, to augment environment related data. The ans should be 'D' none of the statement is correct.

Correct.

2.3k views

When RBI hikes the rates, what impact it may have on the consumer?

1.        Home loans may be obtained at reduced rates.

2.        EMIs on previous loans may increase. 

Which of the statements given above is/are correct?

a.      1 only

b.        2 only

c.        Both 1 and 2

d.        Neither 1 nor 2


Statement 1 is outrightly false. Increase in rate makes fresh home loans costlier.

Statement 2, 'might' word makes it tricky. MCLR attached loans may see a hike in EMI's. 


The answer should be 'B', only second statement is correct. Pls correct me. 

That's the correct line of thought. Kudos

2.3k views

With reference to counter-cyclical fiscal policy, consider the following statements: 

1. It is the strategy adopted to counter recession through fiscal measures. 

2. It aims at reducing taxes and increasing expenditure. 

Which of the statements given above is/are correct?

a.      1 only

b.        2 only

c.        Both 1 and 2

d.        Neither 1 nor 2

2.3k views

Gochisaid

With reference to counter-cyclical fiscal policy, consider the following statements: 

1. It is the strategy adopted to counter recession through fiscal measures. 

2. It aims at reducing taxes and increasing expenditure. 

Which of the statements given above is/are correct?

a.      1 only

b.        2 only

c.        Both 1 and 2

d.        Neither 1 nor 2

C?

That's correct. 

It is the strategy adopted to counter recession or boom through fiscal measures. During recession, the Counter cyclical fiscal policy aims at reducing taxes and increasing expenditure. It aims to create demand in the country to bring an upswing in the country. On the other hand, during boom in an economy, the counter-cyclical fiscal policy aims at increasing taxes and reducing public expenditure. 

2.3k views

Consider the following: 

1. Mesas and Buttas are associated with arid areas. 

2. Peneplains are associated with humid conditions. 

3. Dissected plateaux are associated with humid areas. 

Which of the statements given above is/are correct? 

a. 1 only 

b. 2 only 

c. 1 and 2 only 

d. 1, 2 and 3


2.3k views

The WTO regulations include

1. removal of import tariffs.

2. removal of price controls.

3. removal of industrial licenses.

Select the correct answer using the codes given below.

a.1 only

b. 1 and 2 only

c. 1 and 3 only

d. 1, 2 and 3

2.3k views

Small hydropower plants are those which have a capacity of upto

a. 10 MW

b. 25 MW

c. 50 MW

d. 100 MW

2.3k views

Which of the following Buddhist centres had direct connection with Buddha during his lifetime?  

   1. Uruvela  

  2. Sanchi  

  3. Sarnath  

  4. Rajagriha  

Select the correct answer using the codes given below.  

a. 3 and 4 only  

b. 1, 3 and 4 only   

c. 2 , 3 and 4 only  

d. 1, 2, 3 and 4 only

2.3k views

Chindu Bhagavatam/Yakshaganam is traditional theater art form that belongs to which of the following region.

a)Karnataka 

b)Lakshwadeep 

c)Telangana 

d)Tamil Nadu

A

Correct Answer is C

2.3k views

The creation of the legislative council of

states needs a constitutional amendment.=why is this statement incorrect?

Need some clarification....An amendment is not necessarily always under 368...formation of legis.council needs simple majority yet it is considered to be an amendment right?

When a legislative council is created or abolished, the Constitution of India is also changed. However, still, such type of law is not considered a Constitution Amendment Bill under Article 368.

2.3k views


can somebody clarify reg. the stretch of the eastern ghats...is it from odisha to tamilnadu or from West Bengal to tamilnadu?

The Eastern Ghats stretch from the Mahanadi Valley to the Nigiris in the south according to NCERT. 

Mahanadi Valley - Orissa. 

SO Orissa to TN

2.3k views

Consider the following statements:

1. Cyanobacteria are found in both fresh and marine waters.

2. Cyanobacteria cannot perform photosynthesis. 

3. Cyanobacterial biomass provides a nitrogen source in the form of proteins.

Which of the statements given above is/are correct?

a. 1 and 3 only

b. 1 and 2 only

c. 1 only

d. 2 and 3 only

2.3k views

Gochisaid

Consider the following statements:

1. Cyanobacteria are found in both fresh and marine waters.

2. Cyanobacteria cannot perform photosynthesis. 

3. Cyanobacterial biomass provides a nitrogen source in the form of proteins.

Which of the statements given above is/are correct?

a. 1 and 3 only

b. 1 and 2 only

c. 1 only

d. 2 and 3 only

A

cyanobac can be found anywhere so 1st is correct. they can perform photosynthesis. 3rd option they are nitrogen fixing bacteria,but "in the form of proteins" seems unlikely, so,C?

No, correct answer is C. 


https://www.pib.gov.in/PressReleseDetail.aspx?PRID=1604584


Cyanobacteria (also known as blue-green algae), too can perform photosynthesis and produce sugar by fixing the carbon dioxide in the atmosphere. The yield of sugars from cyanobacteria could potentially be much higher than that of land-based crops. Further, unlike plant-based sugars, cyanobacterial biomass provides a nitrogen source in the form of proteins.

2.3k views

Gochisaid

Small hydropower plants are those which have a capacity of upto

a. 10 MW

b. 25 MW

c. 50 MW

d. 100 MW

B ?

B.

Correct. 

Ministry of New and Renewable Energy has been vested with the responsibility of developing Small Hydro Power (SHP) projects up to 25 MW station capacities. The estimated potential for power generation in the country from such plants is about 20,000 MW. Most of the potential is in Himalayan States as river-based projects and in other States on irrigation canals. The SHP programme is now essentially private investment driven. Projects are normally economically viable and private sector is showing lot of interest in investing in SHP projects.

2.3k views

Consider the following: 

1. Mesas and Buttas are associated with arid areas. 

2. Peneplains are associated with humid conditions. 

3. Dissected plateaux are associated with humid areas. 

Which of the statements given above is/are correct? 

a. 1 only 

b. 2 only 

c. 1 and 2 only 

d. 1, 2 and 3


1 and 3 seem correct,so D?

Correct.

Mesas and Buttas are associated with arid areas, whereas Dissected plateaux are associated  with humid areas. Peneplains are associated with humid conditions, whereas Pediplains are associated with  arid and semi-arid conditions.

2.4k views

The WTO regulations include

1. removal of import tariffs.

2. removal of price controls.

3. removal of industrial licenses.

Select the correct answer using the codes given below.

a.1 only

b. 1 and 2 only

c. 1 and 3 only

d. 1, 2 and 3

B?

Incorrect. 

Correct Answer is D. 


WTO aims for free trade. Anything that hampers free trade is considered a barrier(either tariff or non-tariff) by WTO. Price controls and industrial license are non-tariffbarriers that the WTO aims to remove. Imports tariffs is a tariff barrier to tradewhich again the WTO aims to remove. 

2.3k views

Gochisaid

Consider the following statements:

1. Cyanobacteria are found in both fresh and marine waters.

2. Cyanobacteria cannot perform photosynthesis. 

3. Cyanobacterial biomass provides a nitrogen source in the form of proteins.

Which of the statements given above is/are correct?

a. 1 and 3 only

b. 1 and 2 only

c. 1 only

d. 2 and 3 only

A

cyanobac can be found anywhere so 1st is correct. they can perform photosynthesis. 3rd option they are nitrogen fixing bacteria,but "in the form of proteins" seems unlikely, so,C?

No, correct answer is C. 


https://www.pib.gov.in/PressReleseDetail.aspx?PRID=1604584


Cyanobacteria (also known as blue-green algae), too can perform photosynthesis and produce sugar by fixing the carbon dioxide in the atmosphere. The yield of sugars from cyanobacteria could potentially be much higher than that of land-based crops. Further, unlike plant-based sugars, cyanobacterial biomass provides a nitrogen source in the form of proteins.

if statement 3 is correct,then the answer should be 1 and 3,i.e.option A?

Cyanobacteria can perform photosynthesis...

2.3k views

Which of the following animals have been extinct in wild from India?

1.        Ostrich

2.        Cheetah

3.        Bengal florican

4.        Nilgai

Select the correct answer using the code given below.

a.        1 and 3 only

b.        1 and 2 only

c.        1, 2 and 4 only

d.        2 and 4 only

2.3k views

Which among the following would be included to calculate GDP of a country? 

1. Sale and purchase of stocks

2. Government social security and welfare payments

3. Work done by homemakers for their families.

Select the correct answer using the code given below.

a. 1 and 2 only

b. 2 and 3 only

c. 1 and 3 only

d. None

2.3k views

Which among the following would be included to calculate GDP of a country? 

1. Sale and purchase of stocks

2. Government social security and welfare payments

3. Work done by homemakers for their families.

Select the correct answer using the code given below.

a. 1 and 2 only

b. 2 and 3 only

c. 1 and 3 only

d. None

D?

D Is Correct. 

A product will only be counted in GDP one time in its life. So, current transactions involving assets and property produced in previous periods are not counted in the current GDP. For instance, if a car produced in the year 2000 is resold in 2015, the GDP of 2015 will not include the resale value of the laptop because this is a mere transfer of ownership entailing no creation of new value. 

Purely financial transactions, such as the purchase of stocks, bonds, or certificates of deposit, are not counted. We exclude these items because they are not examples of production of goods and services. These transactions simply involve the transfer of assets from one entity to another. Broker services for trading stocks are included since broker services are newly produced services, part of a nation's GDP. 

Other things not included in the GDP are government social security and welfare payments. Since GDP measures the market values of goods and services, economic activities that do not pass through the regular market channels are excluded in the computation of GDP.                                                                                         

Economic activity that is not part of GDP estimates is unpaid labor, services that people provide for free. The three most important sources of unpaid labor are professionals, volunteers, and homemakers. Think about the massive amount of work done by homemakers for their families. When a family pays a cleaning person, a nanny, or a cook, it is considered part of the GDP. However, when people devote time and effort to cleaning their own homes, taking care of their own children, and cooking for their own families, it is not counted, even though such labor has significant value.

2.2k views

Subsidies for which of the following are considered as non-merit subsidies?

1. Elementary education

2. Solar power plants

3. Fertiliser

Select the correct answer using the code given below.

a. 2 and 3  only

b. 1 and 3 only

c. 1 and 2  only

d. 3 only


2.2k views

Consider the following statements regarding Deocha Pachami:

1. Deocha Pachami is among the biggest iron mines in the world.

2. Deocha Pachami is located in Chattisgarh. 

3. Deposits are at a great depth of approx,. 300m.

Which of the above statements is/are correct?

A. 3 only

B. 1 and 2 only

C. 1and 3 only

D. 1, 2 and 3


2.2k views

The term e-DNA refers to

A. Evolved DNA

B. Environmental DNA

C. Electronic DNA

D. Embedded DNA


2.2k views
@Mzp662829541 that's correct. 

Environmental DNA (eDNA) refers to DNA that can be extracted from environmental samples, such as water, soil or faeces, without first isolating any target organisms.

eDNA is characterized by a complex mixture of nuclear, mitochondrial or chloroplast DNA, and can be intracellular (from living cells) or extracellular. It enables the detection of a species regardless of its life stage or gender.

2.2k views

Consider the following statements regarding Deocha Pachami:

1. Deocha Pachami is among the biggest iron mines in the world.

2. Deocha Pachami is located in Chattisgarh. 

3. Deposits are at a great depth of approx,. 300m.

Which of the above statements is/are correct?

A. 3 only

B. 1 and 2 only

C. 1and 3 only

D. 1, 2 and 3


C?

Incorrect. Answer is A. 

Deocha Pachami is a coal mine - it will eliminate all the other options. 

It's in West Bengal.

2.6k views

Which dance form had greatly inspired the Kerala mural paintings? 

A. Kathakali 

B. Mohiniattam 

C. Kuchipudi 

D. Bharatnatyam

2.6k views


Consider the following: 

(1) Unwritten Constitution 

(2) Independent Judiciary 

(3) Division of Powers 

Which of the following is/are essential features of federalism? 

A. 1 only 

B. 2 and 3 only 

C. 1 and 3 only 

D. 1, 2 and 3 


2.6k views

Which dance form had greatly inspired the Kerala mural paintings? 

A. Kathakali 

B. Mohiniattam 

C. Kuchipudi 

D. Bharatnatyam

A Kathakali

Correct

2.6k views


Consider the following: 

(1) Unwritten Constitution 

(2) Independent Judiciary 

(3) Division of Powers 

Which of the following is/are essential features of federalism? 

A. 1 only 

B. 2 and 3 only 

C. 1 and 3 only 

D. 1, 2 and 3 


B. 2 and 3 only

Correct

2.6k views

With reference to the Union Executive, consider the following statements:

1. The Union Executive consists only of the President, Vice President and the Prime Minister.

2. Under the Indian Constitution, the political executive is only legally constituted.

3. The political executive in India is subjected to only legislative control.

Which of the statements given above is/are correct?

A. 1 and 2 

B. 3 only

C. 1, 2 and 3

D. None of the above

2.6k views

Which of the following countries border the Persian Gulf? 

1.Kuwait

2. Iran

3. Yemen

4. Oman

Select the correct answer using the code given below:

A. 1, 2 and 4 only

B. 2, 3 and 4 only

C. 1, 3 and 4 only

D. 1, 2, 3 and 4

2.6k views

Strachey Commission was appointed in the aftermath of which famine?

a. The Madras Famine of 1782

b. The Guntur Famine of 1833

c. The Great Famine of 1876

d. The Bengal Famine of 1942

2.6k views

Who established Swadeshi Vastu Pracharini Sabha to carry the message of Swadeshi among masses?

a. Bal Gangadhar Tilak

b. Surendranath Banerjee

c. Bipin Chandra Pal

d. Lala Lajpat Rai

2.6k views

Which of the following is/are considered for calculating Minimum Support Price?

(1) Demand and supply

(2) Cost of production

(3) Inter-crop price parity

Select the correct answer using the code given below:

A. 1 and 3 only

B. 2 only

C. 2 and 3 only

D. 1, 2 and 3

2.6k views

Which of the following isnota part of the IMF quota formula?

a. Gross National Product

b. Openness 

c. Economic Variability 

d. International Reserves

2.6k views

Which of the following animals have been extinct in wild from India?

1.        Ostrich

2.        Cheetah

3.        Bengal florican

4.        Nilgai

Select the correct answer using the code given below.

a.        1 and 3 only

b.        1 and 2 only

c.        1, 2 and 4 only

d.        2 and 4 only

nilgais are found in india,and ostrich are extinct,cheetah were to be reintroduced in india i guess ,so B?

Correct. 

Indian researchers have found molecular evidence to confirm the presence of ostriches in India more than 25,000 years ago. Ostrich eggshell pieces in India, mostly in Rajasthan and Madhya Pradesh were studied. However, the bird is extinct in wild now in India. 

Cheetah was declared extinct in 1952. 

Bengal Florican is a rare bustard species that is very well known for its mating dance. It is rarely spotted in Uttar Pradesh, Assam, Arunachal Pradesh. 

Nilgai, also known as Blue Bull, is one of the most commonly found wild animals of northern India. Even though it is an antelope, it looks quite similar in appearance to an ox.

2.6k views

Subsidies for which of the following are considered as non-merit subsidies?

1. Elementary education

2. Solar power plants

3. Fertiliser

Select the correct answer using the code given below.

a. 2 and 3  only

b. 1 and 3 only

c. 1 and 2  only

d. 3 only


Ans. D

D?

Correct. 

Merit goods are those goods and services that the government feels should be subsidised or provided free at the point of use so that consumption does not depend primarily on the ability to pay for the good or service. Merit goods and services create positive externalities. 

Non merit goods are more harmful to consumers than they realize and they generate negative externalities. They are generally directed towards specific groups. Subsidy on the following are classified as merit or non merit subsidy.

Merit: Elementary education, primary health centres, prevention and control of diseases, and ecology and environment. Education (other than elementary), family welfare, sanitation services, welfare of SC, ST and OBCs, social welfare and nutrition, food for those below poverty line (BPL), plant protection, social and farm forestry, rural development, major and medium irrigation (non commercial), non-conventional sources of energy (other than atomic energy), and village and small industries.

Non-Merit: Irrigation (commercial), energy (power), coal and lignite, iron and steel industries, cement and non-metallic industries, petrochemical industries, chemical and pharmaceutical industries, dairy development, engineering industries, transport equipment industries, telecommunications and electronic industries, consumer industries, shipping, civil aviation, road transport, and postal services, food for those above poverty line.

2.6k views

PCFsaid


Which of the following isnota part of the IMF quota formula?

a. Gross National Product

b. Openness 

c. Economic Variability 

d. International Reserves

A I think

Correct. 

The IMF uses a quota formula to help assess a member's relative position. The current quota formula is a weighted average of GDP (weight of 50 percent), openness (30 percent), economic variability (15 percent), and international reserves (5 percent).

2.6k views

With reference to LPG gas consider the following 

  1. It is a byproduct obtained from the refining of crude oil
  2. It is an odourless gas

Which of the above statments areincorrect?

A. 1 only

B. 2 only

C. Both 1 & 2

D. Neither 1 nor 2

D

2.2k views

Which among the following schemes aim at reducing Gender Gaps in School Education?

a. Samagra Shiksha

b. Saakshar Bharat

c. Kishori Shakti Yojana

d. Laadli 

2.2k views

The quarterly report Inflation Expectations Survey of Households (IESH) is released by 

a. NSSO 

b. RBI

c. Department of Consumer Affairs

d. NITI Ayog 


2.2k views

Who among the following are most likely to gain during inflation?

a. Pensioners

b. People living on fixed savings

c. Borrowers repaying old loans

d. Creditors

Q72. Recently the Central Statisti

2.2k views

With reference to the Ghadar movement, consider the following statements:

1. The movement was strongly secular.

2. The Ghadar Movement failed to generate an effective and sustained leadership that was capable of integrating the various aspects of the movement.

3. Objective of Ghadarites was the establishment of an Independent Republic of India.

Which of the statements given above is/are correct?

a. 1 and 2 Only

b. 1 and 3 Only

c. 2 and 3 Only

d. All of the above


2.2k views

White Cheeked Macaque is aprimate species found in which one of the following states of India?

a. Assam

b. Mizoram

c. Manipur

d. Arunachal Pradesh

2.2k views

Gochisaid

Which among the following schemes aim at reducing Gender Gaps in School Education?

a. Samagra Shiksha

b. Saakshar Bharat

c. Kishori Shakti Yojana

d. Laadli 

A?

Correct

2.2k views

White Cheeked Macaque is aprimate species found in which one of the following states of India?

a. Assam

b. Mizoram

c. Manipur

d. Arunachal Pradesh

D. Arunachal Pradesh

Right. 

2.2k views

With reference to South Asia Growth Quadrangle (SAGQ), its composition is 

a. Bangladesh, Bhutan, India and Nepal

b. Myanmar, Bangladesh, Pakistan and Sri Lanka

c. Thailand, Myanmar, Bangladesh and Sri Lanka

d. Maldives, Sri Lanka, India and Pakistan

2.2k views

With reference to the Ghadar movement, consider the following statements:

1. The movement was strongly secular.

2. The Ghadar Movement failed to generate an effective and sustained leadership that was capable of integrating the various aspects of the movement.

3. Objective of Ghadarites was the establishment of an Independent Republic of India.

Which of the statements given above is/are correct?

a. 1 and 2 Only

b. 1 and 3 Only

c. 2 and 3 Only

d. All of the above


D??

Not sure

D is correct. 

The movement was strongly secular in nature. Concern with religion was seen as petty and narrow-minded, and unworthy of revolutionaries. Leaders belonging to, different religions and regions were accepted by the movement.Another marked feature of Ghadar ideology was its democratic and egalitarian content. It was clearly stated by the Ghadarites that their objective was the establishment of an independent republic of India.The Ghadar Movement also failed to generate an effective and sustained leadership that was capable ofintegrating the various aspects of the movement. Har Dayal himself was temperamentally totally unsuited to the role of an organizer; he was a propagandist, an inspirer, an ideologue.

2k views

@Eclectic said


upsc2020 said


Who among the following are most likely to gain during inflation?


a. Pensioners


b. People living on fixed savings


c. Borrowers repaying old loans


d. Creditors


Answer C - Correct

2k views

Which of the following country has coastline 

with Caspian Sea and Black Sea both?

a. Russia

b. Turkey

c. Iran

d. Georgia

2k views

With reference to South Asia Growth Quadrangle (SAGQ), its composition is 

a. Bangladesh, Bhutan, India and Nepal

b. Myanmar, Bangladesh, Pakistan and Sri Lanka

c. Thailand, Myanmar, Bangladesh and Sri Lanka

d. Maldives, Sri Lanka, India and Pakistan

A

Correct. 

The South Asia Growth Quadrangle (SAGQ) was launched in April 1997 by the Foreign Ministers of Bangladesh, Bhutan, India, and Nepal (BBIN). The Ninth Summit of the South Asian Association for Regional Cooperation (SAARC) on May 1997 in Male, Maldives endorsed SAGQ as a subregional initiative under SAARC. 

2k views

With reference to GISAID initiative, consider the following statements:

1. It promotes the international sharing of influenza virus sequences.

2. It is solely privately funded. 

Which of the statements given above is/are correct?

a. 1 only

b. 2 only

c. 1 and  2 only

d. Neither 1n or 2

2k views

Grinika Programme is associated with which country?

a. Uganda

b. Algeria

c. Sudan

d. Rwanda

2k views

Grinika Programme is associated with which country?

a. Uganda

b. Algeria

c. Sudan

d. Rwanda

D. Rwanda

That's correct. PM Had gifted cows here. 

2k views

With reference to GISAID initiative, consider the following statements:

1. It promotes the international sharing of influenza virus sequences.

2. It is solely privately funded. 

Which of the statements given above is/are correct?

a. 1 only

b. 2 only

c. 1 and  2 only

d. Neither 1n or 2

A??

Correct.

It is a PPP model. 

2k views

Which among the following tributaries of river Indus originate in India?

1.        Sutlej

2.        Jhelum

3.        Ravi

4.        Beas

5.        Chenab

Select the correct answer using the code given below.

a.       1, 2, 3, 4 and 5

b.        1, 2, 3 and 4 only

c.        2, 4 and 5 only

d.        2, 3, 4 and 5 only


2k views

Which among the following is/are secondary air pollutants?

1.        Ground Level Ozone

2.        Particulate Matter

3.        Smog

Select the correct answer using the code given below.

a.     1 and 3 only

b.        2 and 3 only

c.        3 only

d.        1 and 2 only


2k views

 Consider the following statements:

1. Tax avoidance refers to avoiding payment of tax through illegal means or fraud.

2. Tax evasion refers to avoiding payment of taxes by taking advantage of loop holes in tax laws. 

Which of the statements given above is/are correct?

a.        1 only

b.        2 only

c.        Both 1 and 2

d.        Neither 1 nor 2

2k views

The Chinese traveller Fa Xian who visited India recorded the some historical informations about India at that time. In this context, which of the following statements is/are correct?

1. There was no religious persecution and cordial relations existed between Hindus and Buddhists.

2. The untouchables were made to live outside the city.

3. He was robbed and molested frequently during his travel in the cities. 

Select the correct answer using the code given below.

a.  1 only

b.  1 and 2 only

c.  2 and 3 only

d.  1, 2 and 3


2k views

Which among the following tributaries of river Indus originate in India?

1.        Sutlej

2.        Jhelum

3.        Ravi

4.        Beas

5.        Chenab

Select the correct answer using the code given below.

a.       1, 2, 3, 4 and 5

b.        1, 2, 3 and 4 only

c.        2, 4 and 5 only

d.        2, 3, 4 and 5 only


D.

D

Correct. 

  Satluj rises from the Manasarovar-Rakas Lakes in western Tibet within 80 km of the source of the Indus.

 Jhelum has its source in a spring at Verinag in the south-eastern part of the Kashmir Valley.

Ravi has its source in Kullu hills near the Rohtang Pass in Himachal Pradesh. 

The origin of Beas is in the southern side of Rohtang Pass in the Pir Panjal mountain range. Beas Kund Lake, at an altitude of 13,326 feet, is the main source that feeds the river. 

The river Chenab (or Chandra Bhaga) is formed after the two streams the Chandra and the Bhaga merge with each other. The Chandra and the Bhaga originate from the south-west and north-west faces of Barelacha pass respectively in the Himalayan canton of Lahul and Spiti valley in Himachal Pradesh. 

2.1k views

1611said


 Consider the following statements:

1. Tax avoidance refers to avoiding payment of tax through illegal means or fraud.

2. Tax evasion refers to avoiding payment of taxes by taking advantage of loop holes in tax laws. 

Which of the statements given above is/are correct?

a.        1 only

b.        2 only

c.        Both 1 and 2

d.        Neither 1 nor 2

D. Neither 1 nor 2


Correct. 

The terms "tax avoidance" and "tax evasion" are often used interchangeably, but they are very different concepts. Tax avoidance is legal, while tax evasion is not. Tax avoidance is the minimizing of taxes by taking advantage of loop holes in law. Tax evasion is the illegal act or practice of failing to pay taxes which are owed. Tax evasion is undertaken by employing unfair means. 

2.1k views

The Chinese traveller Fa Xian who visited India recorded the some historical informations about India at that time. In this context, which of the following statements is/are correct?

1. There was no religious persecution and cordial relations existed between Hindus and Buddhists.

2. The untouchables were made to live outside the city.

3. He was robbed and molested frequently during his travel in the cities. 

Select the correct answer using the code given below.

a.  1 only

b.  1 and 2 only

c.  2 and 3 only

d.  1, 2 and 3


B.??

C?

B is the correct answer. 

Fa Xian gave an account of the administration of Chandragupta Vikramaditya. The accounts of Fa Xian give a clear indication that India was probably never governed better than the era of Chandragupta Vikramaditya. Fa Hien accounts show that Buddhism flourished in parts but the Brahmanical faith was predominant. There was no religious persecution and cordial relations existed between Hindus and Buddhists. 

Fa Xian noticed the condition of those who were treated as ‘untouchable’ by the powerful people. They made them live outside the city. He writes: “If such a man enters a town or a market place, he strikes a piece of wood, in order to keep himself separate; people, hearing this sound, know what it means and avoid touching him or brushing against him.”

Fa-Hein, who travelled in India for 11 years, recorded that he was never molested or robbed. While Huen Tsang narrated that he was attacked by robbers on the way, something which we don’t find in the narrations of Fa Hien who travelled in the Gupta Period. It shows that the administrative machinery deteriorated after the Gupta period. 

2.1k views

With reference to the sedition act of 1870, consider the following statements:

1. Under the act, any person protesting or criticising the government could be arrested without due trial.

2. Mahatma Gandhi was jailed in 1922 on sedition charges under this act.

Which of the statements given above is/are correct?

a. 1 only

b. 2 only

c. Both 1 and 2

d. Neither 1 nor 2

2.1k views

Which of the following statements about “Deep Ocean Mission” is not correct? 

A. A major thrust of the Mission is on the search of metals and rare minerals. 

B. It comprises an offshore desalination plant that will work with tidal energy and 

developing a submersible vehicle. 

C. It is a pilot project under a larger Samudrayaan project for deep ocean mining of 

rare minerals. 

D. It is to be led by the Union Earth Sciences Ministry.

2.1k views

Consider the following multilateral groups: 

1. G4

2. G7

3. G20 

India is not a part of which of the above groups? 

A. 1 and 2 only 

B. 2 only 

C. 1 and 3 only 

D. 1 only 


2.1k views

Which of the following isnota risk of raising overseas sovereign bonds? 

A. Crowding out of Private sector 

B. Rupee Appreciation

C. Negative impact on exports

D. External Debt becoming expensive due to volatility in the currency 


2.1k views

With reference to Hydroponics, consider the following statements:

1. Soil is replaced by a water solution that is rich in macro-nutrients.

2. Water consumption is very high in hydroponics farming.

Which of the statements given above is/are correct?

a. 1 only

b. 2 only

c. Both 1 and 2

d. Neither 1 nor 2


2.1k views

Lower Circuits, a term recently in news, is in reference to

a. F1 racing

b. Fusion Energy

c. Share Market

d. None of the above


2.1k views

Which among the following diseases outbreaks have been declared as pandemics?

1. Spanish Flu, 1918

2. H1N1, 2009

3. Ebola, 2014

Select the correct answer using the code given below.

a.  1 and 3 only

b.  1 and 2 only

c.  2 and 3 only

d.  1, 2 and 3

2.1k views

With reference to the sedition act of 1870, consider the following statements:

1. Under the act, any person protesting or criticising the government could be arrested without due trial.

2. Mahatma Gandhi was jailed in 1922 on sedition charges under this act.

Which of the statements given above is/are correct?

a. 1 only

b. 2 only

c. Both 1 and 2

d. Neither 1 nor 2

C?

C?

Correct. 

Gandhiji was arrested for his articles in Youth India. 

2.2k views

Consider the following multilateral groups: 

1. G4

2. G7

3. G20 

India is not a part of which of the above groups? 

A. 1 and 2 only 

B. 2 only 

C. 1 and 3 only 

D. 1 only 


A?

Correct. 

2.2k views

With reference to Hydroponics, consider the following statements:

1. Soil is replaced by a water solution that is rich in macro-nutrients.

2. Water consumption is very high in hydroponics farming.

Which of the statements given above is/are correct?

a. 1 only

b. 2 only

c. Both 1 and 2

d. Neither 1 nor 2


D?

Wrong. Statement 1 is correct. Water consumption is low in hydroponics farming.

2.2k views

Which among the following diseases outbreaks have been declared as pandemics?

1. Spanish Flu, 1918

2. H1N1, 2009

3. Ebola, 2014

Select the correct answer using the code given below.

a.  1 and 3 only

b.  1 and 2 only

c.  2 and 3 only

d.  1, 2 and 3

B i guess.

Correct. 

2.2k views

Lower Circuits, a term recently in news, is in reference to

a. F1 racing

b. Fusion Energy

c. Share Market

d. None of the above


C.

Correct.

2.2k views

Which of the following isnota risk of raising overseas sovereign bonds? 

A. Crowding out of Private sector 

B. Rupee Appreciation

C. Negative impact on exports

D. External Debt becoming expensive due to volatility in the currency 


C?

Incorrect. 

The question asks not a risk. 

Borrowing overseas allows the government to raise funds in such a way that there is enough domestic credit available for the private sector. It, therefore, reduces the crowding-out effect.

A is the right answer. 

2.2k views

Which of the following correctly explains the term “Bond yield inversion”?

A. Riskier bonds yield more than less risky bonds

B. Riskier bonds yield less than less risky bonds

 C.Long term bonds yield more than short term bonds 

D. Long term bonds yield less than short term bonds.

2.2k views

 Dolutegravir and Efavirenz recently in the news, refer to 

A. Nipah virus Drugs 

B. Vaccines for Measles and Rubella 

C. HIV Drugs 

D. Vaccinesfor Japanese Encephalitis 


2.2k views

Which of the following correctly explains the term “Bond yield inversion”?

A. Riskier bonds yield more than less risky bonds

B. Riskier bonds yield less than less risky bonds

 C.Long term bonds yield more than short term bonds 

D. Long term bonds yield less than short term bonds.

D

That's correct 

2.1k views

Consider the following multilateral groups: 

1. G4

2. G7

3. G20 

India is not a part of which of the above groups? 

A. 1 and 2 only 

B. 2 only 

C. 1 and 3 only 

D. 1 only 


A?

Correct. 

It should be B

India is a part of G4

Right... 

2.1k views

 Dolutegravir and Efavirenz recently in the news, refer to 

A. Nipah virus Drugs 

B. Vaccines for Measles and Rubella 

C. HIV Drugs 

D. Vaccinesfor Japanese Encephalitis 


C

That's correct. 

2.1k views

Consider the following statements with regard to the BandipurTiger Reserve.

1. It is also a National Park and a part of Nilgiri Biosphere Reserve.

2. It has a variety of biomes - dry deciduous forests, moist deciduous forests and shrub lands.

3. The river Kabini runs through the Bandipur Tiger Reserve.

Select the correct answer using the codes given below

A. 1 only

B. 1 and 2 only

C. 2 and 3 only

D. 1, 2 and 3 only

2.1k views

ARS21said

does LTRO come under OMO?

Yes, it's a type of OMO

2.1k views

Consider the following statements with regard to the BandipurTiger Reserve.

1. It is also a National Park and a part of Nilgiri Biosphere Reserve.

2. It has a variety of biomes - dry deciduous forests, moist deciduous forests and shrub lands.

3. The river Kabini runs through the Bandipur Tiger Reserve.

Select the correct answer using the codes given below

A. 1 only

B. 1 and 2 only

C. 2 and 3 only

D. 1, 2 and 3 only

B??

That's correct. River Nugu passes through the park. Kabinu flows around it. 

2.1k views

With reference to the ‘LADIS’ – Least Available Depth Information System, consider the following statements

1. LADIS will ensure that real-time data on least available depths is disseminated for ship/barge and cargo owners so that they can undertake transportation on National Waterways.

2. It is designed by the Inland Waterways Authority of India (IWAI) to facilitate the day-to-day operations of inland vessels.

Select the answer from statements using the codes given below

A. 1 Only

B. 2 Only

C. Both 1 and 2

D. Neither 1 nor 2

2.1k views

West Nile Virus (WNV) fever was reported in India. Consider the following statements with regard to the same. 

1. As per the World Health Organisation (WHO), the West Nile Virus (WNV) is a mosquito-borne disease.

2. Monkeys are the natural hosts of this virus.

3. There is no vaccination or specific treatment available for the virus. 

Select the correct statement using the codes given below.

A. 1 only

B. 1 and 2 only

C. 2 and 3 only

D. 1 and 3 only

2.1k views

Which among the following schemes may contribute to doubling farmer's incomes?

1. e-NAM scheme

2. Per drop more crop

3. Soil Health Card

Select the correct answer using the codes given below


A. 1 only


B. 1 and 2 only


C. 2 and 3 only


D. 1, 2 and 3 only

2.1k views

Which among the following are modes of transmission of Zoonotic diseases?

1. Vector Borne

2. Water Borne

3. Food Borne

Select the correct answer using the codes given below

A. 1 only

B. 1 and 2 only

C. 2 and 3 only

D. 1, 2 and 3 only

2.1k views

India has launched a pilot project to enhance the capacity of forest landscape restoration (FLR) in five states. In this context, consider the following statements

1. FLR is an ongoing process of regaining ecological functionality and enhancing human well-being across deforested or degraded forest landscapes

2. The project is also part of the Bonn Challenge pledge which was undertaken by India in 2015.

3. It is implemented by National Afforestation and Eco-Development Board (NAEB) in partnership with the IUCN

Select the answer using the codes given below

A. 1 only

B. 2 and 3 only

C. 1 and 3 only

D. 1, 2 and 3 only

2.1k views

With reference to Essential Commodities Act, 1955, consider the following statements:

1. The government cannot fix the maximum retail price (MRP) of any packaged product that it declares an “essential commodity”.

2. Department of Consumer Affairs administers The Essential Commodities Act, 1955.

3. The Act empowers the central government to add new commodities to the list of Essential Commodities as and when the need arises. 

Select the answer using the codes given below.

A. 1 only

B. 2 and 3 only

C. 1 and 3 only

D. 1, 2 and 3 only

2.1k views

Spocksaid

Who established Swadeshi Vastu Pracharini Sabha to carry the message of Swadeshi among masses?

a. Bal Gangadhar Tilak

b. Surendranath Banerjee

c. Bipin Chandra Pal

d. Lala Lajpat Rai

A?

That's correct. 

Tilak established Swadeshi V astu Pracharini Sabha, society to promote the use of Swadeshi goods and Swadeshi co-operative stores. The Swadeshi Vastu Pracharini Sabha organised an exhibition of Swadeshi goods, organised lectures, and gave prizes to best essays and the best Ganapati mela using Swadeshi attire.


2.1k views

Spocksaid


#25)With reference to the ‘tropical cyclones’, consider the following statements:

(1) They are known as Willy-willies in Western Australia and Hurricanes in the Atlantic Ocean.

(2) The cyclonic wind movements are clockwise in the northern hemisphere and anti-clockwise in the southern hemisphere

(3) The tropical cyclones are formed mostly on the western margins of the oceans.

Which of the statements given above are correct?

a. 1 and 2 only

b. 1 and 3 only

c. 2 and 3 only

d. 1, 2 and 3


B?

I am not too sure of Statement 3. Western margin of ocean ? Does it mean eastern side of landmass.

2 is definitely wrong.

2.1k views

With reference to Essential Commodities Act, 1955, consider the following statements:

1. The government cannot fix the maximum retail price (MRP) of any packaged product that it declares an “essential commodity”.

2. Department of Consumer Affairs administers The Essential Commodities Act, 1955.

3. The Act empowers the central government to add new commodities to the list of Essential Commodities as and when the need arises. 

Select the answer using the codes given below.

A. 1 only

B. 2 and 3 only

C. 1 and 3 only

D. 1, 2 and 3 only

B.

B.

Correct 

2.1k views

India has launched a pilot project to enhance the capacity of forest landscape restoration (FLR) in five states. In this context, consider the following statements

1. FLR is an ongoing process of regaining ecological functionality and enhancing human well-being across deforested or degraded forest landscapes

2. The project is also part of the Bonn Challenge pledge which was undertaken by India in 2015.

3. It is implemented by National Afforestation and Eco-Development Board (NAEB) in partnership with the IUCN

Select the answer using the codes given below

A. 1 only

B. 2 and 3 only

C. 1 and 3 only

D. 1, 2 and 3 only

A??

Not sure

D?shooting in the dark

D is correct. 

It is a flagship project which is a part of a larger international initiative called ‘Bonn Challenge’. 

The aim of the project is to develop and adapt best practices and monitoring protocols for the Indian states, and build capacity within the five pilot states on forest landscape restoration and Bonn Challenge.

The project launched in partnership with the National Afforestation and Eco-Development Board (NAEB), MoEFCC will eventually be scaled up across the country in subsequent phases.

2.2k views

Which among the following are modes of transmission of Zoonotic diseases?

1. Vector Borne

2. Water Borne

3. Food Borne

Select the correct answer using the codes given below

A. 1 only

B. 1 and 2 only

C. 2 and 3 only

D. 1, 2 and 3 only

D??

D.


Correct. ?

2.2k views

Which among the following schemes may contribute to doubling farmer's incomes?

1. e-NAM scheme

2. Per drop more crop

3. Soil Health Card

Select the correct answer using the codes given below


A. 1 only


B. 1 and 2 only


C. 2 and 3 only


D. 1, 2 and 3 only

D

D

Correct. ?

2.2k views

With reference to the ‘LADIS’ – Least Available Depth Information System, consider the following statements

1. LADIS will ensure that real-time data on least available depths is disseminated for ship/barge and cargo owners so that they can undertake transportation on National Waterways.

2. It is designed by the Inland Waterways Authority of India (IWAI) to facilitate the day-to-day operations of inland vessels.

Select the answer from statements using the codes given below

A. 1 Only

B. 2 Only

C. Both 1 and 2

D. Neither 1 nor 2

C?

A?

C is right. Both statements are correct. 

LADIS will ensure that real-time data on least available depths is disseminatedfor ship/barge and cargo owners so that they can undertake transportation on NWs in a more planned way. An assured depth of waterway is required for seamless movement of vessels. If real time information is made available regarding LADs in stretches of various NWs, it will help transporters by guiding them on the suitability of time of movement.

The portal being hosted on IWAI’s website www.iwai.nic.in has been developed in-house. Initially LAD information will be available for NW-1, NW-2, Indo-Bagladesh Protocol route and NW-3, along with the date of survey. The facility will be expanded to other NWs also.

IWAI has designed LADIS to facilitate the day to day operations of inland vessels plying on National Waterways and to avoid any hindrance in service and operation. It will enhance credibility and efficiency of information sharing to achieve seamless operations on National Waterways, besides pre-empting problems that may occur during movement of vessels.

2.2k views

Spocksaid

West Nile Virus (WNV) fever was reported in India. Consider the following statements with regard to the same. 

1. As per the World Health Organisation (WHO), the West Nile Virus (WNV) is a mosquito-borne disease.

2. Monkeys are the natural hosts of this virus.

3. There is no vaccination or specific treatment available for the virus. 

Select the correct statement using the codes given below.

A. 1 only

B. 1 and 2 only

C. 2 and 3 only

D. 1 and 3 only

D?

A?

D is right. 

Birds are the host. There's no vaccine available as of now. 

2.2k views

Bats are a unique reservoir of rapidly reproducing and highly transmissible viruses. Which among the following virus are Bat Borne?

1. Ebola

2. Lassa

3. Plague

4. Nipah

Select the correct answer using the code given below.

a. 1, 2, 3 and 4

b. 2, 3 and 4

c. 1, 2 and 4

d. 1 and 4

2.2k views

Bats are a unique reservoir of rapidly reproducing and highly transmissible viruses. Which among the following virus are Bat Borne?

1. Ebola

2. Lassa

3. Plague

4. Nipah

Select the correct answer using the code given below.

a. 1, 2, 3 and 4

b. 2, 3 and 4

c. 1, 2 and 4

d. 1 and 4

D?

That's correct. Lassa and Plague are from rodents. 

2.2k views

With reference to Indian freedom struggle, consider the following events :

1. Kanpur Conspiracy case

2. Meerut Conspiracy Case

3. Gandhi-Irwin Pact

What is the correct chronological sequence of the above events ?

a. 1-2-3

b. 2-1-3

c. 3-2-1

d. 3-1-2


2.2k views

With reference to economy, the term fiscal cliff refers to:

a.      Increased tax rates and reduced spending by the government.

b.        Increased tax revenues for the government due to inflation.

c.        Increase in fiscal deficit due to recession

d.        Large external borrowings to finance a balance of payment crisis.


2.2k views

Nuclear agriculture invovles the use of nuclear technology in agriculture. Which of the following are the applications of Nuclear Agriculture?

1.        Development of high yielding seed varieties

2.        Improvement of nutritional value of crops

3.        Controlling pests and insects

Select the correct answer using the code given below.

a.        1 and 2 only

b.        1 and 3 only

c.        2 and 3 only

d.        1, 2 and 3


2.2k views

Which among the following are the factors that can affects the ocean currents?

1.        Shape of coastlines

2.        Relief structures at ocean basin

3.        Rotation of the Earth

Select the correct answer using the code given below.

a.      1 and 3 only

b.        1 and 2 only

c.        2 and 3 only

d.        1, 2 and 3


2.2k views

With reference to Pre-Independence India 'Suhrawardy-Sarat Bose agreement' was

a.      the list of demands given to Lord Curzon after partition of Bengal.

b.        a representation in British Parliament against shifting of capital from Calcutta to Delhi.

c.        a pact stating that Bengal would be an undivided sovereign state after transfer of power.

d.        a peace agreement between Hindus and Muslims after Noakhali riots.


2.2k views

With reference to economy, the term fiscal cliff refers to:

a.      Increased tax rates and reduced spending by the government.

b.        Increased tax revenues for the government due to inflation.

c.        Increase in fiscal deficit due to recession

d.        Large external borrowings to finance a balance of payment crisis.


A.

A is correct. 


The term fiscal cliff refers to more than $500 billion tax increases and across-the-board spending cuts by the USA government. The increase in taxes is in response to the fiscal stimulus programme launched to tackle sub prime crisis. 

2k views

Nuclear agriculture invovles the use of nuclear technology in agriculture. Which of the following are the applications of Nuclear Agriculture?

1.        Development of high yielding seed varieties

2.        Improvement of nutritional value of crops

3.        Controlling pests and insects

Select the correct answer using the code given below.

a.        1 and 2 only

b.        1 and 3 only

c.        2 and 3 only

d.        1, 2 and 3


D

D is correct. 

Nuclear Agriculture is carried out by Bhabha Atomic Research Centre (BARC) to develop high yielding seed varieties by inducing mutations using Gamma radiation. Gamma radiation only accelerates the mutations which otherwise occurs naturally over a much longer periods of time. So far BARC released 42 seed varieties for commercial exploitation. Mutations can be in such a way that the crops mature early, to withstand biotic and abiotic stresses and to obtain better nutritional quality.

While insecticides/pesticides have had an impact, pests and insects are able to learn to resist certain treatments forcing greater quantities of insecticides/pesticides to be used. The development of the sterile insect technique (SIT) is the most proven and common method where nuclear technology has been utilized to control or eliminate insects.

1.9k views

Which among the following are the factors that can affects the ocean currents?

1.        Shape of coastlines

2.        Relief structures at ocean basin

3.        Rotation of the Earth

Select the correct answer using the code given below.

a.      1 and 3 only

b.        1 and 2 only

c.        2 and 3 only

d.        1, 2 and 3


D

D is correct. 

The factors that modify currents include:

•        Direction, shape and configuration of coastline- the disposition of coast line perpendicular to natural flow direction of ocean currents obstruct them. Ocean currents thus flow parallel to coastline. The equatorial current after being obstructed by Brazilian coast bifurcates into Gulf Stream and Brazilian current. 

•        Bottom reliefs - The irregularities of bottom reliefs modify the currents at the surface and bottom. The submarine ridges usually deflect the course of currents. For example, north equatorial current is deflected to the right while crossing mid Atlantic ridge. 

•        Seasonal Variation – There is seasonal change in direction of currents in some areas in response to seasonal change in weather conditions For example in the regions of monsoon climate the currents of the Indian Ocean show seasonal change in their flow directions. The monsoon drifts flow in north east direction under the influence of south west monsoon. They move east to west along the coast during north east monsoon. 


2k views

With reference to Pre-Independence India 'Suhrawardy-Sarat Bose agreement' was

a.      the list of demands given to Lord Curzon after partition of Bengal.

b.        a representation in British Parliament against shifting of capital from Calcutta to Delhi.

c.        a pact stating that Bengal would be an undivided sovereign state after transfer of power.

d.        a peace agreement between Hindus and Muslims after Noakhali riots.


C

C is correct. 

Hussain Shaheed Suhrawardy and Sarat Chandra Bose believed that division of Bengal between India and Pakistan, will ruin its culture and economy. The Suhrawardy-Sarat Bose agreement read thus:

•        Bengal would become an independent undivided sovereign state.

•        Bengal’s Constitution will be drafted by an assembly of 16 Muslims, 14 Hindus.

•        Formation of interim government.

•        Proportional representation of Hindus and Muslims in the legislature.


1.9k views

With reference to Indian freedom struggle, consider the following events :

1. Kanpur Conspiracy case

2. Meerut Conspiracy Case

3. Gandhi-Irwin Pact

What is the correct chronological sequence of the above events ?

a. 1-2-3

b. 2-1-3

c. 3-2-1

d. 3-1-2


A

A is correct. 

Kanpur Conspiracy Case (1924):— British government started the case against four communists-Muzaffar Ahmed, S.A.Dange, Shaukat Usmani and Nalni Gupta. The government alleged that the Communists wanted to deprive the British King of the sovereignty of British India.

Meerut Conspiracy Case: In March 1929, 31 labour leaders were arrested on the charge of conspiracy. The leaders included three Englishmen who had helped in the organization of the workers movement in India. They were taken to Meerut and were tried. The trial lasted for four years and is known as Meerut Conspiracy Case. Many defense committees were formed all over the country and even in England and other foreign countries. The nationalist leaders provided legal defense to the accused. Some of them were acquitted while others were convicted. The workers organizations had been growing and played an active part in the nationalist movement. The British government issued the Public Safety Ordinance in 1929 to remove from India persons it considered British and foreign communist agents.

Gandhi-Irwin Pact:This pact was signed between Mahatma Gandhi and the then Viceroy of India, Lord Irwin on 5 March 1931. The outcome of these talks was the Gandhi Irwin pact. They had eight meetings which lasted for a total of 24 hours. Gandhiji was impressed with Irwin’s sincerity and on behalf of the Indian National Congress agreed to discontinue the Civil Disobedience movement. The Congress agreed to join the second Round Table Conference to chalk out constitutional reforms. Some of the other conditions were that the British would withdraw all orders imposing curbs on the activities of the Indian National Congress. They also agreed to withdraw trials relating to several offences except those involving violence and release of prisoners arrested for participating in the civil disobedience movement. It was also agreed that the British would remove the tax on salt, which allowed Indians to produce, trade, and sell salt legally and for their own use.

1.9k views

Consider the following statements with regards to Global Solar Council. 

1. It has its headquarters in Washington DC.

2. It was founded in 2015 Paris climate Conference.

3. India’s National Solar Energy Foundation is a founding member of the Global Solar Council

Choose the correct answer using code below.

A. 1 and 2

B. 2 and 3

C.1 and 3

D. All statements are correct

1.9k views

Consider the following statements with regards to Global Solar Council. 

1. It has its headquarters in Washington DC.

2. It was founded in 2015 Paris climate Conference.

3. India’s National Solar Energy Foundation is a founding member of the Global Solar Council

Choose the correct answer using code below.

A. 1 and 2

B. 2 and 3

C.1 and 3

D. All statements are correct

B?

Correct Answer is D. 

Global Solar Council is the voice of the world’s solar energy industry, a non-profit body based in Washington D.C. representing national, regional and international associations as well as leading solar sector corporations.

NSEFI is a founding member of the Global Solar Council (GSC) which was launched at Paris during UN COP 21 Climate talks In December 2015-as a private sector response to global warming and Climate Change. GSC consists of over 30 countries solar associations in the world and further growing - with the avowed goal of Solar Growth and spread throughout the world. NSEFI has been playing leading and active role in Global Solar Council (GSC).

NSEFI Chairman Pranav R Mehta becomes the First Indian to Head Global Solar Council(GSC) .

1.9k views

Which of the following statements are correct with regards to Sarus cranes?

1. Sarus crane is a large non-migratory crane found in parts of the Indian Subcontinent, Southeast Asia and Australia.

2. It generally inhabits natural wetlands, and is an omnivorous bird.

3. It is India’s only resident breeding crane.

Choose the correct answer using the code given below.

A. 1 and 2

B. 2 and 3

C. 1 and 3

D. All statements are correct

1.9k views

Consider the following statements with regards to Proton Therapy.

1. It is a radiation therapy which uses protons rather than x-rays to treat cancer.

2. The therapy painlessly delivers radiation through the skin from a machine outside the body

3. Healthy cell loss is less in this therapy as less dose is given.

Choose the correct answer using the code given below.

A. 1 only

B. 1 and 2

C. 2 and 3

D. All statements are correct

1.9k views

Which of the following statements are correct with regards to Sarus cranes?

1. Sarus crane is a large non-migratory crane found in parts of the Indian Subcontinent, Southeast Asia and Australia.

2. It generally inhabits natural wetlands, and is an omnivorous bird.

3. It is India’s only resident breeding crane.

Choose the correct answer using the code given below.

A. 1 and 2

B. 2 and 3

C. 1 and 3

D. All statements are correct

D

Correct

1.9k views

Consider the following statements with regards to Proton Therapy.

1. It is a radiation therapy which uses protons rather than x-rays to treat cancer.

2. The therapy painlessly delivers radiation through the skin from a machine outside the body

3. Healthy cell loss is less in this therapy as less dose is given.

Choose the correct answer using the code given below.

A. 1 only

B. 1 and 2

C. 2 and 3

D. All statements are correct

D??

No, statement 3 is wrong..the dosage given is high. 

1.9k views

Bagru Block Painting was recently in news, it belongs to which state?

A. Maharashtra 

B. Rajasthan

C. Karnataka

D. Madhya Pradesh

1.9k views

The earliest Brahmanical paintings are found in

a. Ajanta Caves

b. Ellora Caves

c. Bagh caves

d. Badami Caves

2.1k views

Consider the following statements about organizations of World Bank Group:

1. International Development Association lends to creditworthy low-income countries.

2. International Finance Corporation focuses on private sector development of developing

countries.

Which of the statements given above is/are correct?

a. 1 only

b. 2 only

c. Both 1 and 2

d. Neither 1 nor 2

2.1k views

With regards to the regulation of minority institutions, consider the following statements.

1. Article 30 of the Constitution deals with the Right of minorities to establish and administer educational institutions.

2. An institution can be a ‘minority institution’ only on the basis of religion.

3. The National Commission for Minority Educational Institutions (NCMEI) Act defines MEI as a college or an educational institution established and administered by a minority or minorities

Select the answer using the codes given below.

A. 1 

B. 1 and 3

C. 2 and 3

D. 1 and 2

2.1k views

Bagru Block Painting was recently in news, it belongs to which state?

A. Maharashtra 

B. Rajasthan

C. Karnataka

D. Madhya Pradesh

B

Correct,

2.1k views

The earliest Brahmanical paintings are found in

a. Ajanta Caves

b. Ellora Caves

c. Bagh caves

d. Badami Caves

D

Correct. 

2.1k views

Consider the following statements about organizations of World Bank Group:

1. International Development Association lends to creditworthy low-income countries.

2. International Finance Corporation focuses on private sector development of developing

countries.

Which of the statements given above is/are correct?

a. 1 only

b. 2 only

c. Both 1 and 2

d. Neither 1 nor 2

C?

Incorrect. Both statements are wrong. 

2.1k views

Which of the following animals have been extinct in wild from India?

1.        Ostrich

2.        Cheetah

3.        Bengal florican

4.        Nilgai

Select the correct answer using the code given below.

a.        1 and 3 only

b.        1 and 2 only

c.        1, 2 and 4 only

d.        2 and 4 only

B

Correct.

2.1k views

Consider the following statements about organizations of World Bank Group:

1. International Development Association lends to creditworthy low-income countries.

2. International Finance Corporation focuses on private sector development of developing

countries.

Which of the statements given above is/are correct?

a. 1 only

b. 2 only

c. Both 1 and 2

d. Neither 1 nor 2

C?

Incorrect. Both statements are wrong. 

Please explain why both are wrong

Statement 1 is for IBRD.

Statement 2 can be correct. I actually intended to put it as private sector and NGO. 

The International Bank for Reconstruction and Development (IBRD) lends to governments of middle-income and creditworthy low-income countries.

The International Development Association (IDA) provides interest-free loans — called credits — and grants to governments of the poorest countries.

The International Finance Corporation (IFC) is the largest global development institution focused exclusively on the private sector. We help developing countries achieve sustainable growth by financing investment, mobilizing capital in international financial markets, and providing advisory services to businesses and governments.

The Multilateral Investment Guarantee Agency (MIGA) was created in 1988 to promote foreign direct investment into developing countries to support economic growth, reduce poverty, and improve people’s lives. MIGA fulfills this mandate by offering political risk insurance (guarantees) to investors and lenders.

The International Centre for Settlement of Investment Disputes (ICSID) provides international facilities for conciliation and arbitration of investment disputes.

2.1k views

With regards to National Housing Bank consider the following statements

1. It is a statutory body set up to operate as the principal agency to promote and regulate housing finance institutions.

2. It launched NHB Residex which is the first official residential housing price index in 2007.

3. RBI has divested 51 percent stakes in NHB to central government. 

Select the answer using the codes given below.


A. 1 and 2


B. 1 and 3


C. 2 and 3


D. 1 , 2 & 3

2.1k views

With regards to National Housing Bank consider the following statements

1. It is a statutory body set up to operate as the principal agency to promote and regulate housing finance institutions.

2. It launched NHB Residex which is the first official residential housing price index in 2007.

3. RBI has divested 51 percent stakes in NHB to central government. 

Select the answer using the codes given below.


A. 1 and 2


B. 1 and 3


C. 2 and 3


D. 1 , 2 & 3

A

That's correct. 


2.1k views
The words ‘Lulu’ and ‘Nana’ are sometimes mentioned in media in reference to
(a) Humanoid robots that can read expressions
(b) Twin comets passing by earth every 237 years
(c) New moons of Saturn that are recently discovered
(d) First genetically edited babies


Source: examshack.com

D

2.1k views
Consider the following statements:
1. It is an important Shakti shrine.
2. It was built by Ahom King Siva Singha in 1725 AD.
3. In the garbhagriha there is no idol and a small pit filled with water is considered as the Goddess.
Which temple does the statements given above describe?
(a) Kamakhya Temple, Guwahati, Assam.
(b) Maa Bhairavi Temple, Berhampur, Odisha.
(c) Ugra Tara Temple, Guwahati, Assam .
(d) Kalighat Kali Temple, Kolkata, West Bengal

Source: examshack.com

A?

2.1k views

Q. Operation Thirst, sometimes seen in news, aims to

(a) Facilitate the States in mobilizing the resources for expanding coverage of Micro Irrigation.

(b) Enhance crop per drop by implementing water-saving technologies and precision irrigation.

(c) Crack down selling of unauthorized Packaged Drinking Water.

(d) None of the above

C

2.1k views

Q-1 Consider the following statments about "United Nations" 

1) The name "United Nations", coined by British PM Winston Churchill was first used in the Decleration of United Nations of 1 January 1942, during the Second World War.

2)  UN was founded as successor of the " League of Nations ".

3)  India attended the San Francisco Conference of 1945, its delegation led by Sir C.P. Ramaswamy Mudaliar

Which of the statements given above is/are correct? 

A) 1 and 2

B) 2 and 3

C) 3 only

D) All of the above.


D?

2k views

Q- 20 Consider the following statements about the veto power.

1. Only the permanent members of the Security Council possess the veto power.

2. It’s a kind of negative power.

3.The Secretary-General uses this power when not satisfied with any decision.

4. One veto can stall a Security Council resolution.

A) 1 and 3 only

B)  2 and 3 

C) 1, 2 and 4

D) All of the above


D? However, statements are a tad ambiguous. 

2k views

Consider the following statements:

  1.  Naupada which is famous for salt fields is located in Odisha
  2. Narayan Sarovar Sanctuary, which is identified as one of the last remaining habitats of the cheetah in India, is located in the Rann of Kutch region

Which of the above statements is/are correct?

a. 1 only

b. 2 only

c. 1 and 2 Both

d. Neither 1 nor 2

2k views

To which wildlife sanctuary does the following descriptions refer?

  1. The flagship species of the sanctuary is the near threatened blackbuck antelope, the sole member of the antelope family in India
  2. It is home to both sand dunes and tropical dry evergreen forests
  3. This site has recorded the second largest congregation of migratory waterbirds in India

Find the correct answer from below options:

 a) Point Calimere Wildlife and Bird Sanctuary

 b) Kumbhalgarh Wildlife Sanctuary

 c) Sariska National Park

 d) Mount Abu Sanctuary

2.2k views

With reference to Phosphorous cycle, consider the following statements:

1.        It involves the movement of phosphorous between the soil, living beings and the atmosphere.

2.        It is the slowest of all the biogeochemical cycles.

3.        Increased use of agricluture fertilizers can alter the phosphorus cycle.

Which of the statements given above is/are correct?

(a)        2 only

(b)        2 and 3 only

(c)        1 and 3 only

(d)        1, 2 and 3

2k views

With reference to Phosphorous cycle, consider the following statements:

1.        It involves the movement of phosphorous between the soil, living beings and the atmosphere.

2.        It is the slowest of all the biogeochemical cycles.

3.        Increased use of agricluture fertilizers can alter the phosphorus cycle.

Which of the statements given above is/are correct?

(a)        2 only

(b)        2 and 3 only

(c)        1 and 3 only

(d)        1, 2 and 3

Ans. B

statement 1 incorrect: Phosphorus cycle is a sedimentary cycle, so role of atmosphere is not there.

Statement 2 correct: It takes millions of years for phosphorus deposited on the Ocean crust to surface due to tectonic or volcanic activity.

Statement 3 correct

That's correct. 

2k views

To which wildlife sanctuary does the following descriptions refer?

  1. The flagship species of the sanctuary is the near threatened blackbuck antelope, the sole member of the antelope family in India
  2. It is home to both sand dunes and tropical dry evergreen forests
  3. This site has recorded the second largest congregation of migratory waterbirds in India

Find the correct answer from below options:

 a) Point Calimere Wildlife and Bird Sanctuary

 b) Kumbhalgarh Wildlife Sanctuary

 c) Sariska National Park

 d) Mount Abu Sanctuary

A? 

all others are in Rajasthan, and tropical dry evergreen found in TN and AP.

C?

A - Point Calimere. 

2.1k views

When the Chief Justice of a High Court acts in an administrative capacity, he is subject to


a) the writ jurisdiction of any of the other judges of the High Court
b) special control exercised by the Chief Justice of India
c) discretionary powers of the Government of the State
d) special powers provided to the Chief Minister in this regard

Source : CSE 1996

C

As per Article 227 (3), the Chief Justice of High Court when acting in an administrative capacity, any rule made by him shall not be inconsistent with the provision of any law in force, and shall require the previous approval of the Governor.

2.1k views

The cost of government’s borrowing from the open market can be gauged from:

a)  the cut-off price for G-Secs

b)  the cut-off yield for G-Secs

c)  the crowding out effect

d)  the overall interest payments

2.1k views

In terms of public expenditure, crowding-out effect implies:

a)  Excess government subsidies distorting resource allocation 

b)  Excess private expenditure producing inflation 

c)  Market exploitation by the private sector due to reckless government expenditure.

d)  Excess government expenditure, raising interest rate and depressing the availability of funds. 

2.1k views

In terms of public expenditure, crowding-out effect implies:

a)  Excess government subsidies distorting resource allocation 

b)  Excess private expenditure producing inflation 

c)  Market exploitation by the private sector due to reckless government expenditure.

d)  Excess government expenditure, raising interest rate and depressing the availability of funds. 

D

Correct. 

2.1k views

The cost of government’s borrowing from the open market can be gauged from:

a)  the cut-off price for G-Secs

b)  the cut-off yield for G-Secs

c)  the crowding out effect

d)  the overall interest payments

B?

B is correct. 

The cut -off yield is the interest rate that should be given by the government to the bond holders as per the bond auction. It is the interest rate for the last bundle auction. Hence it shows the interest cost of the government.

2.1k views

When an Indian student is getting royalties from the industrial design that he provided to an overseas company; the receipt will be included under:

a)  None

b)  Forex

c)  Long term capital account 

d)  Invisible items 

2.1k views

When an Indian student is getting royalties from the industrial design that he provided to an overseas company; the receipt will be included under:

a)  None

b)  Forex

c)  Long term capital account 

d)  Invisible items 

D?

I thought they come under income part in invisible items of current account.

D is correct. Invisible Items. 

2.1k views

When an Indian student is getting royalties from the industrial design that he provided to an overseas company; the receipt will be included under:

a)  None

b)  Forex

c)  Long term capital account 

d)  Invisible items 

I'm a bit confused, Royality is a capital gain so it probablyC)  LTCA

Nope, they are covered under Invisible Items. 

2k views

"It is found in lentic ecosystem, flowers of this plant are capable of thermoregulation and leaves are hydrophobic in nature". The statement best describes which flower? 

a.   Water Hyacinth

b.        Orchids

c.        Sacred Lotus

d.        Sea Buckthorm

B??


A - Water Hyacinth

2k views
  1. The city of Jaunpur was founded in the memory of
    1. Ghiyasuddin Tughlaq
    2. Mohammad-bin-Tughlaq
    3. Firoz Tughlaq
    4. Akbar

B?

2k views

 Chaukhandi Stupa which was declared to be a monument of national importance by the Archaeological Survey of India in June 2019 is located at

A) Sanchi

B) Sarnath

C) Bharhut

D) Thotlakunda

2k views

Space exploration missions being conducted by NASA with the purpose of researching several of the Solar System bodies, including the dwarf planet Pluto are called as

A) New Horizons Mission

B) Exoplanet Exploration Program

C) New Frontiers program

D) VERITAS program

2k views

 Chaukhandi Stupa which was declared to be a monument of national importance by the Archaeological Survey of India in June 2019 is located at

A) Sanchi

B) Sarnath

C) Bharhut

D) Thotlakunda

B) Sarnath (UP )

Right. 

2.1k views

Space exploration missions being conducted by NASA with the purpose of researching several of the Solar System bodies, including the dwarf planet Pluto are called as

A) New Horizons Mission

B) Exoplanet Exploration Program

C) New Frontiers program

D) VERITAS program

Probably A) bcz it was first to explore pluto..

Incorrect. 

Answer isNew Frontiers Program- C 

New Frontiers program is a series of space exploration missions being conducted by NASA with the purpose of researching several of the Solar System bodies, including the dwarf planet Pluto.

.

2k views

Kurzarbeit scheme, recently seen in the news is related to which country. 

 a) Japan 

 b) USA 

 c) Germany 

 d) Saudi Arabia 

 

2k views

Windrush Scheme, recently seen in news is related to which of the following countries? 

 a) Australia 

 b) Japan 

 c) Germany 

 d) United Kingdom 

 

2k views

 WHO’s REPLACE campaign is related to which of the following ?


 a) Replace all petrol and diesel vehicles to Electric Vehicles by 2030. 


 b) Replace all cooling systems in government buildings with energy efficient cooling system. 


 c) Provide protein rich food to children in the government schools. 


 d) Eliminate industrially produced trans-fatty acids from the global food supply.

2k views

Consider the following about ‘Rushikulya’:

1. It flows in the state of Odisha.

2. It makes a very big delta at its mouth.

3. Large scale nesting of olive ridley turtles takes place near the mouth of the river. 

Select the correct answer using the codes given below.

a) 1 and 2

b) 2 and 3

c) 1 and 3

d) 1, 2 and 3


2k views

Q- Which of the following describing best the concept"Samsara" in Buddhism?

A) State of bliss and rest.

B) Beginningless cycle of repeated birth, mundane existence and dying again.

C) A mental state beyond all comprehension.

D) Life is transient and changing.

B

2.1k views

Q-1 Which of the following Article/s under Indian constitution related to "Right to food" ?

1) Article 21

2) Article 47

3) Article 21A

4) Article 39A

Select the correct answer:

A) 2 and 3 only

B) 1 and 2 only

C) 1, 2, and 3 only

D) 1, 2 and 4


B

2.1k views

Q-2  Bhamakalpam dance drama was innovated by Siddhendra Yogiand has a close relation with which of the following classical dance?

A) Bharatnatyam

B) Kathakali

C) Kuchipudi

D) Odissi

C?

2.1k views

Which among the following is correct about ‘Thirumalai Nayak’?

a) Tirumala Nayaka territories comprised much of the old Pandya territories which included Coimbatore, Tirunelveli and Madurai districts

b) Tirumala Nayaka was a great patron of art and architecture

c) Both (a) and (b)

d) Neither (a) nor (b)

2.1k views

Which among the following is correct about ‘Khamba Thoibi’? 

a) Only females participate in this dance

b) It is a popular folk dance of Meghalaya

c) Both (a) and (b)

d) Neither (a) nor (b)

2.1k views

Which among the following is correct about ‘Khamba Thoibi’? 

a) Only females participate in this dance

b) It is a popular folk dance of Meghalaya

c) Both (a) and (b)

d) Neither (a) nor (b)

D?

S1: khamba thoibi is a love of a princess and orphan

S2: popular folk dance of manipur

Absolutely correct. 

2.1k views

Which among the following is correct about ‘Gram Manchitra App’?

a) It is a Geo Spatial based decision support system for the panchayats

b) It aims to determine the various social forestry areas in the Panchayat

c) It aims to track the progress in infrastructure made due to 

d) None of the above

2.1k views

Which among the following is incorrect about ‘Ocean and Cryosphere in a Changing Climate’?

a) It is the first report that discusses the effects of climate change on oceans and cryosphere

b) It is published by the UN Environment Programme (UNEP)

c) It gives us a report on effect of climate change on water bodies only

d) None of the above

2k views

ARS21said

Which among the following is incorrect about ‘Ocean and Cryosphere in a Changing Climate’?

a) It is the first report that discusses the effects of climate change on oceans and cryosphere

b) It is published by the UN Environment Programme (UNEP)

c) It gives us a report on effect of climate change on water bodies only

d) None of the above

Shouldn't it be B)

Since IPCC came out with a report on cryosphere. 

IPCC Special Report on the Ocean and Cryosphere in a Changing Climate (SROCC) was approved at the IPCC's 51st Session (IPCC-51) in September 2019 in Monaco. 

2.2k views

Which among the following is correct about ‘Gram Manchitra App’?

a) It is a Geo Spatial based decision support system for the panchayats

b) It aims to determine the various social forestry areas in the Panchayat

c) It aims to track the progress in infrastructure made due to 

d) None of the above

A) Under Ministry of Panchayat


A?

Correct. 

2.2k views

Which school of painting in India has Roman Grecian influence? 

A. Company paintings 

B . Bazaar painting 

C. Bengal school of art 

D. kangra painting

2.2k views

Which school of painting in India has Roman Grecian influence? 

A. Company paintings 

B . Bazaar painting 

C. Bengal school of art 

D. kangra painting

B

2.2k views

Which among the following organization publishes the ‘Frontiers Report’?

a) United Nations Environment Program

b) Interpol

c) International Court of Justice

d) World Bank


2.2k views

'National Waterway – 1’, doesn’t pass through which among the following?

a) Kanpur

b) Patna

c) Prayagraj

d) Haldia

2.2k views

Gochisaid

'National Waterway – 1’, doesn’t pass through which among the following?

a) Kanpur

b) Patna

c) Prayagraj

d) Haldia

A?

Correct.

2.2k views

Consider the following about Secretary of the Panchayat

  1. Secretary of the panchayat is elected by the Gram Panchayat from amongst its members.
  2. Secretary of the panchayat is responsible for calling the meeting of the Gram Sabha and Gram Panchayat and keeping a record of the proceedings.

Which of the above statements is/are correct?

 a) 1 only

 b) 2 only

 c) Both 1 and 2

 d) None

2.2k views

Which of the following is/are the constitutional functions of the Indian Parliament?

  1. Settling disputes between various government departments.
  2. Supervising the work of State legislatures
  3. Appointing the heads of all constitutional bodies

Select the correct codes:

 a) 1, 2

 b) 2, 3

 c) 1, 3

 d) None

2.2k views

Which among the following is correct about ‘Shala Darpan portal’?

a) It is an initiative of Human Resources Development

b) It aims evaluation of teachers and students

c) Both (a) and (b)

d) Neither (a) nor (b)

2.2k views

Is there permission write questions from test series of other institutions here.

Yes, we can always discuss more questions here. However, I recommend not to to quote which institute/Website it is. 

All for sharing more and discussing. 

2.2k views

Gochisaid

Which among the following organization publishes the ‘Frontiers Report’?

a) United Nations Environment Program

b) Interpol

c) International Court of Justice

d) World Bank


A?

A?

Correct

2.2k views

Which among the following is correct about ‘Shala Darpan portal’?

a) It is an initiative of Human Resources Development

b) It aims evaluation of teachers and students

c) Both (a) and (b)

d) Neither (a) nor (b)

A

C?

not sure

C is correct. 


Shala Darpan provides an integrated platform to meet the educational and administrative needs of all stakeholders-students, teachers and parents. It is capable of sending and receiving effective dialogue between every stakeholder.

The major components include integrated content management portal which consists of 636 websites of Navodaya schools across the country. It will contain information relating to the service record, transfer/posting, disciplinary action, ACR tracking and the portal will improve mess management and hostel conditions.

Apart from this, it will also strengthen budget and finance management system.The best part is this portal has been designed from OPEN SOURCE TECHNOLOGY and hence the cost incurred was very low and up-dation cost in future will remain low.

The major effect will be end to end info sharing, proper analytical details, better transparency in evaluation of teachers and students both, 654 bilingual platform for each school and role-based access to have better evaluation at every stage.

Another golden leaf is that this platform at this minimum cost abides by the Guidelines for Indian govt. Websites(GIGS).

The Navodaya Vidyalayas are co-educational residential schools established by the Navodaya Vidyalaya Samiti which is an autonomous organization under MHRD to provide quality modern education.

2.1k views

Which of the following is/are the constitutional functions of the Indian Parliament?

  1. Settling disputes between various government departments.
  2. Supervising the work of State legislatures
  3. Appointing the heads of all constitutional bodies

Select the correct codes:

 a) 1, 2

 b) 2, 3

 c) 1, 3

 d) None

D?

D

Correct.

2.1k views

Consider the following about Secretary of the Panchayat

  1. Secretary of the panchayat is elected by the Gram Panchayat from amongst its members.
  2. Secretary of the panchayat is responsible for calling the meeting of the Gram Sabha and Gram Panchayat and keeping a record of the proceedings.

Which of the above statements is/are correct?

 a) 1 only

 b) 2 only

 c) Both 1 and 2

 d) None

B

Correct.

2.1k views

 Consider the following countries:

1. Australia

2. Canada

3. China

4. India

5. Japan

6. USA

Which of the above are ‘Free trade partners’ of 

the ASEAN?

(a) 1, 2, 4 and 5 only

(b) 3, 4, 5 and 6 only

(c) 1, 3, 4 and 5 only

(d) 2, 3, 4 and 6 only

2.1k views

Which of the following countries share their border with Dead Sea?

1. Israel

2. Lebanon

3. Jordan

Select the correct answer using the codes given 

below:

(a) 1 only

(b) 1 and 3 only

(c) 2 and 3 only

(d) 1, 2 and 3


2k views

Recently India opened South Asia's first cross border petroleum products pipeline to which country?

(a) Nepal

(b) Bangladesh

(c) Bhutan

(d) None of the above

2.1k views

Recently India opened South Asia's first cross border petroleum products pipeline to which country?

(a) Nepal

(b) Bangladesh

(c) Bhutan

(d) None of the above

A

Correct. 

2.1k views

 Consider the following countries:

1. Australia

2. Canada

3. China

4. India

5. Japan

6. USA

Which of the above are ‘Free trade partners’ of 

the ASEAN?

(a) 1, 2, 4 and 5 only

(b) 3, 4, 5 and 6 only

(c) 1, 3, 4 and 5 only

(d) 2, 3, 4 and 6 only

C

Correct. 

2k views

Which of the following countries share their border with Dead Sea?

1. Israel

2. Lebanon

3. Jordan

Select the correct answer using the codes given 

below:

(a) 1 only

(b) 1 and 3 only

(c) 2 and 3 only

(d) 1, 2 and 3


B

Guide To Jordan: Things First-Time Travelers Should Know

2k views

Which of the following causes Pneumonia?

1. Virus

2. Bacteria

3. Fungi

Select the correct answer from the code given 

below:

(a) 1 only

(b) 1 and 2 only

(c) 2 and 3 only

(d) 1,2 and 3

2k views

Which of the following statements are correct with respect to K-4 ballistic missile?

1. It is a submarine version of Agni – V

2. It has been indigenously developed by 

DRDO.

Select the correct answer using the code given 

below:

(a) 1 only

(b) 2 only

(c) Both 1 and 2

(d) Neither 1 nor 2

2k views

1. The Dark Web only hosts for undertaking illegal activities

2. The surface Web provides for 90% of the internet content 

3. Deep web includes intranet hosted by firms and government 

Which of the statement/s given above is/are 

correct?

(a) 1 only

(b) 1 and 2 only

(c) 2 and 3 only

(d) 3 only

2k views

Consider the following statements about Ultima Thule:

1. It is found in Kuiper belt.

2. Its was discovered by the NASA Voyager

Which of the statements given above is/are 

correct?

(a) 1 only

(b) 2 only

(c) Both 1 and 2

(d) Neither 1 nor 2

2k views

Which of the following statements are correct about PhotoDNA technology? 

1. It creates a unique digital signature of an image known as Hash. 

2. It is like a facial recognition software. 

3. A PhotoDNA Hash is not reversible and therefore cannot be used to recreate an image. 

Select the correct answer using the code given 

below: 

(a) 2 and 3 only 

(b) 1 and 3 only 

(c) 1, 2 and 3 only 

(d) None of the above

2k views

Which of the following causes Pneumonia?

1. Virus

2. Bacteria

3. Fungi

Select the correct answer from the code given 

below:

(a) 1 only

(b) 1 and 2 only

(c) 2 and 3 only

(d) 1,2 and 3

D

Correct. 

2k views

1. The Dark Web only hosts for undertaking illegal activities

2. The surface Web provides for 90% of the internet content 

3. Deep web includes intranet hosted by firms and government 

Which of the statement/s given above is/are 

correct?

(a) 1 only

(b) 1 and 2 only

(c) 2 and 3 only

(d) 3 only

D

Correct. 

2k views

Consider the following statements about Ultima Thule:

1. It is found in Kuiper belt.

2. Its was discovered by the NASA Voyager

Which of the statements given above is/are 

correct?

(a) 1 only

(b) 2 only

(c) Both 1 and 2

(d) Neither 1 nor 2

A

Correct. 

2k views

Which among the following categories of goods are covered under Index of Industrial Production (IIP)?

1. Primary Goods

2. Intermediate Goods

3. Basic Goods

4. Infrastructure Goods

Select the correct answer using the 

code given below:

(a) 1 and 2 only

(b) 1, 2 and 3 only

(c) 1, 2 and 4 only

(d) 1, 2, 3 and 4


2.1k views

Which among the following factors contributes the highest to the GDP in India?

(a) Investment Expenditure

(b) Consumption Expenditure

(c) Government Expenditure

(d) Net Exports

2.1k views

Which of the following statements are correct about PhotoDNA technology? 

1. It creates a unique digital signature of an image known as Hash. 

2. It is like a facial recognition software. 

3. A PhotoDNA Hash is not reversible and therefore cannot be used to recreate an image. 

Select the correct answer using the code given 

below: 

(a) 2 and 3 only 

(b) 1 and 3 only 

(c) 1, 2 and 3 only 

(d) None of the above

B, Microsoft in its website says PhotoDNA technology don't recognise faces in the photos.

Right. 

2.1k views

Which among the following factors contributes the highest to the GDP in India?

(a) Investment Expenditure

(b) Consumption Expenditure

(c) Government Expenditure

(d) Net Exports

B, Indian economy is highly consumption based.

Correct.

2.1k views

Which among the following categories of goods are covered under Index of Industrial Production (IIP)?

1. Primary Goods

2. Intermediate Goods

3. Basic Goods

4. Infrastructure Goods

Select the correct answer using the 

code given below:

(a) 1 and 2 only

(b) 1, 2 and 3 only

(c) 1, 2 and 4 only

(d) 1, 2, 3 and 4


C,   IIP classification : Primary, Capital, Infra, Durable, Non durable, intermidiate Goods

Correct.

2.1k views

Consider the following statements:

1. Under WTO, countries classify themselves as developing or developed.

2. Being classified as a developing country, automatically makes themeligible to get preference from the developed countries.

Which of the statements given above is/are correct?

(a) 1 only

(b) 2 only

(c) Both 1 and 2

(d) Neither 1 nor 2

2.1k views

Which among the following Indicators are being tracked by the RBI under the Prompt Corrective Action (PCA) Framework? 

1. Capital Adequacy ratio (CAR) 

2. Gross NPA 

3. Leverage ratio 

4. Return on Assets 

Select the correct answer using the code given below: 

(a) 1 and 2 only 

(b) 1, 3 and 4 only 

(c) 1, 2 and 4 only 

(d) 1, 2, 3 and 4


2.1k views

Which among the following Indicators are being tracked by the RBI under the Prompt Corrective Action (PCA) Framework? 

1. Capital Adequacy ratio (CAR) 

2. Gross NPA 

3. Leverage ratio 

4. Return on Assets 

Select the correct answer using the code given below: 

(a) 1 and 2 only 

(b) 1, 3 and 4 only 

(c) 1, 2 and 4 only 

(d) 1, 2, 3 and 4


B, Net NPAs

Correct. 

2.1k views

Consider the following statements:

1. Under WTO, countries classify themselves as developing or developed.

2. Being classified as a developing country, automatically makes themeligible to get preference from the developed countries.

Which of the statements given above is/are correct?

(a) 1 only

(b) 2 only

(c) Both 1 and 2

(d) Neither 1 nor 2

D

A is the right answer.

There are no WTO definitions of “developed” and “developing” countries. Members announce for themselves whether they are “developed” or “developing” countries. However, other members can challenge the decision of a member to make use of provisions available to developing countries.

@Eclectic 

@JustTimberlake 

@samudragupt 



2.1k views

Which among the following is not a ‘Tropical Grasslands’?

a) Llanos

b) Sallsbury

c) Catinga

d) Steppes

1.9k views

Consider the following about ‘International Finance Corporation (IFC)’:

1. The IFC is a member of the World Bank Group.

2. Its aim is to create opportunities for people to escape poverty and achieve better living 

standards by mobilizing financial resources for private enterprise.

Select the correct answer using the codes given below.

a) 1 only

b) 2 only

c) Both 1 and 2

d) Neither 1 nor 2

1.9k views

Consider the following about ‘Bhashan Char Island’:

1. Bangladesh has sovereign control over the island.

2. It is located on the mouth of river Meghna river.

Select the correct answer using the codes given below.

a) 1 only

b) 2 only

c) Both 1 and 2

d) Neither 1 nor 2

1.9k views

Consider the following about ‘Bhashan Char Island’:

1. Bangladesh has sovereign control over the island.

2. It is located on the mouth of river Meghna river.

Select the correct answer using the codes given below.

a) 1 only

b) 2 only

c) Both 1 and 2

d) Neither 1 nor 2

C should be the ans, Rohingyas will be relocated to this island.

Correct.


The Bhashan Char is an uninhabited island around 30 kilometre east of Hatiya island in the South East Bangladesh. Hatiya has a population of 600,000. Bhashan Char island was formed about two decades ago on the mouth of river Meghna.

Pics

Approximately 6000-7000 Rohingya refugees have agreed to be relocated to the newly built camp on the Bhashan Char Island.

1.9k views

Which among the following is not a ‘Tropical Grasslands’?

a) Llanos

b) Sallsbury

c) Catinga

d) Steppes

D...Steppes

Correct.

1.9k views

Consider the following about ‘International Finance Corporation (IFC)’:

1. The IFC is a member of the World Bank Group.

2. Its aim is to create opportunities for people to escape poverty and achieve better living 

standards by mobilizing financial resources for private enterprise.

Select the correct answer using the codes given below.

a) 1 only

b) 2 only

c) Both 1 and 2

d) Neither 1 nor 2

C ?

Correct.

1.9k views

Consider the following about ‘Tidal Locking’:

1. It is a phenomena where the rotational period of a celestial body matches its orbital period.

2. Earth’s natural satellite expresses this phenomena.

Select the correct answer using the codes given below.

a) 1 only

b) 2 only

c) Both 1 and 2

d) Neither 1 nor 2

2k views

‘National Mission on Cultural Mapping of India’ is a part of which among the following programmes?

a) PRASAD

b) HRIDAY

c) Ek Bharat Shreshtha Bharat

d) None of these

1.9k views

Consider the following about ‘Gravitational Waves’:

1. Gravitational waves propagate at a a speed much lesser than the speed of light.

2. Gravitational waves are predicated on the Newton's law of universal gravitation. 

3. The Laser Interferometer Gravitational-Wave Observatory (LIGO) experiment had made the first observation of gravitational waves.

Select the correct answer using the codes given below.

a) 1 and 3 only

b) 2 and 3 only

c) 3 only

d) 1, 2 and 3

1.9k views

Nikesaid

Consider the following about ‘Tidal Locking’:

1. It is a phenomena where the rotational period of a celestial body matches its orbital period.

2. Earth’s natural satellite expresses this phenomena.

Select the correct answer using the codes given below.

a) 1 only

b) 2 only

c) Both 1 and 2

d) Neither 1 nor 2

C

Correct. 

1.9k views

Which among the following is not a ‘Tropical Grasslands’?

a) Llanos

b) Sallsbury

c) Catinga

d) Steppes

D

Correct. 

1.9k views

ARS21said

Q: In context of the Indian economy, which of the following measures can be used to reduce inflation?

1. Increasing Cash Reserve Ratio (CRR).

2. Increasing Statutory Liquidity Ratio (SLR).

3. Reducing Reverse Repo Rate.


A. 1 and 2 only

B. 1, 2 and 3

C. 2 and 3 only

D. 1 only

a?

2k views

C?

2k views

ARS21said

Q: In context of the Indian economy, which of the following measures can be used to reduce inflation?

1. Increasing Cash Reserve Ratio (CRR).

2. Increasing Statutory Liquidity Ratio (SLR).

3. Reducing Reverse Repo Rate.


A. 1 and 2 only

B. 1, 2 and 3

C. 2 and 3 only

D. 1 only

a?

B

ans should be A

if we reduce reverse reporate,bank wont be parking excess money with rbi... 

@Neyawn help plz!!

Which was my thought process, and I marked A. 

2k views
The term Operation Clean Art, recently seen in news, is related to
A. Smuggling of Elephant tusks
B. Ancient Indian paintings
C. Smuggling of Mongoose hair
D. Indian Sculptures
2k views

The term ‘Torrefaction’ recently seen in news is related to which among the following?

a) New technology for soft landing on moon

b) Physical property of metals defining their ability to be hammered, pressed, or rolled into 
thin sheets without breaking

c) Process for making high energy bio-coal from agri-waste

d) A method to produce bio-diesel

2k views

As per the Central Water Commission’s (CWC) ‘Status of Trace and Toxic Metals in Indian Rivers, 2019’, consider the following statements.


1. Iron is most common contaminant found in Indian rivers.

2. Concentration of sodium chloride is very high in all rivers in India.

3. Two-thirds of the water quality stations spanning India’s major rivers are contaminated by one or more heavy metals.

Which of the above statements is/are correct?

a) 2 and 3 only

b) 2 only

c) 1 and 3 only

d) 1, 2 and 3

2k views

Which of the following are the reasons for the existence of deserts on the western margin of continents?

1. Westerlies

2. Easterlies

3. Cold Ocean currents

Select the correct answer using the code given below:

a 1 and 3 only
b 2 only
c 2 and 3 only
d 3 only

D?

2k views
The term Operation Clean Art, recently seen in news, is related to
A. Smuggling of Elephant tusks
B. Ancient Indian paintings
C. Smuggling of Mongoose hair
D. Indian Sculptures

C

Correct 

2k views

The term ‘Torrefaction’ recently seen in news is related to which among the following?

a) New technology for soft landing on moon

b) Physical property of metals defining their ability to be hammered, pressed, or rolled into 
thin sheets without breaking

c) Process for making high energy bio-coal from agri-waste

d) A method to produce bio-diesel

C

Correct 

2k views

As per the Central Water Commission’s (CWC) ‘Status of Trace and Toxic Metals in Indian Rivers, 2019’, consider the following statements.


1. Iron is most common contaminant found in Indian rivers.

2. Concentration of sodium chloride is very high in all rivers in India.

3. Two-thirds of the water quality stations spanning India’s major rivers are contaminated by one or more heavy metals.

Which of the above statements is/are correct?

a) 2 and 3 only

b) 2 only

c) 1 and 3 only

d) 1, 2 and 3

C ?

Yes.. correct. 

2k views
Consider the following statements about 
the Manibeli Declaration:

1. This Declaration calls for a moratorium on the World Bank funding of large dams. 
2. It highlighted the difficulties faced by the poor due to development projects.
3. It was named after a village of Punjab.
Which of the statements given above is/are correct?

(a) 1 and 2 only 
(b) 2 and 3 only
(c) 2 only
(d) 1, 2 and 3
2.1k views
Consider the following statements about the Pachmarhi Biosphere Reserve:

1. It is located in the Satpura mountain range.
2. Dhoopgarh peak is inside it.
3. Bhils are the principal tribal population of the region.

Which of the statements given above are correct?

(a) 1 and 2 only
(b) 2 and 3 only
(c) 1 and 3 only
(d) 1, 2 and 3
2k views
Which of the following Acts are administered by the Ministry of Environment, Forest and Climate 
Change?

1. The Public Liability Insurance Act, 
1991
2. The Water Cess Act, 1977
3. The River Boards Act, 1956

Select the correct answer using the code given below:

(a) 1 and 2 only
(b) 2 and 3 only
(c) 1 and 3 only
(d) 1, 2 and 3

2k views
 Gogabeel, a wetland , Important Bird area has been recently declared as a Community reserve in
A) Uttar Pradesh
B) Jharkhand
C) Bihar
D) Chattisgarh
2k views
The Term Bio rock technology which is recently in news is related to

A) Extraction of Minerals from ores in Environmental friendly manner.

B) Extraction of Minerals from ores by utilizing Microbes and plants.

C) An Artificial structure to facilitate revival of Coral reefs.

D) A Technologies to filter Arsenic impurities from aquifers.
2k views
‘Sisseri River Bridge’ recently seen in news is located in:

a) Uttarakhand

b) Jammu and Kashmir

c) Sikkim

d) Arunachal Pradesh
2.1k views
Consider the following about ‘Bali Yatra’:

1. The festival is celebrated every year from the day of Kartika Purnima.

2. This festival is held in Odisha.

Select the correct answer using the codes given below.

a) 1 only

b) 2 only

c) Both 1 and 2

d) Neither 1 nor 2
2.1k views
‘Sisseri River Bridge’ recently seen in news is located in:

a) Uttarakhand

b) Jammu and Kashmir

c) Sikkim

d) Arunachal Pradesh

D.

Correct. 

2.4k views
Artificial leaf or Quantum leaf a new technology in news developed by researchers of Indian Institutes of Science Education and Research is a

A) Technology to control weed proliferation in Agricultural lands.

B) Solar Photovoltaic module to generate electrical energy.

C) Technology to generate Syngas.

D) An innovation in Solar cells to align with direction of Sun's rotation.
2.1k views
» show previous quotes

C? 

Correct.

The Gogabil Lake in Katihar district, one of the largest wetlands of Bihar. Gogabeel, an ox-bow lake in Bihar’s Katihar district, has been declared as the state’s first ‘Community Reserve’.  Nearly 300 migratory birds come to the lake and its vicinity during monsoon and winter from the Caspian Sea and Siberian region. Among the different species of migratory birds, there are around 90 species of threatened birds, including stork, black-necked stork and white Ibis. Spread over 73.88 acres of land, this ox-bow shaped lake is formed by the Mahananda and Kankahar in the north and Ganga in the south.

4.6k views
Artificial leaf or Quantum leaf a new technology in news developed by researchers of Indian Institutes of Science Education and Research is a

A) Technology to control weed proliferation in Agricultural lands.

B) Solar Photovoltaic module to generate electrical energy.

C) Technology to generate Syngas.

D) An innovation in Solar cells to align with direction of Sun's rotation.

C? 

Correct Answer

2.1k views
Mission Raksha Gyan Shakti is related to

a) Enhance the private sector participation in defence manufacturing.

b) Boost Export of India’s Defence production

c) Boost Intellectual Property Right (IPR) culture in indigenous defence industry

d) None
2.1k views

Consider the following statements regarding President’s rule.

1. The imposition of the President’s rule requires the sanction of both the houses of Parliament.

2. President’s rule cannot be revoked without Cabinet approval even if the Prime Minister deems it necessary.

Which of the statements given above is/are correct?

a) 1 only
b) 2 only
c) Both 1 and 2
d) Neither 1 nor 2

2.1k views

Consider the following statements regarding President’s rule.

1. The imposition of the President’s rule requires the sanction of both the houses of Parliament.

2. President’s rule cannot be revoked without Cabinet approval even if the Prime Minister deems it necessary.

Which of the statements given above is/are correct?

a) 1 only
b) 2 only
c) Both 1 and 2
d) Neither 1 nor 2

A

That's correct. 

2.1k views

Consider the following statements regarding President’s rule.

1. The imposition of the President’s rule requires the sanction of both the houses of Parliament.

2. President’s rule cannot be revoked without Cabinet approval even if the Prime Minister deems it necessary.

Which of the statements given above is/are correct?

a) 1 only
b) 2 only
c) Both 1 and 2
d) Neither 1 nor 2

D?

No, A is the correct answer. 


The imposition of the President’s rule requires the sanction of both the houses of Parliament.

If approved, it can go on for a period of six months. However, the imposition cannot be extended for more than three years, and needs to be brought before the two houses every six months for approval.

To revoke President’s Rule, the government can use a special Section in the Union government’s Transaction of Business Rules (Rule 12), which allows for revocation of President’s Rule without Cabinet approval if the Prime Minister “deems it necessary”.

-Rule 12 of the Government of India (Transaction of Business) Rules, 1961, allows the Prime Minister to depart from laid down norms at his discretion. It says, “The Prime Minister may, in case or classes of cases permit or condone a departure from these rules, to the extent he deems necessary.”

-The Cabinet can subsequently give post-facto approval for any decision taken under Rule 12.

2.1k views
Mission Raksha Gyan Shakti is related to

a) Enhance the private sector participation in defence manufacturing.

b) Boost Export of India’s Defence production

c) Boost Intellectual Property Right (IPR) culture in indigenous defence industry

d) None

C

Correct

2.1k views
Consider the following about ‘Ashram Schools’:
1. Ashram schools are residential schools which impart education up to the 
secondary level.
2. The scheme provides for 100% central financing. 
Select the correct answer using the codes given below.
a) 1 only
b) 2 only
c) Both 1 and 2
d) Neither 1 nor 2
2.3k views
Which among the following is correct about ‘Kalapani River’?

a) It is one of the headwaters of the Kali River

b) Lipulekh Pass is in the vicinity of this river

c) Both (a) and (b)

d) Neither (a) nor (b)
Today
2.3k views
With reference to ‘Atal Beemit Vyakti Kalyan Yojana’, which among the following statements is/are correct? 

1. The Insured Person should have contributed not less than 78 days during each of the 
preceding four contribution periods.

2. The claim for relief under the Atal Beemit Kalyaan Yojana will be payable after the three 
months of his/her clear unemployment.

Select the correct answer using the codes given below.

a) 1 only

b) 2 only

c) Both 1 and 2

d) Neither 1 nor 2
2.3k views
Consider the following about ‘Ashram Schools’:
1. Ashram schools are residential schools which impart education up to the 
secondary level.
2. The scheme provides for 100% central financing. 
Select the correct answer using the codes given below.
a) 1 only
b) 2 only
c) Both 1 and 2
d) Neither 1 nor 2

C

Correct. 

2.3k views
Which among the following is correct about ‘Kalapani River’?

a) It is one of the headwaters of the Kali River

b) Lipulekh Pass is in the vicinity of this river

c) Both (a) and (b)

d) Neither (a) nor (b)
Today

C is the right answer. 

2.7k views
Consider the following about ‘Global Snow Leopard and Ecosystem Protection Program (GSLEP)’:

1. Its goal is to secure the long-term survival of the snow leopard in its natural ecosystem.

2. The GSLEP is a joint initiative of range country governments. 

3. India is also a part of GSLEP.

Select the correct answer using the codes given below.

a) 1 and 3 only
b) 2 and 3 only
c) 1 only
d) 1, 2 and 3
2.3k views
Which among the following is correct about ‘DARPAN scheme’? 

a) It has been launched by the Ministry of Finance

b) It aims to realize financial inclusion and IT modernization of Post offices

c) It is a joint effort of Ministry of Finance and Ministry of Communications and Information 
Technology

d) None of the above
2.3k views
Consider the following about ‘Global Snow Leopard and Ecosystem Protection Program (GSLEP)’:

1. Its goal is to secure the long-term survival of the snow leopard in its natural ecosystem.

2. The GSLEP is a joint initiative of range country governments. 

3. India is also a part of GSLEP.

Select the correct answer using the codes given below.

a) 1 and 3 only
b) 2 and 3 only
c) 1 only
d) 1, 2 and 3

D??

Correct. 


The GSLEP is a joint initiative of range country governments, international agencies, civil society, and the private sector. Its goal is to secure the long-term survival of the snow leopard in its natural ecosystem.

The goal of the GSLEP is for the 12 snow leopard range countries, with support from conservation agencies, NGOs and others to work together to identify and secure at least 20 healthy populations of snow leopards across the cat's range by 2020, or "20 by 2020". Many of these populations will cross international boundaries.

The three criteria that will secure healthy populations of snow leopards are populations that represent at least 100 breeding age snow leopards, contain adequate and secure prey populations and have connectivity to other snow leopard populations.

This is an interim goal for the years through to 2020. During the coming years, agreement will be reached on the steps needed to achieve the ultimate goal of ensuring that healthy snow leopard populations remain the icon of the mountains of Asia for generations to come.

2.2k views
Which among the following is correct about ‘DARPAN scheme’? 

a) It has been launched by the Ministry of Finance

b) It aims to realize financial inclusion and IT modernization of Post offices

c) It is a joint effort of Ministry of Finance and Ministry of Communications and Information 
Technology

d) None of the above

B??

Correct. 


Ministry of Communications and Information Technology launched ―DARPAN - Digital Advancement of Rural Post Office for A New India. The project aimed at realizing financial inclusion of un-banked rural population. It is IT modernization project which intends to provide a low power technology solution to each branch postmaster to improve service delivery. DARPAN offers core banking services such as cash deposit and withdrawal in savings bank and recurring deposit, mini statement, Aadhaar seeding and daily transaction report.

2.2k views
Consider the following statements:

1) The inflation rate of Indian economy during 2014-2019 was 4.5%.

2) During 2006-16, India was able to raise 50 million people out of poverty.

3) Currently, the Central Government debt is more than 50% of GDP.

Which of the above statement(s) is/are incorrect?

a) 1 only

b) 2 only

c) 2 and 3 only

d) All of the above

2.2k views
Which of the following are prominent themes of Union Budget 2020-21?

1) Aspirational India

2) Economic Development for all

3) Geriatric care

Select the correct answer using the codes given below.

a) 1 and 2 only

b) 2 and 3 only

c) 1 and 3 only

d) All of the above
2.4k views
Which of the following are components of Aspirational India according to the Budget,  2020-21?

1) Agriculture, Irrigation, and Rural Development

2) Empowerment of women and other vulnerable sections of society.

3) Wellness, Water, and Sanitation

4) Education and Skills

Select the correct answer using the codes given below.

a) 1 and 2 only

b) 3 and 4 only

c) 1, 3 and 4 only

d) All of the above
2.4k views
Consider the following statements about the Krishi Udaan scheme:

1) This scheme will be launched by the Ministry of commerce to help farmers export their produce internationally and nationally.

2) Under this scheme, special focus will be given to the North-East and tribal districts.

Which of the above statement(s) is/are correct?

a) 1 only

b) 2 only

c) Both 1 and 2 

d) Neither 1 nor 2
2.4k views
Consider the following statements:

1) The Bru tribe inhabit an area spread across parts of Mizoram, Tripura and the Chittagong Hill 
Tracts in Bangladesh.

2) They speak a dialect which is of Tibeto-Burmese origin.

Which of the above statement(s) is/are correct?

a) 1 only

b) 2 only

c) Both 1 and 2

d) Neither 1 nor 2
2.3k views
Which of the following are prominent themes of Union Budget 2020-21?

1) Aspirational India

2) Economic Development for all

3) Geriatric care

Select the correct answer using the codes given below.

a) 1 and 2 only

b) 2 and 3 only

c) 1 and 3 only

d) All of the above


Geriatric care is not among the three themes. 3rd theme is ‘Caring Society’. 

Three prominent themes of the Budget:

1) Aspirational India - better standards of living with access to health, education and better 

jobs for all sections of the society

2) Economic Development for all - “Sabka Saath , Sabka Vikas , Sabka Vishwas”.

3) Caring Society - both humane and compassionate; Antyodaya as an article of faith.

2.3k views
Which of the following isnotpart of the Quadrilateral Security Dialogue (Quad):

a) USA

b) Japan

c) Australia

d) South Korea
2.3k views
Kurukh of West Bengal, in news, is a type of 

A. Jute infecting Disease 

B. Block Painting 

C. Language 

D. Musical Instrument
2.3k views
Which of the following indicators are considered for the calculating the Global Hunger Index (GHI)

1) Undernourishment of the population

2) Mortality rate of children under the age of five

3) Neonatal Mortality

4) Maternal mortality

Select the correct answer using the codes given below.

a) 1 only

b) 1 and 2 only

c) 2, 3 and 4 only

d) All of the above
2.2k views
Recently, which of the following country has been declared ‘reciprocating territory’ by India?

a) UAE

b) Oman

c) Saudi Arabia

d) Kuwait.


2.2k views
@upsc2020 D southkorea 


Correct. 

2.2k views
Kurukh of West Bengal, in news, is a type of 

A. Jute infecting Disease 

B. Block Painting 

C. Language 

D. Musical Instrument

C

2.2k views

Correct. 

Request you to please reply to the question by quoting it. 

2.2k views
Consider the following statements about the FATF Grey List and Black List:
1) The countries included in the Grey List, are not safe and are supporting money laundering and 
terror funding.
2) The countries included in the Black List, are countries non-cooperative tax havens for terror 
funding.
Which of the statement(s) is/are incorrect?
a) 1 only
b) 2 only
c) Both 1 and 2
d) Neither 1 nor 2
2.7k views
» show previous quotes

C

Incorrect. Answer would be D

Question asked incorrect statement. 


The FATF is a Paris based organization that combats terrorism funding and money laundering. 

Grey List - The countries that are not safe and are supporting money laundering and terror funding are included in the list. 

Black List - The countries that is uncooperative tax havens for terror funding. They are also called Non-Cooperative Countries or Territories (NCCTs).  

The lists now have been renamed to “Call for Action” and “Monitored Jurisdiction”. The Call for Action countries or the Black List Countries include Iran and North Korea. The Monitored  Jurisdiction or Grey List include Iceland, Ghana, Botswana, Syria, Yemen, Pakistan, Mongolia and Cambodia. 

2.5k views
The ‘Idlib province’ was recentlyinnews. In which of the following country ‘Idlib province’ is located?
a) Iraq
b) Iran
c) Syria
d) Turkey
2.4k views
Consider the following statements:

1) The ‘Operation Vanilla’ has been launched to provide assistance to the affected population of Madagascar post devastation caused by Cyclone Diane.
2) The ‘Operation Vanilla’ has been jointly launched by the Indian and South African Navy.

Which of the above statement(s) is/are correct?
a) 1 only
b) 2 only
c) Both 1 and 2
d) Neither 1 nor 2
2.4k views
Consider the following statements with respect to Central Board of Directors of RBI. 

1. Deputy Governors are fulltime official directors of the Central Board.

2. The members of the Central Board are appointed by the Governor, RBI.

3. The deputy governors are not eligible for re-appointment to the Board. 

Which of the above given statement/s is/are correct?

a. 1 only 
b. 2 only 
c. 1 and 3 only 
d. None of the above
2.3k views
Which of the following statements is/are correct about the Appellate Body of WTO? 

1. It is composed of 7 members appointed for a term of 4 years by the dispute settlement body. 

2. The dispute settlement body comprises of all the members of WTO.

Select the correct answer using the code given below: 

a) 1 only 
b) 2 only 
c) Both 1 and 2 
d) Neither 1 nor 2
2.3k views
Which among the following organisations has constituted "PRODUCE Fund" for the benefit of Farmer Producer Organisations (FPOs)? 

a) SFAC 
b) NABARD 
c) NITI Aayog 
d) National Cooperative Development 
Corporation (NCDC)
2.3k views
The Promoters of a particular company sell their shares directly to the Public in order to dilute their stakes. Which among the following best describes this mechanism of Issuance of shares?

a) Initial Public Offer (IPO)
b) Follow-on-Public Offer (FPO)
c) Offer for Sale (OFS)
d) Rights Issue
2.3k views
 The ‘CT-TTX’ (Counter Terrorism Table Top Exercise) was in news recently. It is associated with which of the following. 

a) SCO 
b) QUAD 
c) BRICS 
d) ASEAN
 

2.2k views
Consider the following statements with reference to ‘South Asia Co-operative Environment Programme' (SACEP). 


1) SACEP supports national government’s efforts for environmental protection and sustainable development. 
2) Afghanistan, India, Nepal, Sri Lanka and Pakistan are the member countries of SACEP. 
Which of the above statements given above is/are correct? 

a) 1 only 
b) 2 only 
c) Both 1 and 2  
d) Neither 1 nor 2 
 

2.3k views
Consider the following statements with reference to ‘Doklam’. 

1. It is strategically located at the tri-junction of Pakistan, Afghanistan and China 
2. Hunza Valley is located in this region bordering with the Wakhan Corridor 

Select the correct answer using the code given below: 
a) 1 only 
b) 2 only 
c) Both 1 and 2 
d) Neither 1 not 2 
 

2.3k views
Consider the following statements with reference to ‘Doklam’. 

1. It is strategically located at the tri-junction of Pakistan, Afghanistan and China 
2. Hunza Valley is located in this region bordering with the Wakhan Corridor 

Select the correct answer using the code given below: 
a) 1 only 
b) 2 only 
c) Both 1 and 2 
d) Neither 1 not 2 
 

d?

Correct. 

 Doklam (in Standard Bhutanese), Zhoglam (in Standard Tibetan), or Donglang (in Chinese), is an area with a plateau and a valley, lying between Tibet's Chumbi Valley to the north, Bhutan's Ha Valley to the east and India's Sikkim state to the west.  It has been depicted as part of Bhutan in the Bhutanese maps since 1961, but it is also claimed by China.  

To date, the dispute has not been resolved despite several rounds of border negotiations between Bhutan and China. The area is of strategic importance to all three countries.  


2.3k views
Which among the following organisations has constituted "PRODUCE Fund" for the benefit of Farmer Producer Organisations (FPOs)? 

a) SFAC 
b) NABARD 
c) NITI Aayog 
d) National Cooperative Development 
Corporation (NCDC)

B?

Correct. 

2.2k views
 The ‘CT-TTX’ (Counter Terrorism Table Top Exercise) was in news recently. It is associated with which of the following. 

a) SCO 
b) QUAD 
c) BRICS 
d) ASEAN
 

B?

Correct. 

The National Investigation Agency hosted the first counter-terrorism exercise for the "Quad" countries -- India, the US, Japan and Australia -- at the NIA headquarters in Delhi in November, 19. 

The 'CT-TTX' (counter-terrorism table-top exercise) is first such engagement among the Quad countries on regional and global issues of common interest and also in the domain of counter-terrorism and cooperation.



2.2k views
Which among the following is likely to occur in Northern hemisphere at the time of Summer Solstice?

1. Longer days and shorter nights
2. Vertical rays of the sun on the Tropic of Cancer at noon
3. Shorter days and longer nights
4. Receiving more heat than it loses heat.

Select the correct answer from the codes given below:
a) 1 and 4 only
b) 1, 2 and 4 only
c) 2 and 3 only
d) 2, 3 and 4 only
2.2k views
Consider the following oceanic features:

1. Abyssal Plain
2. Continental Slope
3. Continental Shelf
4. Continental Rise

Which of the following is the correct sequence of given above features as we move from Coast to the Ocean?

a) 1 - 3 - 2 - 4
b) 3 - 2 - 4 - 1
c) 1 - 2 - 3 - 4
d) 3 - 2 - 1 - 4
2.2k views
With reference to the 'Red Soils in India', consider the following statements:

1. Red soil develops on crystalline igneous rocks in areas of low rainfall in the eastern and southern part of Deccan plateau.

2. These soils develop a reddish colour due to diffusion of aluminium in crystalline and metamorphic rocks.

Which of the given above statement(s) is/are incorrect?

a) 1 only
b) 2 only
c) Both 1 and 2
d) Neither 1 nor 2
2.2k views
Which of the following best describes the term 'Khadin'?

a) It is a type of medicinal grass found in western Rajasthan.

b) It is a traditional rainwater harvesting system for agriculture used in Rajasthan.

c) It is a type of primitive subsistence farming system in Odisha.

d) It is the name given to salt patches in Kutch region of Gujarat.
2.2k views

@Anjali8 @AlexanderSupertramp 

More questions above for practice!

2.2k views
Which of the following are the features of a mountain crossed by youthful rivers?

1. Gorges
2. V-shaped valleys
3. Rapids
4. Waterfalls

Select the correct answer from the codes given below:

a) 1 and 2 only
b) 1, 2 and 3 only
c) 2, 3 and 4 only
d) 1, 2, 3 and 4
2.2k views
Which of the following best describes the term 'Khadin'?

a) It is a type of medicinal grass found in western Rajasthan.

b) It is a traditional rainwater harvesting system for agriculture used in Rajasthan.

c) It is a type of primitive subsistence farming system in Odisha.

d) It is the name given to salt patches in Kutch region of Gujarat.

b?

Correct. 

2.2k views
Consider the following statements:
1. State Legislature can prescribe residence as a condition for certain employment in a State.
2. The State can provide for reservation of appointments in favour of any backward class that is not adequately represented in the state services.
Which of the statements given above is/are correct?
a) 1 only
b) 2 only
c) Both 1 and 2
d) Neither 1 nor 2

2.4k views
Which of the following provisions of the Constitution seek to achieve political equality in India?
1. No person is to be declared ineligible for inclusion in electoral rolls on grounds of religion, race, caste or sex.
2. Equality of opportunity in matters of public employment.
3. Elections to the Lok Sabha and the state assemblies to be on the basis of adult suffrage.
4. Abolition of untouchability.
Select the correct answer using the code given below.
a) 1 and 3 only
b) 1, 3 and 4 only
c) 2 and 4 only
d) 1, 2, 3 and 4
2.3k views
Consider the following statements:
1. State Legislature can prescribe residence as a condition for certain employment in a State.
2. The State can provide for reservation of appointments in favour of any backward class that is not adequately represented in the state services.
Which of the statements given above is/are correct?
a) 1 only
b) 2 only
c) Both 1 and 2
d) Neither 1 nor 2

C

B is correct. 

Article 35(a)(i)):

Parliament shall have and the legislature of a state shall nothave power to make laws - 


a) Prescribing residence as a condition for certain employments or appointments in a state or union territory or local authority or other authority (Article 16).

b) Empowering courts other than the Supreme Court and the high courts to issue directions, orders and writs of all kinds for the enforcement of fundamental rights (Article 32).

c) Restricting or abrogating the application of Fundamental Rights to members of armed forces, police forces, etc. (Article 33).

d) Indemnifying any government servant or any other person for any act done during the operation of martial law in any area (Article 34).

Article 16 provides for equality of opportunity for all citizens in matters of employment or appointment to any office under the State. No citizen can be discriminated against or be ineligible for any employment or office under the State on grounds of only religion, race, caste, sex, descent, place of birth or residence. 

Three exceptions to general rule of equality of opportunity in public employment:

1. Parliament can prescribe residence as a condition for certain employment or appointment in a state or union territory or local authority or other authority. As the Public Employment (Requirement as to Residence) Act of 1957 expired in 1974, there is no such provision for any state except Andhra Pradesh and Telangana.

2. The State can provide for reservation of appointments or posts in favour of any backward class that is not adequately represented in the state services.

3. A law can provide that the incumbent of an office related to religious or denominational institution or a member of its governing body should belong to a particular religion or denomination.


2.3k views
Which of the following provisions of the Constitution seek to achieve political equality in India?
1. No person is to be declared ineligible for inclusion in electoral rolls on grounds of religion, race, caste or sex.
2. Equality of opportunity in matters of public employment.
3. Elections to the Lok Sabha and the state assemblies to be on the basis of adult suffrage.
4. Abolition of untouchability.
Select the correct answer using the code given below.
a) 1 and 3 only
b) 1, 3 and 4 only
c) 2 and 4 only
d) 1, 2, 3 and 4

A

Correct. -A

There are two provisions in the Constitution that seek to achieve political equality. One, no person is to be declared ineligible for inclusion in electoral rolls on grounds of religion, race, caste or sex (Article 325). Two, elections to the Lok Sabha and the state assemblies to be on the basis of adult suffrage (Article 326). 

2.3k views
Which of the following provisions of the Constitution seek to achieve political equality in India?
1. No person is to be declared ineligible for inclusion in electoral rolls on grounds of religion, race, caste or sex.
2. Equality of opportunity in matters of public employment.
3. Elections to the Lok Sabha and the state assemblies to be on the basis of adult suffrage.
4. Abolition of untouchability.
Select the correct answer using the code given below.
a) 1 and 3 only
b) 1, 3 and 4 only
c) 2 and 4 only
d) 1, 2, 3 and 4

a

Yes, A is correct. 

2.3k views
Consider the following statements regarding Proportional Representation (PR) System?
1. Every party gets seats in the legislature in proportion to the percentage of votes that it gets in an election.
2. Under this system, a candidate who wins the election may not get the majority of votes.
3. The Constitution of India prescribes this system for the elections to the Rajya Sabha and Vidhan Parishads.
Which of the statements given above is/are correct?
a) 1 and 2 only
b) 2 and 3 only
c) 1 and 3 only
d) 1, 2 and 3

2.3k views
In context of the Indian Constitution, the violation of which of the following rights will not have a Constitutional Remedy?
a) Right to agitate for the protection of language.
b) Right of a ten year old child to get free elementary education by the State.
c) Right to participate in the management of the Industries.
d) Right against custodial harassment
2.3k views
In context of the Indian Constitution, the violation of which of the following rights will not have a Constitutional Remedy?
a) Right to agitate for the protection of language.
b) Right of a ten year old child to get free elementary education by the State.
c) Right to participate in the management of the Industries.
d) Right against custodial harassment

C. Part of DPSP. Rest 3 are Fundamental Rights. 

Correct. Article 43 A of Constitution talks of Participation of workers in the management of Industries which is a directive principle , hence no constitutional remedy.

2.3k views
Consider the following statements regarding Proportional Representation (PR) System?
1. Every party gets seats in the legislature in proportion to the percentage of votes that it gets in an election.
2. Under this system, a candidate who wins the election may not get the majority of votes.
3. The Constitution of India prescribes this system for the elections to the Rajya Sabha and Vidhan Parishads.
Which of the statements given above is/are correct?
a) 1 and 2 only
b) 2 and 3 only
c) 1 and 3 only
d) 1, 2 and 3

C

Correct. 

2.2k views
Consider the following statements:

1. The term ‘Bharat’ is not mentioned anywhere in the Constitution.
2. ‘Territory of India’ represents more territory than ‘Union of India’.

Which of the statements given above is/are correct?
a) 1 only
b) 2 only
c) Both 1 and 2
d) Neither 1 nor 2
2.2k views
Consider the following statements regarding Public Service Commissions.

1. Members of state service commission are appointed by the Governor but cannot be removed by Governor.
2. President can establish a Joint Public Service commission if requested by two or more states.
3. UPSC can serve the needs of state but only with the approval of Governor.

Which of the statements given above is/are correct?
a) 1 and 2 only
b) 3 only
c) 1 and 3 only
d) 1 only
2.2k views
With reference to “Gram Nyayalayas”, consider the following statements?

1. It has power to hear both criminal and civil cases.
2. Nyayalayas shall be guided by the principles of natural justice.
3. Both Local social activists and lawyers can act as mediators or conciliators.

Which of the statements given above is/are correct?
a) 1 and 2 only
b) 2 and 3 only
c) 1 and 3 only
d) 1, 2 and 3
2.2k views
Consider the following statements:

1. The term ‘Bharat’ is not mentioned anywhere in the Constitution.
2. ‘Territory of India’ represents more territory than ‘Union of India’.

Which of the statements given above is/are correct?
a) 1 only
b) 2 only
c) Both 1 and 2
d) Neither 1 nor 2

B. Statement 1 is incorrect, as Bharat is mentioned in Article 1. Statement 2 is correct as territory of India includes UT and other areas that may be acquired. 

Correct.

2.3k views
» show previous quotes

A. 

Incorrect. 

Correct Answer is D.

Parliamentcan establish a Joint State Public Service Commission for two or more states on the request of the state legislatures concerned. The chairman and members of the JSPSC are appointed by the President.

UPSC can serve the needs of a state on the request of the state governor and with theapproval of the President.

2.3k views
Mr. X was born in France. His grandfather, who was born in India migrated to France in 1930s. He wants to acquire Indian citizenship. He has approached Indian High Commission in France for this purpose.

Consider the following statements regarding his eligibility for Indian citizenship:

1. He is not eligible because his grandfather migrated from India before the commencement of the Constitution.
2. He is eligible because his grandfather was born in India.

Which of the statements given above is/are correct?
a) 1 only
b) 2 only
c) Both 1 and 2
d) Neither 1 nor
2.3k views
A bill will be deemed to be a money bill if it contains which of the following matters

1. The regulation of the borrowing of money by the Union government.
2. The withdrawal of money from Contingency Fund of India.
3. The imposition of fines.
4. The imposition of any tax by any local authority.
5. The appropriation of money out of the Consolidated Fund of India.

Select the correct answer using the code given below.
a) 1 and 5 only
b) 2, 3 and 5 only
c) 1, 2 and 5 only
d) 1, 2, 4 and 5
2.3k views

Hello people, 

Sorry for asking this question here. Which sites do you follow to solve Daily CA quiz other than forum and insight. 

TIA. 

Better to focus on a few and re - revise. :)

2.3k views
Mr. X was born in France. His grandfather, who was born in India migrated to France in 1930s. He wants to acquire Indian citizenship. He has approached Indian High Commission in France for this purpose.

Consider the following statements regarding his eligibility for Indian citizenship:

1. He is not eligible because his grandfather migrated from India before the commencement of the Constitution.
2. He is eligible because his grandfather was born in India.

Which of the statements given above is/are correct?
a) 1 only
b) 2 only
c) Both 1 and 2
d) Neither 1 nor

@kanoonwala 

2.3k views
Which of the following posts come under the jurisdiction of Lokpal?

1. The Prime Minister
2. Group D Officers
3. Member of Parliament
4. Group A Officers

Which of the options given above is/are correct?
a) 1 and 4 only
b) 3 and 4 only
c) 1, 3 and 4
d) 1, 2, 3 and 4
2.3k views
Consider the following statements about the
'Hinayana School' of Buddhist Philosophy:

1. It is the heterodox school of Buddhist
philosophy.
2. Its scriptures are written in Pali.
3. They believed that Buddha is an incarnation
of God and started his idol worship.

Which of the statement(s) given above is/are correct?
a) 1 only
b) 2 only
c) 2 and 3 only
d) 1, 2 and 3
2.3k views
Bishkek Declaration - It is
related to which of the following organisation?

a) Eurasian Economic Union
b) Organisation of Islamic Cooperation (OIC)
c) Central Asian Cooperation Organization
(CAC)
d) Shanghai Cooperation Organisation (SCO)
2.3k views
 With reference to 'Swadeshi movement', consider the following statements:

1. The movement led to setting up of National
Council of Education.

2. Tilak's Ganpati and Shivaji festival became
the medium of swadeshi propaganda.

3. Immediate annulment of Bengal partition was
a result of the swadeshi movement.

Which of the statement(s) given above is/are correct?

a) 1 only
b) 1 and 2 only
c) 2 and 3 only
d) 1, 2 and 3
2.3k views
Consider the following statements about the
'Hinayana School' of Buddhist Philosophy:

1. It is the heterodox school of Buddhist
philosophy.
2. Its scriptures are written in Pali.
3. They believed that Buddha is an incarnation
of God and started his idol worship.

Which of the statement(s) given above is/are correct?
a) 1 only
b) 2 only
c) 2 and 3 only
d) 1, 2 and 3

A

2.3k views
Bishkek Declaration - It is
related to which of the following organisation?

a) Eurasian Economic Union
b) Organisation of Islamic Cooperation (OIC)
c) Central Asian Cooperation Organization
(CAC)
d) Shanghai Cooperation Organisation (SCO)

D

2.2k views
With reference to Biofertilisers, consider the
following statements:

1. These are biologically active substances that
are applied in soil to enrich the soil with nitrogen, phosphorous, organic matter.

2. Nostoc and Anabaena are free living photosynthetic organisms capable of fixing atmospheric nitrogen.

3. Mycorrhizal fungi acts as bio-fertliser and are
known to occur naturally on the roots of forest
trees and crop plants.

Which of the statement(s) given above is/are correct?
a) 1 only
b) 1 and 2 only
c) 2 and 3 only
d) 1, 2 and 3
2.5k views
Consider the following oceanic features:

1. Abyssal Plain
2. Continental Slope
3. Continental Shelf
4. Continental Rise

Which of the following is the correct sequence of given above features as we move from Coast to the Ocean?

a) 1 - 3 - 2 - 4
b) 3 - 2 - 4 - 1
c) 1 - 2 - 3 - 4
d) 3 - 2 - 1 - 4

d

c is

correct

B should be the answer

@upsc2020 confirm please.

B is correct.

Divisions of the Ocean Floors 

The ocean floors can be divided into four major divisions: (i) the Continental Shelf; (ii) the Continental Slope; (iii) the Deep Sea Plain; (iv) the Oceanic Deeps. 

Besides, these divisions there are also major and minor relief features in the ocean floors like ridges, hills, sea mounts, guyots, trenches, canyons, etc. 

Continental Shelf

The continental shelf is the extended margin of each continent occupied by relatively shallow seas and gulfs. It is the shallowest part of the ocean showing an average gradient of 1° or even less. The shelf typically ends at a very steep slope, called the shelf break. The width of the continental shelves varies from one ocean to another. The average width of continental shelves is about 80 km. 

The shelves are almost absent or very narrow along some of the margins like the coasts of Chile, the west coast of Sumatra, etc. 

Continental Slope

The continental slope connects the continental shelf and the ocean basins. It begins where the bottom of the continental shelf sharply drops off into a steepslope. The gradient of the slope region varies between 2-5°. The depth of the slope region varies between 200 and 3,000 m. The slope boundary indicates the end of the continents. Canyons and trenches are observed in this region. 

Continental rise

The continental rise is an underwater feature found between the continental slope and the abyssal plain. This feature can be found all around the world, and it represents the final stage in the boundary between continents and the deepest part of the ocean. The environment in the continental rise is quite unique, and many oceanographers study it extensively in the hopes of learning more about the ocean and geologic history.

Abyssal plain

An abyssal plain is an underwater plain on the deep ocean floor, usually found at depths between 3,000 metres (9,800 ft) and 6,000 metres (20,000 ft). Lying generally between the foot of a continental rise and a mid-ocean ridge, abyssal plains cover more than 50% of the Earth’s surface. They are among the flattest, smoothest, and least explored regions on Earth. Abyssal plains are key geologic elements of oceanic basins (the other elements being an elevated mid-ocean ridge and flanking abyssal hills).

2.4k views
With reference to Biofertilisers, consider the
following statements:

1. These are biologically active substances that
are applied in soil to enrich the soil with nitrogen, phosphorous, organic matter.

2. Nostoc and Anabaena are free living photosynthetic organisms capable of fixing atmospheric nitrogen.

3. Mycorrhizal fungi acts as bio-fertliser and are
known to occur naturally on the roots of forest
trees and crop plants.

Which of the statement(s) given above is/are correct?
a) 1 only
b) 1 and 2 only
c) 2 and 3 only
d) 1, 2 and 3

D

Correct

Using Biofertilizer is one of the energy efficient and pollution free method is to exploit the ability of certain microorganisms like bacteria, algae and fungi to fix atmospheric nitrogen, solubilize phosphorus, decompose organic material or oxidize sulphur in the soil. When they are applied in the soil, they enhance growth and yield of crops, improve soil fertility and reduce pollution. Thus bio-fertilizers are living or biologically active products or microbial inoculants of bacteria, algae and fungi (separately or in combination) which are able to enrich the soil with nitrogen, phosphorus, organic matter etc.

Blue green algae (BGA or cyanobacteria) like Nostoc and Anabaena are free livingp photosyntheticorganisms also capable of fixing atmospheric nitrogen. In the flooded rice fields blue green algae serves as a nitrogen biofertilizer.

Mycorrhizal fungi acts as biofertilizer and occur naturally on roots of forest trees and crop plants. Mycorrhizal fungi are hosted by the trees in their roots to receive nutrients from the soil. These fungi provide essential nutrients like nitrogen, phosphorus and potassium from soil in exchange for carbon from the tree.

2.4k views
Consider the following statements about a National Park:

1. It is inscribed as World Heritage Site by UNESCO.

2. Himalayan Musk Deer and Himalayan Tahr are important fauna found in this park.

3. It is situated in the transition zone between Western and Eastern Himalayas.

4. The Park is opened for the visitors only for 6 months in a year.

The above statements refer to which of the
following National Park?

a) Gangotri National park
b) Pin Valley National Park.
c) Dachigam National Park
d) Nanda Devi National Park
2.4k views
What of the following correctly describes the
concept of 'Anchor Banks'?

a) Banks that will offer insurance services along with normal banking services.
b) Banks that will take drive of consolidation process among public sector banks.
c) Banks that will purchase rising NPAs of Public Sector Banks.
d) Banks to lend for short-term and less-risky sectors of economy.
2.4k views
Consider the following protected areas:

1. Nagarhole National Park
2. Mukurthi National Park
3. Silent Valley National Park

Which of the given above protected area(s)is/are located in the state of Karnataka?

a) 1 only
b) 1 and 3 only
c) 2 and 3 only
d) None of the above
2.4k views
Consider the following statements wrt. Carrying Capacity: 

1. The population of any species will decrease below carrying capacity and will increase above carrying capacity.

2. Carrying capacity varies for different species and is affected by food availability, water
supply and living space.

Which of the statement(s) given above is/are correct?

a) 1 only
b) 2 only
c) Both 1 and 2
d) Neither 1 nor 2
2.4k views
Consider the following wildlife sanctuaries:

1. Bura-Chapori Wildlife Sanctuary
2. Laokhowa Wildlife Sanctuary
3. The Hoollongapar Gibbon Sanctuary
4. Gumti Wildlife Sanctuary
5. Sepahijala Wildlife Sanctuary

Which of the above wildlife Sanctuaries are not located in Assam?

a) 1 and 3 only
b) 1, 3 and 4 only
c) 1, 2 and 3 only
d) 4 and 5 only
2.3k views
Which of the following can be used for increasing soil fertility?

1. Bacteria
2. Algae
3. Fungi
4. Virus

Select the correct answer from the codes given below:

a) 1 and 2 only
b) 1, 2 and 3 only
c) 1, 3 and 4 only
d) 1, 2, 3 and 4
2.3k views

navinprasad1011649said

#42)According to Buddhist tradition, Maha Sammata was

A. The first monarch of the world

B. A compassionate Bodhisattva who renounced his life for the Sangha

C. A character of Gautam Buddha’s previous life

D. None of the above


A - First monarch

2.3k views
Consider the following statements about a National Park:

1. It is inscribed as World Heritage Site by UNESCO.

2. Himalayan Musk Deer and Himalayan Tahr are important fauna found in this park.

3. It is situated in the transition zone between Western and Eastern Himalayas.

4. The Park is opened for the visitors only for 6 months in a year.

The above statements refer to which of the
following National Park?

a) Gangotri National park
b) Pin Valley National Park.
c) Dachigam National Park
d) Nanda Devi National Park

d

Correct. Opens only in summer months. 

2.4k views
What of the following correctly describes the
concept of 'Anchor Banks'?

a) Banks that will offer insurance services along with normal banking services.
b) Banks that will take drive of consolidation process among public sector banks.
c) Banks that will purchase rising NPAs of Public Sector Banks.
d) Banks to lend for short-term and less-risky sectors of economy.

B?

or A?

B is correct. 

2.4k views
Consider the following protected areas:

1. Nagarhole National Park
2. Mukurthi National Park
3. Silent Valley National Park

Which of the given above protected area(s)is/are located in the state of Karnataka?

a) 1 only
b) 1 and 3 only
c) 2 and 3 only
d) None of the above

a

silent valley- kerala

Correct. 

Nagarhole - KArnataka 

Mukurthi - TamilNadu

2.4k views
Which of the following gases are capable of trapping the outgoing infrared radiation from the earth?

1. Methane
2. Nitrous oxide
3. Water vapour
4. Chlorofluorocarbons
5. Carbon dioxide

Select the correct answer from the codes given below:
a) 1 and 2 only
b) 1, 2 and 3 only
c) 2, 3, 4 and 5 only
d) 1, 2, 3, 4 and 5
2.3k views
Which of the following statements is /are correct about Sentinelese Tribes?

1. It is a tribe that lives in Lakshwadeep Islands.
2. They are hunter-gatherers and are fiercely hostile to outside contact.
3. Traditional areas occupied by the tribes have been declared as reserves.

Choose the correct answer using the codes given below:
a) 1, 2 and 3
b) 1 and 3 only
c) 2 and 3 only
d) 2 only
2.3k views
Consider the following statements about the Predators in an ecosystem:

1. They help in transferring the energy fixed by the lower trophic level to higher trophic level.
2. They help in checking the growth of prey population.
3. They help in preventing the extinction of species due to increment in inter-specific competition.

Which of the given above statement(s) is/are correct?

a) 1 only
b) 1 and 2 only
c) 3 only
d) 1, 2 and 3
2.3k views
Which of the following are herbivores marine mammals?

1. Sea cows
2. Sea lion
3. Seals
4. Manatees
5. Dugong

Choose the correct answer from the given codes given below:
a) 1, 2 and 4 only
b) 1, 4 and 5 only
c) 3, 4 and 5 only
d) 2, 3, 4 and 5 only
2.3k views

Which of the following is the correct meaning of Bio-prospecting?


a) The process of exploring molecular, genetic and species-level diversity for products of economic importance.
b) The process of exploring molecular, genetic and species-level diversity for conservationof biodiversity.
c) The process of exploring molecular, genetic and species-level diversity to look for reasons of extinction of certain species.
d) The process of exploring molecular, genetic and species-level diversity to find origins of the earth.

2.3k views
Consider the following statements about the Predators in an ecosystem:

1. They help in transferring the energy fixed by the lower trophic level to higher trophic level.
2. They help in checking the growth of prey population.
3. They help in preventing the extinction of species due to increment in inter-specific competition.

Which of the given above statement(s) is/are correct?

a) 1 only
b) 1 and 2 only
c) 3 only
d) 1, 2 and 3

b?


Correct

By consuming the food from lower tropics they derive the energy from it and transfer it to higher trophic level. As they keep the prey population under control, the interspecific competition gets reduced among the prey species.

2.2k views
Which of the following are herbivores marine mammals?

1. Sea cows
2. Sea lion
3. Seals
4. Manatees
5. Dugong

Choose the correct answer from the given codes given below:
a) 1, 2 and 4 only
b) 1, 4 and 5 only
c) 3, 4 and 5 only
d) 2, 3, 4 and 5 only

b

Correct

Sea Cows, Manatees and Dugongs are herbivores marine mammals. They make their living grazing on sea grasses, marine algae in coastal temperate and tropical waters of North America, Asia and Africa. Seals and sea lions come under the order of carnivores.

2.2k views

Which of the following is the correct meaning of Bio-prospecting?


a) The process of exploring molecular, genetic and species-level diversity for products of economic importance.
b) The process of exploring molecular, genetic and species-level diversity for conservationof biodiversity.
c) The process of exploring molecular, genetic and species-level diversity to look for reasons of extinction of certain species.
d) The process of exploring molecular, genetic and species-level diversity to find origins of the earth.

a

Correct - A

2.2k views
Which of the following statements is /are correct about Sentinelese Tribes?

1. It is a tribe that lives in Lakshwadeep Islands.
2. They are hunter-gatherers and are fiercely hostile to outside contact.
3. Traditional areas occupied by the tribes have been declared as reserves.

Choose the correct answer using the codes given below:
a) 1, 2 and 3
b) 1 and 3 only
c) 2 and 3 only
d) 2 only


C is correct. 

The Sentinelese are a negrito tribe who live on the North Sentinel Island of the Andamans. The inhabitants are connected to the Jarawa on the basis of physical, as well as linguistic similarities. Their numbers are believed to be less than 150 and as low as 40.

They resist contact with outsiders and are frequently seen to be violently reacting to any contacts.

The entire North Sentinel Island along with 5 km coastal sea from high water mark is notified as tribal reserve.

2.2k views
Which among the following pieces of architecture belongs to the Mauryan period?

a) Queen and attendants in Badami
b) Bhitargaon temple
c) Yakshi figure from Didarganj
d) Lepakshi temple
2.2k views
The 1938 Session of Indian National Congress was of great significance in the history of Indian Freedom Movement due to:

a) The iteration of the meaning of Swaraj for the first time in the Congress session.
b) The Congress committed itself to economic planning and setting up a National Planning Committee.
c) Decision to launch the Quit India movement.
d) Decision to participate in the Round Table Conference in London was taken.
2.2k views
Which of the following literary source(s) have reference to the Age of Mauryas?
1. Arthashastra
2. Indika
3. Mudrarakshasa
4. Amarakosha

Select the correct answer using the codes given below:
a) 1 only
b) 1, 2 and 3 only
c) 1 and 2 only
d) 1, 2, 3 and 4
2.2k views
Consider the following statements about Photosynthesis:

1. Only plants with green leaves are capable of photosynthesis to produce energy.
2. Higher concentration of carbon dioxide helps in increasing the rate of photosynthesis.
3. Both glucose and oxygen are by-products of photosynthesis.

Which of the statement(s) given above is/are correct?
a) 1and 2 only
b) 1 only
c) 2 and 3 only
d) 1 and 3 only
2.2k views
Which of the following best describes the term 'Phantom Capital'?

a) It is the investment done by the government in the loss making PSUs.
b) It is the revenue generated by selling of shares of large PSUs by the government.
c) It is a strategic funding by consortium of corporate companies in prospective startups.
d) The capital invested by foreign companies into shell companies in order to avoid corporate taxes.
2.6k views
With reference to the High Temperature Fuel Cell system in news, consider the following statements:

1. It is developed by the Council of Scientific and Industrial Research in partnership with ISRO.
2. The system generates power using bio-methane, with heat and water as bi-products.

Which of the statement(s) given above is/are correct?
a) 1 only
b) 2 only
c) Both 1 and 2
d) Neither 1 nor 2
2.6k views

For which Question?

2.6k views
Which of the following best describes the term 'Phantom Capital'?

a) It is the investment done by the government in the loss making PSUs.
b) It is the revenue generated by selling of shares of large PSUs by the government.
c) It is a strategic funding by consortium of corporate companies in prospective startups.
d) The capital invested by foreign companies into shell companies in order to avoid corporate taxes.

D

Correct.


Phantom investments are foreign investment that passes through empty corporate shells that don't do any real business. These shells, also called special purpose entities, have no real business activities. Rather, they carry out holding activities, conduct intrafirm financing, or manage intangible assets—often to minimize multinationals’ global tax bill. Such financial and tax engineering blurs traditional FDI statistics and makes it difficult to understand genuine economic integration. These investments in empty corporate shells in tax havens undermine tax collection in advanced emerging market, and developing economies.

2.5k views
With reference to the High Temperature Fuel Cell system in news, consider the following statements:

1. It is developed by the Council of Scientific and Industrial Research in partnership with ISRO.
2. The system generates power using bio-methane, with heat and water as bi-products.

Which of the statement(s) given above is/are correct?
a) 1 only
b) 2 only
c) Both 1 and 2
d) Neither 1 nor 2

B..

On the occasion of the Council of Scientific and Industrial Research (CSIR) Foundation Day, the President of India introduced India’s first indigenously developed high-temperature based Fuel Cell System. It is developed under the Public-Private Partnership (PPP) model by the Council of Scientific and Industrial Research (CSIR) in partnership with Indian industries. It is built under India’s flagship programme named ‘New Millennium Indian Technology Leadership Initiative (NMITLI)’. 

It is a 5.0 kW fuel cell system that generates power in a green manner. It takes methanol or bio-methane as the input and produces heat and water as its bi-products, which can be further used. This helps to attain an efficiency that is greater than 70%, which is difficult to achieve by other energy sources. Fuel cell produces electricity in the form of Direct current (DC). In a direct circuit electrons are flow in only one direction. The Hydrogen Fuel Cells can be used in many applications like transport, backup power to produce electricity during the failure of the electricity grid.


2.5k views

tamtarsaid

Q. The visit of foreigners in India to see the various places/events in the country, amounts to which of the following in terms of economy:

(a) Production

(b) Consumption

(c) Import

(d) Export

D.. 

2.5k views

Gochisaid

It's easy to get out of touch from Prelims preparation. We should have a judicious mix. Let's continue to post Questions and contribute!

Agreed. 

2.3k views

Agogsaid

@upsc2020 S Sallsbury a tropical grassland??


No, Salisbury is a chalk grassland. These have thin lime rich soil. 

2.3k views


Which of the following are the compulsory provisions of the 73rd Constitutional amendment act? 

1. Reservation of one-third seats for women in panchayats at all the three levels. 
2. Fixing tenure of 5 years for panchayats at all levels and holding fresh elections within six months in the event of suppression of any panchayat. 

Which of the statements given above is/are correct? 
a) 1 only 
b) 2 only 
c) Both 1 and 2 
d) Neither 1 nor 2 
 

2.4k views
‘Kharchi Puja’ recently seen in news is associated with 
a) Telangana 
b) Rajasthan 
c) Bihar 
d) Tripura
2.4k views

The term 'Contango' is associated with 

a) Stock Market 

b) Environmental Pollution 

c) Pandemics 

d) Military Theory

2.4k views


Which of the following are the compulsory provisions of the 73rd Constitutional amendment act? 

1. Reservation of one-third seats for women in panchayats at all the three levels. 
2. Fixing tenure of 5 years for panchayats at all levels and holding fresh elections within six months in the event of suppression of any panchayat. 

Which of the statements given above is/are correct? 
a) 1 only 
b) 2 only 
c) Both 1 and 2 
d) Neither 1 nor 2 
 

C?

Correct.

2.3k views
‘Kharchi Puja’ recently seen in news is associated with 
a) Telangana 
b) Rajasthan 
c) Bihar 
d) Tripura

D tripura

Correct. 

 Kharchi Puja is one of Tripura’s most important festivals involving the worship of ‘the 14 Gods’. The festival also highlights the equal participation of people from both tribal and non-tribal communities.

2.3k views

Which of the following statements are correct with respect to State Legislature? 

1.The constitution does not stipulate any maximum recess permitted between two sessions of Legislative Council in the State.

2.The constitution mandates a minimum notice period to be given to the MLA’s before a session of the house is convened. 

A.1 only

B.2 only

C.Both 1 and 2

D.Neither 1 nor 2

A ? This one's tricky. 

2.5k views
Which of the following National Parks are also
designated as Marine Protected Areas?
1. Campbell Bay
2. Bhitarkanika
3. Sundarbans
4. Rani Jhansi
5. Saddle Peak
Select the correct answer using the code given
below.
(a) 1, 2 and 5 only
(b) 1, 3 and 4 only
(c) 2, 3, 4 and 5 only
(d) 1, 2, 3, 4 and 5                                                                                                                        .
2.6k views
The advantages of manure is/are:
1. It enhances the water holding capacity of the soil. 
2. It makes the soil porous due to which exchange of gases becomes easy. 
3. It increases the number of friendly microbes. 
4. It improves the texture of the soil. 

Option:
A.  Only 1 is correct 
B. 1, 3 and 4 are correct
C.  1, 2 and 3 are correct
D. All are correct
2.6k views
Which of the following National Parks are also
designated as Marine Protected Areas?
1. Campbell Bay
2. Bhitarkanika
3. Sundarbans
4. Rani Jhansi
5. Saddle Peak
Select the correct answer using the code given
below.
(a) 1, 2 and 5 only
(b) 1, 3 and 4 only
(c) 2, 3, 4 and 5 only
(d) 1, 2, 3, 4 and 5                                                                                                                        .

D? Bhitarkanika and Sunderbans have to be MPA's given the biodiversity and wetlands. Leaves C and D. Going with D because Campbell Bay has bay in it. Has to be protected. 

D is correct.


http://wiienvis.nic.in/Database/MPA_8098.aspx

2.7k views
The advantages of manure is/are:
1. It enhances the water holding capacity of the soil. 
2. It makes the soil porous due to which exchange of gases becomes easy. 
3. It increases the number of friendly microbes. 
4. It improves the texture of the soil. 

Option:
A.  Only 1 is correct 
B. 1, 3 and 4 are correct
C.  1, 2 and 3 are correct
D. All are correct

b or d

not really sure about statement 2.

d?

D - All are correct. 

The addition of organic matter to the soil usually increases the water holding capacity of the soil. This is because the addition of organic matter increases the number of micropores and macropores in the soil either by “gluing” soil particles together or by creating favourable living conditions for soil organisms. Increased organic matter contributes indirectly to soil porosity (via increased soil faunal activity). Fresh organic matter stimulates the activity of macrofauna such as earthworms, which create burrows lined with the glue-like secretion from their bodies. 

Statements picked from NCERT 8th

 

@AlexanderSupertramp @Homo Neanderthalensis @Patootie 

4.9k views

With reference to sprinkler irrigation, consider the following statements:

1. Sprinklers are best suited to sandy soils.

2. It is the best technique for watering fruit plants.

Which of the statements above is / are correct?

a. 1 only

b. 2 only

c.  1 and 2 Both

d. Neither 1 nor 2

2.6k views

With reference to composite fish culture, consider the following statements:

1. Fast growing compatible species of fish of similar feeding habits are stocked together.

2. The composite fish culture system is a technology developed by the Indian Council of Agricultural Research.

Which of the statements above is / are correct?

a. 1 only

b. 2 only

c.  1 and 2 Both

d. Neither 1 nor 2

2.6k views

With reference to sprinkler irrigation, consider the following statements:

1. Sprinklers are best suited to sandy soils.

2. It is the best technique for watering fruit plants.

Which of the statements above is / are correct?

a. 1 only

b. 2 only

c.  1 and 2 Both

d. Neither 1 nor 2

A is correct

Best technique for watering fruit plants is Drip Irrigation. 

2.4k views

With reference to composite fish culture, consider the following statements:

1. Fast growing compatible species of fish of similar feeding habits are stocked together.

2. The composite fish culture system is a technology developed by the Indian Council of Agricultural Research.

Which of the statements above is / are correct?

a. 1 only

b. 2 only

c.  1 and 2 Both

d. Neither 1 nor 2

@upsc2020 a?

B is correct answer.

Fast growing compatible species of fish ofdifferentfeeding habits are stocked together.If all fish have similar feeding habits they would compete for food and kill each other. In this system, they introduce fish which have varied patterns of eating. Some eat from top layer of pond, some from bottom most.

2.4k views

Which among the following are capable of forming a cell wall?

1. Bacteria

2. Human Cells

3. Virus

Select the correct answer.

a. 1 and 2 only

b. 1 only

c. 1 and 3 only

d. 1, 2 and 3


2.3k views

With reference to composite fish culture, consider the following statements:

1. Fast growing compatible species of fish of similar feeding habits are stocked together.

2. The composite fish culture system is a technology developed by the Indian Council of Agricultural Research.

Which of the statements above is / are correct?

a. 1 only

b. 2 only

c.  1 and 2 Both

d. Neither 1 nor 2

@upsc2020 a?

B is correct answer.

Fast growing compatible species of fish ofdifferentfeeding habits are stocked together.If all fish have similar feeding habits they would compete for food and kill each other. In this system, they introduce fish which have varied patterns of eating. Some eat from top layer of pond, some from bottom most.

Was it developed by ICAR?

Yes in the 1970s/80s, it was developed by ICAR. 

2.3k views

Which among the following are capable of forming a cell wall?

1. Bacteria

2. Human Cells

3. Virus

Select the correct answer.

a. 1 and 2 only

b. 1 only

c. 1 and 3 only

d. 1, 2 and 3


B

Correct. 


2.2k views

With reference to adaptive immunity, consider the following statements.

1. It is a short lived form of immune response. 

2. It is the body's first line of defense against germs.

3. Vaccination is a form of adaptive immunity. 

Select the correct answer.


a. 1 and 2 only


b. 3 only


c. 2 and 3 only


d. 1, 2 and 3

2.9k views

With reference to adaptive immunity, consider the following statements.

1. It is a short lived form of immune response. 

2. It is the body's first line of defense against germs.

3. Vaccination is a form of adaptive immunity. 

Select the correct answer.


a. 1 and 2 only


b. 3 only


c. 2 and 3 only


d. 1, 2 and 3

Absolutely right. 

2.8k views
Which of the following statements is
correct?
a) Both the National Population Register and the
National Register of Citizens (NRC) contain
demographic and biometric details of the
registered person.

b) The National Population Register (NPR)is
being prepared by the Office of Registrar
General and Census Commissioner of India.

c) Provision related to the National Register of
Indian Citizens (NRC) was added into the
Citizenship Act via amendment in 2003.

d) All of the statements given above are correct.


This question is from Forum Simulator Test-8. The answer in the key is -->A

Can someone clarify please? Why is Option B wrong? Isn't NPR being conducted by Office of Registrar
General and Census Commissioner of India?

Also, C is correct. 2003 Amendment (notified in 2004) introduced NRIC?

Yes, c seems to be correct.

Section 14 a- Central Government may maintain a National Register of Indian Citizens and for that purpose establish a National Registration Authority.

2.7k views
Which of the following statements is
correct?
a) Both the National Population Register and the
National Register of Citizens (NRC) contain
demographic and biometric details of the
registered person.

b) The National Population Register (NPR)is
being prepared by the Office of Registrar
General and Census Commissioner of India.

c) Provision related to the National Register of
Indian Citizens (NRC) was added into the
Citizenship Act via amendment in 2003.

d) All of the statements given above are correct.


This question is from Forum Simulator Test-8. The answer in the key is -->A

Can someone clarify please? Why is Option B wrong? Isn't NPR being conducted by Office of Registrar
General and Census Commissioner of India?

Also, C is correct. 2003 Amendment (notified in 2004) introduced NRIC?

Yes, c seems to be correct.

Section 14 a- Central Government may maintain a National Register of Indian Citizens and for that purpose establish a National Registration Authority.

Yes. Registrar General of Births and Deaths is the National Registration Authority. The office of Registrar General and Census Commissioner is clubbed into one office - Hence RGCC. NPR is hence being conducted by RGCC? 

Yes, RGCC will conduct.

2.7k views
Consider the following statements:

1. GM  crops could lead to erosion of biodiversity and pollute gene pools of endangered plant species.

2. Gene Use Restriction Technology has been permitted by India recently under Plant Varieties Protection and Farmers' Rights Act. 

Which of the statement/s is correct? 

A. 1 only

B. 2 only 

C. 1 and 2 Both

D. Neither 1 nor 2
2.7k views
Consider the following statements:

1. GM  crops could lead to erosion of biodiversity and pollute gene pools of endangered plant species.

2. Gene Use Restriction Technology has been permitted by India recently under Plant Varieties Protection and Farmers' Rights Act. 

Which of the statement/s is correct? 

A. 1 only

B. 2 only 

C. 1 and 2 Both

D. Neither 1 nor 2

A? Not heard of 2nd option.

A is correct. 

India has prohibited the registration of plant varieties which are made using GURT like the terminator technology. 

2.7k views
With reference to the Private member’s bill, consider the following statements:
1. It can only be introduced in the House by members other than members of rulling  government.
2. Private Member Bills can be a Constitutional Amendment Bill.
3. No Private Member Bill has been passed till date by Parliament.

Which of the above statements is/are NOT correct?
a. 1 only
b. 2 only
c. 1 and 3 only
d. None of these
2.4k views
India has signed a MoU for the implementation of Mahatma Gandhi International Convention Centre (MGICC) with?

a. Uganda
b. Nigeria
c. Niger
d. Kenya
2.4k views
What is common to the places known as Ganweriwala and Sadikpur Sinauli?

a. Recently discovered uranium deposits
b. Water reservoirs
c. Harappan sites
d. Tropical rain forests
2.4k views
Consider the following.
1. Arctic Council
2. Shanghai Co-Operation Organizations
3. Organization of Islamic Cooperation (OIC)

In which of the above organizations, India has an observer status, not a permanent member status?

a. 1 and 2 only
b. 1 and 3 only
c. 1 only
d. 2 and 3 only
2.4k views
Consider the following statements regarding Seagrasses.

1. Seagrasses are important for environment as they are one of the largest absorbers of carbon.

2. South-Eastern coast of India hosts the maximum extent of seagrass beds in India.

Which of the above statements is/are correct?
a. 1 only
b. 2 only
c. Both 1 and 2
d. Neither 1 nor 2
2.4k views

Hi please reply to the post. The Double quotes sign - use that. 

Wouldn't know otherwise answer is to which Question.

2.6k views
What is common to the places known as Ganweriwala and Sadikpur Sinauli?

a. Recently discovered uranium deposits
b. Water reservoirs
c. Harappan sites
d. Tropical rain forests

C. Sadikpur Sinauli was recently in news

That's correct.

2.9k views
Consider the following statements regarding Seagrasses.

1. Seagrasses are important for environment as they are one of the largest absorbers of carbon.

2. South-Eastern coast of India hosts the maximum extent of seagrass beds in India.

Which of the above statements is/are correct?
a. 1 only
b. 2 only
c. Both 1 and 2
d. Neither 1 nor 2

C. Coz sea cow dugong would prolly need good amount of them

Correct. 

Palk Bay and the Gulf of Mannar, in Tamil Nadu, on the south-east coast of India, host the maximum extent (3,000 hectares) of seagrass beds in India.

2.8k views
Consider the following statements regarding Eklavya Model Residential Schools(EMRSs) scheme.

1. The scheme of EMRSs was introduced in the year 1997-98 to ensure tribal students get access to quality education in the remote tribal areas.
2. The scheme is being implemented by the Ministry of Human Resource Department.
3. Every block with more than 50% ST population or at least 20,000 tribal persons will have an EMRS by 2022.

Which of the above statements is/are correct?
a. 1 and 2 only
b. 1 only
c. 3 only
d. 1, 2 and 3
2.6k views
According to the constitution of India, which of the following authorities is/are barred from future appointment to any public offices by the central and state government after retirement? 

 1. Finance Commission Chairman
 2. Chairman of Union Public Service Commission
 3. Comptroller and Auditor General of India

Select the correct answer using the code given below.
a. 1 and 2 only
b. 2 and 3 only
C. 1 and 3 only
d. 1, 2 and 3
2.6k views
Consider the following statements regarding Eklavya Model Residential Schools(EMRSs) scheme.

1. The scheme of EMRSs was introduced in the year 1997-98 to ensure tribal students get access to quality education in the remote tribal areas.
2. The scheme is being implemented by the Ministry of Human Resource Department.
3. Every block with more than 50% ST population or at least 20,000 tribal persons will have an EMRS by 2022.

Which of the above statements is/are correct?
a. 1 and 2 only
b. 1 only
c. 3 only
d. 1, 2 and 3

B?

Correct.

In the context of establishing quality residential schools for the promotion of education, Eklavya Model Residential Schools (EMRSs) for ST students are set up in States / UTs with provisioning of funds through “Grants under Article 275(1) of the Constitution”.

The scheme is being implemented by the Ministry of Tribal Affairs, Government of India.

The establishing of EMRSs are based on demand of the concerned States/UTs with availability of land as an essential attribute. It is mandatory for the state to provide 15 to 20 acres of land to implement this scheme.

This scheme was introduced in the year 1997-98 to ensure tribal students get access to quality education in the remote tribal areas.

2.6k views

upsc2020said

Consider the following statements regarding Eklavya Model Residential Schools(EMRSs) scheme.

1. The scheme of EMRSs was introduced in the year 1997-98 to ensure tribal students get access to quality education in the remote tribal areas.
2. The scheme is being implemented by the Ministry of Human Resource Department.
3. Every block with more than 50% ST population or at least 20,000 tribal persons will have an EMRS by 2022.

Which of the above statements is/are correct?
a. 1 and 2 only
b. 1 only
c. 3 only
d. 1, 2 and 3

B?

Correct.

In the context of establishing quality residential schools for the promotion of education, Eklavya Model Residential Schools (EMRSs) for ST students are set up in States / UTs with provisioning of funds through “Grants under Article 275(1) of the Constitution”.

The scheme is being implemented by the Ministry of Tribal Affairs, Government of India.

The establishing of EMRSs are based on demand of the concerned States/UTs with availability of land as an essential attribute. It is mandatory for the state to provide 15 to 20 acres of land to implement this scheme.

This scheme was introduced in the year 1997-98 to ensure tribal students get access to quality education in the remote tribal areas.

why is 3rd statement wrong?

Every block with more than 50% ST population and at least 20,000 tribal persons, will have an Eklavya Model Residential School by the year 2022. 

Both criteria are essential. 

2.5k views
With reference to Raja Ram Mohan Roy, consider the following statements:

1. He established the Brahmo Samaj in 1828, which acted against the evil practices  that existed in the Hindu society, especially, the practice of ‘Sati’.
2. In 1803, he published his first book Tuhfat-ul- Muwahhidin in which he argues for 
monotheism.
3. He considered departure from the scriptures as justified, if reason demanded it.

Which of the statements given above is/are correct?
a. 1 and 2 only
b. 2 and 3 only
c. 1 and 3 only
d. 1, 2 and 3
2.8k views
Which of the following is the basic difference between edge computing and cloud 
computing?

1. The basic difference between edge computing and cloud computing lies in where the 
data processing takes place.
2. An edge computing network reduces the amount of data that travels over the 
network as compared to cloud computing.
3. Edge computing can be performed at full potential even at 3G and 4G networks.

Select the correct answer.

a. 1 and 2 only
b. 2 and 3 only
c. 1 and 3 only
d. 1, 2 and 3
2.8k views

Bheda Bheda Philosophy was propounded by

A. Madhabacharya

B. Nimbarkara

C. Ramanuja

D. Vallabacharya

2.8k views
Which of the following is the basic difference between edge computing and cloud 
computing?

1. The basic difference between edge computing and cloud computing lies in where the 
data processing takes place.
2. An edge computing network reduces the amount of data that travels over the 
network as compared to cloud computing.
3. Edge computing can be performed at full potential even at 3G and 4G networks.

Select the correct answer.

a. 1 and 2 only
b. 2 and 3 only
c. 1 and 3 only
d. 1, 2 and 3

A?

Ais correct.

@AlexanderSupertramp 

Edge computing enables data to be analysed, processed, and transferred at the edge of a network. The idea is to analyse data locally, closer to where it is stored, in real-time without latency, rather than send it far away to a centralised data centre. So whether you are streaming a video on Netflix or accessing a library of video games in the cloud, edge computing allows for quicker data processing and content delivery.

The basic difference between edge computing and cloud computing lies in where the data processing takes place.

5G networks are capable of supporting edge workloads. 

2.5k views
Which of the following is the basic difference between edge computing and cloud 
computing?

1. The basic difference between edge computing and cloud computing lies in where the 
data processing takes place.
2. An edge computing network reduces the amount of data that travels over the 
network as compared to cloud computing.
3. Edge computing can be performed at full potential even at 3G and 4G networks.

Select the correct answer.

a. 1 and 2 only
b. 2 and 3 only
c. 1 and 3 only
d. 1, 2 and 3

A?

Ais correct.

@AlexanderSupertramp 

Edge computing enables data to be analysed, processed, and transferred at the edge of a network. The idea is to analyse data locally, closer to where it is stored, in real-time without latency, rather than send it far away to a centralised data centre. So whether you are streaming a video on Netflix or accessing a library of video games in the cloud, edge computing allows for quicker data processing and content delivery.

The basic difference between edge computing and cloud computing lies in where the data processing takes place.

5G networks are capable of supporting edge workloads. 

@upsc2020,I don't think we can say that the last statement is wrong. Since it says edge computing can be performed with 3g or 4g too, we need to say that it doesn't work with 3g or 4g to negate that statement. And as we know it can work on slow networks too, how can we say that statement 3 is wrong?

Am I missing something here?

For edge computing to work successfully, to process that much data, 5G is essential. It cannot work at it's full capacity at 3G. It cannot process that much data at 3G. 

2.9k views
Consider the following.
1. Virupaksha Temple
2. Badava Linga Temple
3. Narasimhan (half-man half-lion) sitting on the coil of a giant seven-headed snake

Which of the above sitescan be found at Hampi, a UNESCO World Heritage Site? 
a. 1 and 2 only
b. 1 only
c. 2 and 3 only
d. 1, 2 and 3
2.8k views
Nur-Sultan Declaration in news recently is related to:
a. Smart Cities
b. Forest Fires
c. Land Degradation
d. Solar Energy
2.8k views
Which of the following can reduce the deficit in Balance of Payments?

1. Reducing import custom duties
2. Restrictive monetary policy
3. Appreciation of domestic currency
Select the correct answer using the code given below.

a. 1 and 3 only
b. 2 and 3 only
c. 2 only
d. 1 and 2 only
2.8k views
Gig Economy is often seen in the news. Which of the following constitute gig economy?

1. Uber drivers and Swiggy delivery executives
2. IT Analyst at Google
3. Salon owner

Select the correct answer using the code given below:

a. 1 only
b. 1 and 3 only
c.  2 only
d. 1, 2 and 3
2.8k views
Which of the following do not come under the ambit of RTI?
1. BCCI
2. Intelligence Bureau
3. Office of CJI
4. Central Economic Intelligence Bureau.
5. NGO’s substantially financed by state
government. 
Select the correct answer using the code given 
below:
a) 2, 3 and 5 only
b) 1, 2, 3 and 4 only
c) 1, 2, and 4 only 
d) 1, 2, 3, 4 and 5
2.6k views
Consider the following statement regarding No Confidence Motion: 
1. It is not mentioned in the Constitution. 
2. A special majority is needed to pass 
this motion.

Which of the statements given above is/are 
correct? 
a) 1 only 
b) 2 only 
c) Both 1 and 2 
d) Neither 1 nor 2
2.6k views
Which of the following is not a classical Indian Language recognised by the Government of India?

a) Tamil
b) Sanskrit
c) Prakrit
d) Malayalam
2.6k views
Which of the following are principles of Indian Nuclear Doctrine?

1. Minimum Credible Deterrence
2. No First Use
3. 2nd Strike Retaliation

Select the correct answer from the codes given 
below:

a) 1 and 2 only
b) 2 and 3 only
c) 1 and 3 only
d) 1, 2 and 3

2.6k views
Consider the following statement regarding No Confidence Motion: 
1. It is not mentioned in the Constitution. 
2. A special majority is needed to pass 
this motion.

Which of the statements given above is/are 
correct? 
a) 1 only 
b) 2 only 
c) Both 1 and 2 
d) Neither 1 nor 2

c

A. Simple majority is used 

2.4k views
Which of the following is not a classical Indian Language recognised by the Government of India?

a) Tamil
b) Sanskrit
c) Prakrit
d) Malayalam

C? Prakrit

Correct.

2.4k views
Which of the following statement(s) is/are correct about 'Compensatory Afforestation Fund Act, 2016'?

1. This act seeks to establish the National Compensatory Afforestation Fund under the Public Account of India, and a State Compensatory Afforestation Fund under the Public Account of each state.
2. The National Fund will get 10% of funds collected and the remaining 90% will go to
respective State Funds.

Select the correct answer from the codes given below:

(a) 1 only
(b) 2 only
(c) Both 1 and 2
(d) Neither 1 nor 2
2.4k views
Consider the following statements about 'Indian
Board for Wildlife (IBWL)':

1. It is headed by Union Minister of Environment,
Forest and Climate Change.
2. No alternation of boundaries in national parks
and wildlife sanctuaries can be done without
approval of the NBWL.

Which of the statement(s) given above is/are correct?

(a) 1 only
(b) 2 only
(c) Both 1 and 2
(d) Neither 1 nor 2
2.4k views
Consider the following statements about 'Petro
Plants':

1. These plants require highly fertile land for their
growth.
2. These plants are alternative sources of liquid
fuels.
3. Jatropha curcas is a good example of petro
plant.

Which of the statements given above are correct?
(a) 1 and 2 only
(b) 2 and 3 only
(c) 1 and 3 only
(d) 1, 2 and 3
2.4k views
Which of the following are the 'Ground Water
Contaminants'?

1. Nitrates
2. Pathogens
3. Trace metals
4. Organic compounds

Select the correct answer from the codes given below:

(a) 1, 2, and 3 only
(b) 1, 3 and 4 only
(c) 3 and 4 only
(d) 1, 2, 3 and 4
2.4k views
'Nalgonda Technique' is used for which of the
following purpose?

(a) Protection of birds
(b) Soil Conservation
(c) Water Purification
(d) Biogas energy

2.4k views
Consider the following statements about Taiga
Forest Ecosystem:

1. It is characterized by coniferous forest having
spruce and pines.
2. It is the world's largest biome apart from the
oceans.

Which of the statement(s) given above is/are correct?
(a) 1 only
(b) 2 only
(c) Both 1 and 2
(d) Neither 1 nor 2
2.4k views
Consider the following statement regarding No Confidence Motion: 
1. It is not mentioned in the Constitution. 
2. A special majority is needed to pass 
this motion.

Which of the statements given above is/are 
correct? 
a) 1 only 
b) 2 only 
c) Both 1 and 2 
d) Neither 1 nor 2

c

A. Simple majority is used 

Thanks for correcting. Since it isn't mentioned in Laxmikanth I took it to be special majority since it can destabilise the government.  

Further, found these facts-

  • According to Rule 198(2) states that if the speaker is of theopinion that the motion is in order,he/she shall read the motion to the house, requesting those members who are in favour of it to rise in their places.
  • If at least 50 members support the motion, the Speaker declares that the leave is granted.
  • Once admitted, it has to be passed within 10 days in the house.

 

I just have one doubt here- in the first statement it says that if the speaker is of the opinion... - Her does this mean the speaker has the discretion to admit the motion ( like in the case of impeachment of a judge) or does s/he just have to check if the procedural/administrative requirements are followed.

Yes, it is at the discretion of the speaker. 

2.6k views
Consider the following statements about Taiga
Forest Ecosystem:

1. It is characterized by coniferous forest having
spruce and pines.
2. It is the world's largest biome apart from the
oceans.

Which of the statement(s) given above is/are correct?
(a) 1 only
(b) 2 only
(c) Both 1 and 2
(d) Neither 1 nor 2

a?

C is correct.

Found only in the northern hemisphere [due to great east-west extent. Absent in the southern hemisphere because of the narrowness in the high latitudes]. The predominant vegetation is evergreen coniferous forest. The conifers, which require little moisture are best suited to this type of sub-Arctic climate. The greatest single band of the coniferous forest is the taiga (a Russian word for coniferous forest) in Siberia. In Europe the countries that have a similar type of climate and forests are Sweden and Finland. There are small amounts of natural coniferous forest in Germany, Poland, Switzerland, Austria and other parts of Europe. There are four major species in the coniferous forests – Pine, Fir, e.g. Douglas fir and balsam fir; Spruce and Larch.

The taiga or boreal forest is the world's largest land biome.

2.6k views
Which of the following statement(s) is/are correct about 'Compensatory Afforestation Fund Act, 2016'?

1. This act seeks to establish the National Compensatory Afforestation Fund under the Public Account of India, and a State Compensatory Afforestation Fund under the Public Account of each state.
2. The National Fund will get 10% of funds collected and the remaining 90% will go to
respective State Funds.

Select the correct answer from the codes given below:

(a) 1 only
(b) 2 only
(c) Both 1 and 2
(d) Neither 1 nor 2

c

Correct.

Compensatory Afforestation Fund

The CAF Act was passed by the centre in 2016 and the related rules were notified in 2018.

The CAF Act was enacted to manage the funds collected for compensatory afforestation which till then was managed by ad hoc Compensatory Afforestation Fund Management and Planning Authority (CAMPA).

Compensatory afforestation means that every time forest land is diverted for non-forest purposes such as mining or industry, the user agency pays for planting forests over an equal area of non-forest land, or when such land is not available, twice the area of degraded forest land.

As per the rules, 90% of the CAF money is to be given to the states while 10% is to be retained by the Centre.

The funds can be used for treatment of catchment areas, assisted natural generation, forest management,wildlife protection and management, relocation of villages from protected areas, managing human-wildlife conflicts, training and awareness generation, supply of wood saving devices and allied activities.

The Act seeks to establish the National Compensatory Afforestation Fund under the Public Account of India and a State Compensatory Afforestation Fund under the Public Account of each state.The payments into the funds include compensatory afforestation, NPV, and any project-specific payments. The National Fund will get 10% of funds collected and the remaining 90% will go to respective State Fund. The collected funds will be utilized for afforestation, regeneration of forest ecosystem, wildlife protection and infrastructure development.

2.6k views
Consider the following statements about 'Petro
Plants':

1. These plants require highly fertile land for their
growth.
2. These plants are alternative sources of liquid
fuels.
3. Jatropha curcas is a good example of petro
plant.

Which of the statements given above are correct?
(a) 1 and 2 only
(b) 2 and 3 only
(c) 1 and 3 only
(d) 1, 2 and 3

B

Hydrocarbon producing plants can become alternative energy sources, which can be inexhaustible and ideal for liquid fuel.These plants called petro plants/petro crops can be grown on land which are unfit for agriculture and not covered with forests. Some potential Petro-crop species belong to family Asclepiadaceae and Euphorbiaceae. Jatropacurcas is an important petro plant.

2.6k views
Which of the following are the 'Ground Water
Contaminants'?

1. Nitrates
2. Pathogens
3. Trace metals
4. Organic compounds

Select the correct answer from the codes given below:

(a) 1, 2, and 3 only
(b) 1, 3 and 4 only
(c) 3 and 4 only
(d) 1, 2, 3 and 4

Common groundwater contaminants:

1) Nitrates: Dissolved nitrates commonly contaminate groundwater. High level of nitrates can cause blue baby disease (Methaemoglobinaemia) in children, may form carcinogens and can accelerate eutrophication in surface waters. Sources of nitrates include sewage, fertilizers, air pollution, landfills and industries.

2) Pathogens: Poor hygiene of well and inadequate segregation of drainage charcoal from wells may cause pathogenic contamination of good water seepage from solid waste dumps and municipal drains may also cause pathogenic contamination. Bacteria and viruses cause water borne diseases such as typhoid, cholera, dysentery, polio, and hepatitis may pass into groundwater through discharges from sewage, landfills, septic tanks and livestock shelters.

3) Trace metals: Include lead, mercury, cadmium, copper, chromium and nickel. These metals can be toxic and carcinogenic. Seepage of industrial and mine discharges, fly ash ponds of thermal power plants can lead to metals in groundwater.

4) Organic compounds: Seepage of agricultural run- off loaded with organic compounds like pesticides and may cause pesticide pollution of ground water.

2.6k views
'Nalgonda Technique' is used for which of the
following purpose?

(a) Protection of birds
(b) Soil Conservation
(c) Water Purification
(d) Biogas energy

C

The community used technique for community water supply is called Nalgonda Technique developed by National Environmental Engineering Research Institute (NEERI), Nagpur. It has following components:

- Reactor(s)

- Sump well

- Sludge drying bed.

This is a batch method for community up to 200 populations. The tank is equipped with mechanical agitator operated manually or by electric motor. Water is pumped or poured into the tank and required amount of alum, lime or sodium bicarbonate and bleaching powder is added with constant stirring. The contents are stirred slowly for ten minutes and allowed to settle for two hours. The defluridated supernatant water is withdrawn and supplied. The sludge from the bottom is discarded.

2.6k views
Consider the following statements about 'Indian
Board for Wildlife (IBWL)':

1. It is headed by Union Minister of Environment,
Forest and Climate Change.
2. No alternation of boundaries in national parks
and wildlife sanctuaries can be done without
approval of the NBWL.

Which of the statement(s) given above is/are correct?

(a) 1 only
(b) 2 only
(c) Both 1 and 2
(d) Neither 1 nor 2

B. It is headed by the PM. I guess you mentioned IBWL in place of National Board or Wildlife. Please correct me, if wrong.  

Yes, correct. 

2.6k views
Which of the following are the 'Ground Water
Contaminants'?

1. Nitrates
2. Pathogens
3. Trace metals
4. Organic compounds

Select the correct answer from the codes given below:

(a) 1, 2, and 3 only
(b) 1, 3 and 4 only
(c) 3 and 4 only
(d) 1, 2, 3 and 4

a?

D. Explanation above.

2.6k views
Consider the following statements about the Parliamentary Proceedings:

1. The Parliament cannot initiate contempt proceedings against any Judge of the Supreme Court or of a High Court for the discharge of his 
duties. 
2. The Courts cannot inquire into the proceedings of the Parliament on the ground of any alleged irregularity of the procedure. 

Which of the statements given above is/are correct?

(a) 1 only
(b) 2 only
(c) Both 1 and 2
(d) Neither 1 nor 2
2.6k views
Which of the following statements best describes the term “Operation Vanilla”? 

(a) Affiliation of the island nations of Seychelles, Madagascar, Reunion, Mauritius, Comoros and Mayotte in the Indian Ocean to promote tourism.
(b) SAARC initiative to stabilize the supply of fruits and flowers. 
(c) Humanitarian assistance and disaster relief provided to cyclone hit Madagascar. 
(d) None of the above.
2.6k views
Which of the following crops were cultivated in the Indus Valley Civilization?

1. Pea
2. Wheat
3. Millet

Select the correct answer using the code 
given below:

(a) 1 only
(b) 2 and 3 only
(c) 1 and 2 only
(d) 1, 2 and 3
2.6k views

upsc2020said

Consider the following statements about the Parliamentary Proceedings:

1. The Parliament cannot initiate contempt proceedings against any Judge of the Supreme Court or of a High Court for the discharge of his 
duties. 
2. The Courts cannot inquire into the proceedings of the Parliament on the ground of any alleged irregularity of the procedure. 

Which of the statements given above is/are correct?

(a) 1 only
(b) 2 only
(c) Both 1 and 2
(d) Neither 1 nor 2

C?

Cis correct.

Article 121 – Restriction on discussion in the Parliament - No discussion shall take place in the Parliament with respect to the conduct of any Judge of the Supreme Court or of a High Court in the discharge of his duties except upon a motion for presenting an address to the President praying for the removal of the Judge.

Article 122 – Courts not to inquire into the proceedings of the Parliament - The validity of any proceedings in the Parliament shall not be called in question on the ground of any alleged irregularity of procedure.

@balwintejas 

2.5k views
Which of the following statements best describes the term “Operation Vanilla”? 

(a) Affiliation of the island nations of Seychelles, Madagascar, Reunion, Mauritius, Comoros and Mayotte in the Indian Ocean to promote tourism.
(b) SAARC initiative to stabilize the supply of fruits and flowers. 
(c) Humanitarian assistance and disaster relief provided to cyclone hit Madagascar. 
(d) None of the above.

c

Cis correct.

“Operation Vanilla”  launched by the Indian Navy to help the cyclone-hit Madagascar. INS Airavat is a large amphibious ship that was on a mission-based deployment near Seychelles. Itimmediately diverted to Madagascar to provide assistance to the flood affected people. The assistance of India to Madagascar is in consonance with the Indian Navy's Foreign Co-operation Initiatives, in line with the Prime Minister's vision of 'Security and Growth for all in the Region (SAGAR)

@balwintejas 

2.8k views

upsc2020said

Which of the following crops were cultivated in the Indus Valley Civilization?

1. Pea
2. Wheat
3. Millet

Select the correct answer using the code 
given below:

(a) 1 only
(b) 2 and 3 only
(c) 1 and 2 only
(d) 1, 2 and 3

D

D is correct. 

2.8k views
Consider the following statements about 'Petro
Plants':

1. These plants require highly fertile land for their
growth.
2. These plants are alternative sources of liquid
fuels.
3. Jatropha curcas is a good example of petro
plant.

Which of the statements given above are correct?
(a) 1 and 2 only
(b) 2 and 3 only
(c) 1 and 3 only
(d) 1, 2 and 3

B?

That's correct.

2.7k views
Consider the following statements about the 1000 Springs Initiative:

1. It is an initiative of the Ministry of Jal Shakti.
2. It aims to provide access to piped drinking water to the tribal people living around natural springs.

Which of the statements given above is/are correct?
(a) 1 only
(b) 2 only
(c) Both 1 and 2
(d) Neither 1 nor 2
2.7k views
Consider the following statements about Mission Purvodaya:

1. The Mission aims at the integrated development of the North-Eastern India.
2. It aims to promote horticulture development and tribal handicrafts.
3. It will be implemented by the North Eastern Council.

Which of the statements given above are incorrect?

(a) 1 and 2 only
(b) 2 and 3 only
(c) 1 and 3 only
(d) 1, 2 and 3
2.7k views
Consider the following statements about the Negotiable Warehouse Receipts (NWRs):

1. The Negotiable Warehouse Receipts can be issued in both the physical and the electronic forms.
2. Only warehouses registered with the Warehousing Development and Regulatory Authority can issue the NWRs.
3. The Warehousing Development and Regulatory Authority is a statutory body under the Ministry of Food Processing Industries.

Which of the statements given above is/are correct?
(a) 2 only
(b) 1 and 2 only
(c) 1 and 3 only
(d) 2 and 3 only
2.6k views
Which among the following best describes the concept of "Dark Fibre"?

(a) Optical fibre network used for connecting to the Dark Net.
(b) Unused optical fibre network.
(c) Illegal optical fibre network laid down by the private telecom companies in ecologically sensitive areas.
(d) None of the above.
2.6k views
Which of the following are imposed in case a country’s government is subsidizing its exporters?

(a) Anti-Dumping Duty
(b) Counter-vailing Duty
(c) Safeguard Duty
(d) Customs Duty
2.4k views
Consider the following statements:

1. The main source of World Bank funding is share capital of member countries
2. The main source of funds for International Monetary Fund (IMF) is borrowing from specific member countries.

Select the correct answer using the 
code given below:
(a) 1 only
(b) 2 only
(c) Both 1 & 2
(d) Neither 1 nor 2
2.4k views
Which of the following statements is true regarding ‘replacement level fertility’?

(a) It is the level of fertility at which population remains constant
(b) It is the level of fertility at which population growth remains constant
(c) It is the level of fertility at which a population exactly replaces itself from one generation to another
(d) None of the above
2.4k views
The unemployment caused due to the workers living far from the regions and are unable to move to the locations where jobs are available is an example of which type of unemployment:

(a) Cyclical 
(b) Frictional 
(c) Structural
(d) Disguised

2.4k views
FCI stores the food grains procured for the central pool in the following:

1. FCI’s own godowns
2. Central Warehousing godowns
3. State Warehousing godowns
4. Godowns hired from private sector

Select the correct answer using the code given below:
(a) 1 only
(b) 1, 2 & 3
(c) 2 & 4
(d) All of the above

2.4k views

upsc2020said

FCI stores the food grains procured for the central pool in the following:

1. FCI’s own godowns
2. Central Warehousing godowns
3. State Warehousing godowns
4. Godowns hired from private sector

Select the correct answer using the code given below:
(a) 1 only
(b) 1, 2 & 3
(c) 2 & 4
(d) All of the above

D

Correct. Taken from Economic Survey

2.7k views

upsc2020said

The unemployment caused due to the workers living far from the regions and are unable to move to the locations where jobs are available is an example of which type of unemployment:

(a) Cyclical 
(b) Frictional 
(c) Structural
(d) Disguised

C

Correct.

2.7k views

upsc2020said

Which of the following statements is true regarding ‘replacement level fertility’?

(a) It is the level of fertility at which population remains constant
(b) It is the level of fertility at which population growth remains constant
(c) It is the level of fertility at which a population exactly replaces itself from one generation to another
(d) None of the above

C. It is 2.1

Correct. 

Replacement level fertility is usually marked at 2.1. But the required replacement level fertility for India is higher than the usual benchmark of 2.1. The reason is the skewed sex ratio, because of which a woman would have to give birth to more than 2.1 children in order for the population to replace itself. Estimates suggest that the effective replacement level fertility after taking into account the skewed sex ratio could be around 2.15 to 2.2 for India with a sex ratio of 1.11 (Replacement level fertility is the level of fertility at which a population exactly replaces itself from one generation to the next.)

2.7k views

upsc2020said

Consider the following statements:

1. The main source of World Bank funding is share capital of member countries
2. The main source of funds for International Monetary Fund (IMF) is borrowing from specific member countries.

Select the correct answer using the 
code given below:
(a) 1 only
(b) 2 only
(c) Both 1 & 2
(d) Neither 1 nor 2

D?

Read somewhere that WB borrows large chunk of its capital from the market.


Correct. 

(d)

World Bank has two sources of funds:

1. Share capital of each member country based on their share in GDP

2. Issuance of bonds in the international financial markets

Out of the above two sources, main source is issuance of bonds (debt finance)

IMF has again two sources of funds

1. QUOTA (every member country has put/subscribed funds)

2. Borrowing from specific member countries

Out of the above two sources, main source is QUOTA

2.7k views
Which of the following statements are correct regarding Black Carbon?

1. It contributes to warming by converting incoming solar radiation to heat. 
2. It also influences cloud formation, and impacts regional circulation and rainfall patterns.
3. When deposited on ice and snow, Black Carbon and co-emitted particles reduce surface albedo and heat the surface, thereby resulting in melting of ice.
4. Per unit of mass, it has a warming impact on climate weaker than carbon dioxide.

Select the correct answer using the code 
given below:

(a) 2, 3 and 4 only
(b) 1, 2 and 3 only
(c) 1, 2 and 4 only
(d) 1, 2, 3 and 4
2.7k views
Consider the following statements:

1. The Speaker of the Lok Sabha is empowered to either withdraw any member from the House or name any persons for suspension for smooth 
functioning of the House.
2. A motion shall be presented in the Lok Sabha for the suspension of the named person and on being passed by the House, results in suspension of the member for the remainder of the Session of the House.
3. The motion for the suspension of the member of the Rajya Sabha is moved by the Chairman and is adopted by the Council. 

Which of the statements given above are  correct?

(a) 1 and 3 only
(b) 1 and 2 only
(c) 2 and 3 only
(d) 1, 2 and 3
2.7k views
Consider the following statements regarding the Fishing Cat:

1. It is a globally vulnerable species that is elusive and found in a very few places in south and south-east Asia.
2. Wild cat species usually hunt on ground, but the Fishing Cat hunts in water.
3. In India, the Fishing Cat is included in Schedule II of the Wildlife (Protection) Act, 1972.

Which of the statements given above are correct?

(a) 1 and 3 only
(b) 1 and 2 only
(c) 2 and 3 only
(d) 1, 2 and 3
2.7k views
Which of the following are Zoonotic viral  diseases?

1. Rabies
2. Leishmaniasis
3. Brucellosis
4. Yellow Fever
5. Influenza

Select the correct answer using the code given below:

(a) 1 and 5 only
(b) 1, 2 and 3 only
(c) 1, 4 and 5 only
(d) 1, 2, 3, 4 and 5
2.7k views
Consider the following pairs:

City in news      Country /Region
1. Idlib                 Iraq 
2. Baghuz           Syria 
3. Hodiedah       Yemen

Which of the pairs given above is/are correctly matched?

(a) 1 only 
(b) 1 and 2 only 
(c) 2 and 3 only 
(d) 1, 2 and 3
2.7k views
Which of the following statements are correct regarding Black Carbon?

1. It contributes to warming by converting incoming solar radiation to heat. 
2. It also influences cloud formation, and impacts regional circulation and rainfall patterns.
3. When deposited on ice and snow, Black Carbon and co-emitted particles reduce surface albedo and heat the surface, thereby resulting in melting of ice.
4. Per unit of mass, it has a warming impact on climate weaker than carbon dioxide.

Select the correct answer using the code 
given below:

(a) 2, 3 and 4 only
(b) 1, 2 and 3 only
(c) 1, 2 and 4 only
(d) 1, 2, 3 and 4

B

Black Carbon is an important contributor to warming, because it is very effective at absorbing light and heating its surroundings. Per unit of mass, Black Carbon has a warming impact on climate that is 460-1,500 times stronger than CO2. 

When suspended in the atmosphere, Black Carbon contributes to warming by converting incoming solar radiation to heat. It also influences cloud formation, and impacts regional circulation and rainfall patterns.

2.5k views
Consider the following statements:

1. The Speaker of the Lok Sabha is empowered to either withdraw any member from the House or name any persons for suspension for smooth 
functioning of the House.
2. A motion shall be presented in the Lok Sabha for the suspension of the named person and on being passed by the House, results in suspension of the member for the remainder of the Session of the House.
3. The motion for the suspension of the member of the Rajya Sabha is moved by the Chairman and is adopted by the Council. 

Which of the statements given above are  correct?

(a) 1 and 3 only
(b) 1 and 2 only
(c) 2 and 3 only
(d) 1, 2 and 3

B

The Chairman may name a member,who disregards the authority of the Chair or abuses the rules of the Council by persistently and willfully obstructing the business of the Council. 

After a member of the Council has been named, then the House may adopt a motion suspending the member from the service of the House for a period not exceeding the remainder of the session. The House may, however, by another motion, terminate the suspension. 

Unlike the Lok Sabha, the motion for the suspension of a member of the Rajya Sabha is not moved by the Chairman, but is adopted by the Council.

2.5k views
Consider the following statements regarding the Fishing Cat:

1. It is a globally vulnerable species that is elusive and found in a very few places in south and south-east Asia.
2. Wild cat species usually hunt on ground, but the Fishing Cat hunts in water.
3. In India, the Fishing Cat is included in Schedule II of the Wildlife (Protection) Act, 1972.

Which of the statements given above are correct?

(a) 1 and 3 only
(b) 1 and 2 only
(c) 2 and 3 only
(d) 1, 2 and 3

B

West Bengal government officially declared the Fishing Cat as the State Animal. From 2016, a programme called “Know Thy Neighbours” (a Fishing Cat monitoring protocol bythe community) was initiated, under which enthusiastic residents were given a camera trap, so that they could monitor their backyard cats. They were also trained on how to identify different individuals and then encouraged them to name them. Pisciculture, intensive aquaculture, shrimp farming etc., are few of the threats to the Fishing Cat.

2.5k views
Which of the following are Zoonotic viral  diseases?

1. Rabies
2. Leishmaniasis
3. Brucellosis
4. Yellow Fever
5. Influenza

Select the correct answer using the code given below:

(a) 1 and 5 only
(b) 1, 2 and 3 only
(c) 1, 4 and 5 only
(d) 1, 2, 3, 4 and 5


2.5k views
Consider the following pairs:

City in news      Country /Region
1. Idlib                 Iraq 
2. Baghuz           Syria 
3. Hodiedah       Yemen

Which of the pairs given above is/are correctly matched?

(a) 1 only 
(b) 1 and 2 only 
(c) 2 and 3 only 
(d) 1, 2 and 3

C

Idlib is a city in Syria. It has been in the news because of the Syrian crisis and the ISIS. 

Baghuz was the last village under the control of the ISIS. 

Hodiedah has been in the news because of a humanitarian crisis that is unfolding in Yemen because of the civil war.

2.5k views
Which of the following statements is/are 
correct?

1. The southern slopes of the Himalayas have thick vegetation, as compared to the northern slopes. 
2. Deodar is endemic to the western Himalayas. 

Select the correct answer using the code 
given below:

(a) 1 only 
(b) 2 only 
(c) Both 1 and 2 
(d) Neither 1 nor 2
2.5k views
Consider the following statements about the Consolidated Sinking Fund:

1. Investment by the state governments in the Consolidated Sinking Fund is mandatory.

2. Consolidated Sinking Funds are maintained by the state governments with the RBI.

Which of the statements given above is/are correct?

(a) 1 only 
(b) 2 only
(c) Both 1 and 2
(d) Neither 1 nor 2
2.5k views
Which of the following soils have properties similar to that of their parent material?

1. Red soils
2. Black soils
3. Alluvial soils
4. Laterite soils

Select the correct answer using the code given below:

(a) 1 and 2 only 
(b) 3 and 4 only 
(c) 1, 2 and 4 only 
(d) 1, 2, 3 and 4
2.5k views

upsc2020said

Consider the following statements about the Consolidated Sinking Fund:

1. Investment by the state governments in the Consolidated Sinking Fund is mandatory.

2. Consolidated Sinking Funds are maintained by the state governments with the RBI.

Which of the statements given above is/are correct?

(a) 1 only 
(b) 2 only
(c) Both 1 and 2
(d) Neither 1 nor 2

B

Correct.

The stategovernments are maintaining The Consolidated Sinking Fund (CSF) and the Guarantee Redemption Fund (GRF) with the Reserve Bank of India (RBI) as buffers for the repayment of their liabilities. Currently, the stategovernments can avail of the Special Drawing Facility (SDF) from the Reserve Bank of India against the collateral of the funds in the CSF andthe GRF. The rate of interest charged is 100 bps below the Repo Rate, at which the Ways and Means Advances are extended to the state governments. 

In order to further incentivize adequate maintenance of these Funds by the state governments and to encourage them to increase the corpus of these Funds, it has been decided to lower the rate of interest on the SDF from 100 bps below the Repo Rate to 200 bps below the Repo Rate.The scheme is, at present, voluntary in nature for the states. However, most states maintain the Consolidated Sinking Fund with the RBI.

3.1k views

mhs11said

Which of the following soils have properties similar to that of their parent material?

1. Red soils
2. Black soils
3. Alluvial soils
4. Laterite soils

Select the correct answer using the code given below:

(a) 1 and 2 only 
(b) 3 and 4 only 
(c) 1, 2 and 4 only 
(d) 1, 2, 3 and 4

C?

Correct. 

Zonal soils develop regionally by weathering of the parent rocks and hence, have properties similar to that of the parent rocks. 

For example – Red soil is formed by the decomposition of granite and gneiss. 

Black soil is formed by the decomposition of basaltic rock. Due to this, they show properties similar to that of their parent rocks. 

However, azonal soils are generally transported soils and they are chemically transformed during the process of transportation, for example – Alluvial soil. 

Laterite is a soil and rock type rich in iron and aluminium and is commonly considered to have formed in hot and wet tropical areas. Nearly all laterites are of rusty-red coloration, because of high ironoxide content. They develop by intensive and prolonged weathering of the underlying parent rock.

3.1k views

mhs11said

Which of the following statements is/are 
correct?

1. The southern slopes of the Himalayas have thick vegetation, as compared to the northern slopes. 
2. Deodar is endemic to the western Himalayas. 

Select the correct answer using the code 
given below:

(a) 1 only 
(b) 2 only 
(c) Both 1 and 2 
(d) Neither 1 nor 2

A?

C is correct. 

The southern slopes of the Himalayas carry a thicker vegetation cover, because of relatively higher precipitation, than the drier north-facing slopes. At higher altitudes, mosses and lichens form part of the tundra vegetation.

Deodar, a highly valued endemic species, grows mainly in the western part of the Himalayan range. Deodar is a durable wood, used mainly in construction activity.

3.1k views
Temperate forests are found in which of the following places in India?

1. Annamalai Hills 
2. Maikal Range 
3. Satpura Range

Select the correct answer using the code 
given below:

(a) 1 only 
(b) 1 and 2 only 
(c) 2 and 3 only 
(d) 1, 2 and 3
3k views
» show previous quotes

A?

Altitude 1000-2000

Eastern Himalayas and western ghats

Other two are mostly tropical dry deciduous

D is correct. 

Southern mountain forests include the forests found in three distinct areas of the Peninsular India, viz., the Western Ghats, the Vindhyas and the Nilgiris. As they are closer to the Tropics and only 1,500 m above the sea level, vegetation is temperate in the higher regions and sub-tropical on the lower regions of the Western Ghats, especially in Kerala, Tamil Nadu and Karnataka. 

The temperate forests are called Sholas in the Nilgiris, Annamalai Hills and Palani hills. Some of the other trees of economic significance of this forest include, magnolia, laurel, cinchona and wattle. 

Such forests are also found in the Satpura and the Maikal ranges.


2.9k views
» show previous quotes

Thanks. 

However I have never read any standard source mentioning these forest in Satpura/maikal areas. 

How do we come across such information?

Even I had not read it anywhere. Came across the below source randomly and was surprised. Re-checked it was in Natural Vegetation Chapter , NCERT. 


2.8k views

mhs11said

» show previous quotes

@upsc2020  Could you tell the answer to this question?

C is correct. 

Balance of payments summarizes an economy’s transactions with the rest of the world for a specified time period. To correct deficit in BoP, a country will have to take steps which would:

a. attract more foreign investment

b. increase exports and decrease unnecessary imports

Reducing import duty would lead to increase in imports and hence would lead to more deficit in BoP.

Second is correct as restrictive monetary policy(lower money supply) would lead to fall in prices of goods in country which would make exports competitive in international market.

Appreciation of increase in value of domestic currency would make exports less competitive.


2.7k views

22 - Polity

23 - Geography

24 - Environment

25 - History

26 - S &T

27 - Economics

28 - CA 

I will post 10/12 questions from all these subjects daily which I feel are important. Please add on as required. 

2.4k views

22 - Polity

23 - Geography

24 - Environment

25 - History

26 - S &T

27 - Economics

28 - CA 

I will post 10/12 questions from all these subjects daily which I feel are important. Please add on as required. 

Starting with Polity. 

2.4k views

Merging of Union Territories requires amendment to which schedule of the Constitution?

a. First

b. Third

c. Seventh

d. Second

2.3k views

Consider the following statements:

1. If a member leaves the party as a result of the merger of party with another party the rule of defection applies if the party whip has been issued.

2. If a member leaves his party temporarily on account of being elected as the presiding officer of the house to maintain the dignity of the position, he is not considered as a defecting member.

Which of the statements given above is/are correct?

a. 1 only

b. 2 only

c. Both 1 and 2

d. Neither 1 nor 2

2.3k views

Who among the following can enjoy parliamentary privileges'?

1. President

2. Prime Minister

3.Attorney General

Select the correct answer using code below.

a. 1 and 2 only

b. 2 and 3 only

c. 1 and 3 only

d. 1, 2 and 3


2.9k views

Consider the following statements:

1. The number of judges of HC will be determined by the order of the President.

2. Strength of Supreme Court Judges can be increased through special majorityof the Parliament.

Which of the statements given above is/are correct?

a. 1 only

b. 2 only

c. Both 1 and 2

d. Neither 1 nor 2

2.9k views

Consider the following statements:

1. Interstate river water disputes fall within the jurisdiction of Art 262 and 131. 

2. Art 131  cannot be used to resolve dispute which involves a private party and a Government on the other side .

Which of the statements given above is/are correct?

a. 1 only

b. 2 only

c. Both 1 and 2

d. Neither 1 nor 2

2.9k views
Which of the following statements is/are correct regarding “Vice President?

1. Electoral College for his election consists of only elected members of the Parliament.
2. To be eligible for election as Vice-President, a person should have completed 30 years of age.
3. He should not be a member of either House of Parliament or a House of the state legislature.

Select the correct answer using the code given below:
a) 1 and 2 only
b) 3 only
c) 2 and 3 only
d) 1, 2 and 3 
2.8k views
Consider the following statements:

1. Martial law affects only Fundamental Rights and not centre-state relations and distribution of revenue.

2. It has specific provisions in the constitution under Centre State administrative relations. 

Which of the statements given above is/are correct?

a) 1 only
b) 2 only
c) Both 1 and 2
d) Neither 1 nor 2 
2.8k views
If a law is passed by Parliament, then it becomes mandatory for a state to abide by it in letter and spirit. Which among the following idea best describes this concept?

a.  Welfare State
b. Unitary State
c. Separation of powers
d. None of the above
2.8k views
If a law is passed by Parliament, then it becomes mandatory for a state to abide by it in letter and spirit. Which among the following idea best describes this concept?

a.  Welfare State
b. Unitary State
c. Separation of powers
d. None of the above

b seems to be most suitable. shows the unitary tilt of the Indian polity.

That's correct.

Article 256takes forward the idea of a unitary state as it ensures that the executive power of the Union shall extend to the giving of such directions to a State as may appear to the Government of India to be necessary for that purpose. It also stresses on the fact that the executive power of every State shall be so exercised as to ensure compliance with the laws made by Parliament and any existing laws which apply in that State.

2.7k views

mhs11said

Consider the following statements:

1. Martial law affects only Fundamental Rights and not centre-state relations and distribution of revenue.

2. It has specific provisions in the constitution under Centre State administrative relations. 

Which of the statements given above is/are correct?

a) 1 only
b) 2 only
c) Both 1 and 2
d) Neither 1 nor 2 

D

Incorrect.Answer is A

Article 34 provides for the restrictions onfundamental rightswhile martial law is in force in any area.  It empowers the Parliament to indemnify any government servant or any other person for any act done by him in connection with the maintenance or restoration of order in any area where martial law was in force. The Parliament can also validate any sentence passed, punishment inflicted, forfeiture ordered or other act done under martial law in such area. It has no specific provision in the Constitution. It is implicit.

2.7k views

mhs11said

Which of the following statements is/are correct regarding “Vice President?

1. Electoral College for his election consists of only elected members of the Parliament.
2. To be eligible for election as Vice-President, a person should have completed 30 years of age.
3. He should not be a member of either House of Parliament or a House of the state legislature.

Select the correct answer using the code given below:
a) 1 and 2 only
b) 3 only
c) 2 and 3 only
d) 1, 2 and 3 

B

Correct. 

2.7k views

mhs11said

Consider the following statements:

1. Interstate river water disputes fall within the jurisdiction of Art 262 and 131. 

2. Art 131  cannot be used to resolve dispute which involves a private party and a Government on the other side .

Which of the statements given above is/are correct?

a. 1 only

b. 2 only

c. Both 1 and 2

d. Neither 1 nor 2

B

Correct. 

2.6k views

mhs11said

Who among the following can enjoy parliamentary privileges'?

1. President

2. Prime Minister

3.Attorney General

Select the correct answer using code below.

a. 1 and 2 only

b. 2 and 3 only

c. 1 and 3 only

d. 1, 2 and 3


B

Correct.

2.6k views

Who among the following can enjoy parliamentary privileges'?

1. President

2. Prime Minister

3.Attorney General

Select the correct answer using code below.

a. 1 and 2 only

b. 2 and 3 only

c. 1 and 3 only

d. 1, 2 and 3


c

No, correct answer is B. President doesn't enjoy parliamentary privileges. 

2.6k views

Consider the following statements:

1. The number of judges of HC will be determined by the order of the President.

2. Strength of Supreme Court Judges can be increased through special majorityof the Parliament.

Which of the statements given above is/are correct?

a. 1 only

b. 2 only

c. Both 1 and 2

d. Neither 1 nor 2

@mhs11 

2.5k views

mhs11said

Merging of Union Territories requires amendment to which schedule of the Constitution?

a. First

b. Third

c. Seventh

d. Second

A. First schedule has names and boundaries of states and UTs.

Correct. 

2.6k views

mhs11said

Consider the following statements:

1. If a member leaves the party as a result of the merger of party with another party the rule of defection applies if the party whip has been issued.

2. If a member leaves his party temporarily on account of being elected as the presiding officer of the house to maintain the dignity of the position, he is not considered as a defecting member.

Which of the statements given above is/are correct?

a. 1 only

b. 2 only

c. Both 1 and 2

d. Neither 1 nor 2

Did not understand the first statement 😅. Second statement is correct

First statement is wrong. Would you want to tell what's the condition for merger?

2.6k views

mhs11said

» hide previous quotes

mhs11said

Consider the following statements:

1. If a member leaves the party as a result of the merger of party with another party the rule of defection applies if the party whip has been issued.

2. If a member leaves his party temporarily on account of being elected as the presiding officer of the house to maintain the dignity of the position, he is not considered as a defecting member.

Which of the statements given above is/are correct?

a. 1 only

b. 2 only

c. Both 1 and 2

d. Neither 1 nor 2

Did not understand the first statement 😅. Second statement is correct

First statement is wrong. Would you want to tell what's the condition for merger?

2/3rd members of the party need to agree to the merger. What is the role of party whip? And why would whip be issued?

Generally added to confuse. 

2.5k views
Geography
2.5k views
Which of the following best describes the term
'Khadin'?
a. It is a type of medicinal grass found in western
Rajasthan.
b. It is a traditional rainwater harvesting system
for agriculture used in Rajaasthan.
C. It is a type of primitive subsistence farming
system in Odisha.
d. It is the name given to salt patches in Kutch
region of Gujarat.
2.5k views
Which of the following are the features of a mountain crossed by youthful rivers?

1. Gorges
2. V-shaped valleys
3. Rapids
4. Waterfalls

Select the correct answer from the codes given below:

a. 1 and 2 only
b. 1, 2 and 3 only
c. 2, 3 and 4 only
d. 1, 2, 3 and 4
2.5k views
With reference to the 'Coriolis Force', consider the following statements:

1. Coriolis force affects both the wind direction and the wind speed.

2. Coriolis Effect is only seen in the Earth and not in other planets of the solar system.

Which of the statement(s) given above is/are
correct?

(a) 1 only
(b) 2 only
(c) Both 1 and 2
(d) Neither 1 nor 2
2.5k views
Which of the following are the reasons for the low salinity of ocean waters near the Equator?

1. Heavy rainfall
2. High rate of evaporation
3. Huge quantities of water brought to oceans
by the rivers

Select the correct answer from the codes given below.

(a) 1 only
(b) 2 and 3 only
(c) 1 and 3 only
(d) 1, 2 and 3
2.5k views
While moving from Goa to Puducherry, you will
pass through which of the following type of vegetation?

1. Tropical Evergreen Forest
2. Tropical Deciduous Forest
3. Tropical Thorn Forest
4. Littoral and Swamp Forest

Select the correct answer from the codes given
below:
a. 1 and 2 only
b. 1, 2 and 4 only
c. 2 and 4 only
d. 1, 2, 3 and 4
2.5k views
In recent years,there is a tendency for the sugar
mills to concentrate in the southern and western
states of India. In this context, which of the
following is/are the correct reasons?

1. The cane produced in these states have a higher sucrose content.
2. The cooler climate also ensures a longer crushing season for the producers.
3. The cooperatives are more successful in these states.

Select the correct answer from the codes given below:

a. 1 and 2 only
b. 2 and 3 only
c. 1 and 3 only
d. 1, 2 and 3
2.5k views

mhs11said

Which of the following are the features of a mountain crossed by youthful rivers?

1. Gorges
2. V-shaped valleys
3. Rapids
4. Waterfalls

Select the correct answer from the codes given below:

a. 1 and 2 only
b. 1, 2 and 3 only
c. 2, 3 and 4 only
d. 1, 2, 3 and 4

D

Correct. 

2.8k views

mhs11said

With reference to the 'Coriolis Force', consider the following statements:

1. Coriolis force affects both the wind direction and the wind speed.

2. Coriolis Effect is only seen in the Earth and not in other planets of the solar system.

Which of the statement(s) given above is/are
correct?

(a) 1 only
(b) 2 only
(c) Both 1 and 2
(d) Neither 1 nor 2

D? Coriolis effect is due to rotation, so it must be present in other planets as well?

Correct. 

Jupiter has the fastest rotation in the solar system. On Jupiter, the Coriolis effect actually transforms north-south winds into east-west winds.

2.8k views

mhs11said

Which of the following are the reasons for the low salinity of ocean waters near the Equator?

1. Heavy rainfall
2. High rate of evaporation
3. Huge quantities of water brought to oceans
by the rivers

Select the correct answer from the codes given below.

(a) 1 only
(b) 2 and 3 only
(c) 1 and 3 only
(d) 1, 2 and 3

A? High rate of evaporation increases salinity. And rivers are not present throughout the equator.

Correct

2.8k views

mhs11said

While moving from Goa to Puducherry, you will
pass through which of the following type of vegetation?

1. Tropical Evergreen Forest
2. Tropical Deciduous Forest
3. Tropical Thorn Forest
4. Littoral and Swamp Forest

Select the correct answer from the codes given
below:
a. 1 and 2 only
b. 1, 2 and 4 only
c. 2 and 4 only
d. 1, 2, 3 and 4

D.  All these types of forests can be seen in Goa itself! 

Correct

2.8k views

mhs11said

In recent years,there is a tendency for the sugar
mills to concentrate in the southern and western
states of India. In this context, which of the
following is/are the correct reasons?

1. The cane produced in these states have a higher sucrose content.
2. The cooler climate also ensures a longer crushing season for the producers.
3. The cooperatives are more successful in these states.

Select the correct answer from the codes given below:

a. 1 and 2 only
b. 2 and 3 only
c. 1 and 3 only
d. 1, 2 and 3

C. Eliminating statement 2 since there are warmer climates.

It's D. One drawback of sugar industries in the north and price volatility was that it has a very short crushing season (3-4 months of winters). So the mills have a very narrow window in northern states.

D is correct.

India stands second as a world producer of sugar but occupies the first place in the production of gur and khandsari. The raw material used in this industry is bulky, and in haulage its sucrose content reduces. The mills are located in Uttar Pradesh, Bihar, Maharashtra, Karnataka, Tamil Nadu, Andhra Pradesh, Gujarat, Punjab, Haryana and Madhya Pradesh. Sixty per cent mills are in Uttar Pradesh and Bihar. In recent years, there is a tendency for the mills to shift and concentrate in the southern and western states, especially in Maharashtra, This is because the cane produced here has a higher sucrose content.

The cooler climate also ensures a longer crushing season.

2.8k views
Consider the following statements:

1. The places east of Greenwich Meridian will
be ahead of Greenwich time and those in west
will be behind it.

2. In India, the time difference between two
places is maximum of one hour.

Which of the statement(s) given above is/are correct?

(a) 1 only
(b) 2 only
(c) Both 1 and 2
(d) Neither 1 nor 2
2.8k views
The main reason that the Earth experiences highest temperature in the sub-tropics in the Northern Hemisphere rather than at Equator is:

(a) Sub-tropical areas tend to have less cloud
cover than equatorial areas.

(b) Sub-tropical areas have longer day hours in
the summer than the equatorial.

(c) Sub-tropical areas have an enhanced
greenhouse effect compared to equatorial
areas.

(d) Sub-tropical areas are nearer to the oceanic
areas than the equatorial locations.
2.8k views
Which of the following mountain range is spread
to only one Indian state?

(a) Aravali
(b) Satpura
(c) Ajanta
(d) Sahyadri
2.8k views
Consider the following crops:

1. Groundnut
2. Wheat
3. Millet
4. Safflower

Which among the given above crops are 'Rabi crops'?

(a) 1 and 3 only
(b) 2, 3 and 4 only
(c) 2 and 4 only
(d) 1, 2 and 4 only
2.8k views
Can anyone share map for Rivers?

Hope this helps.

4.4k views
If India is experiencing economic growth, then which of the following statements must hold true:
1. Real GDP is increasing
2. Nominal GDP is increasing
3. Rate of growth of real GDP is increasing
4. Rate of growth of nominal GDP is increasing

Select the correct answer using the code given below:

(a) 1 only
(b) 1 & 2 only
(c) 1 & 3 only
(d) 1 & 4 only
2.3k views
Capital formation in a country will necessarily lead to which of the following:

1. Increase in ICOR
2. Decreasein ICOR
3. Economic growth

Select the correct answer using the code given below:

(a) 1 & 3 only
(b) 2 & 3 only
(c) 3 only
(d) None of the above
2.3k views
If a country’s growth rate is good but there is no corresponding growth in employment, then which of the following could be the reasons:

(a) The growth is coming from better utilization of existing capacity and not because of increase in investment

(b) The growth is coming from increase in investment but not because of better utilization of existing capacity

(c) The growth is coming from both as a result of increase in investment and increase in capacity utilization

(d) None of the above
3.1k views
Economic growth in a country will necessarily have to occur if:

(a) There is technological progress in the country
(b) There is population growth in the country
(c) There is capital formation in the country
(d) The country's exports are increasing
3.1k views
Which of the following are part of capital account transaction?

1. Masala bonds
2. Purchase of capital equipment from abroad
3. NRI deposits in Indian banks

Select the correct answer using the code given below:
(a) 1 only
(b) 1 & 2 only
(c) 3 only
(d) 1 & 3 only
3.1k views

mhs11said

If India is experiencing economic growth, then which of the following statements must hold true:
1. Real GDP is increasing
2. Nominal GDP is increasing
3. Rate of growth of real GDP is increasing
4. Rate of growth of nominal GDP is increasing

Select the correct answer using the code given below:

(a) 1 only
(b) 1 & 2 only
(c) 1 & 3 only
(d) 1 & 4 only

A?

Correct.

3k views

mhs11said

Economic growth in a country will necessarily have to occur if:

(a) There is technological progress in the country
(b) There is population growth in the country
(c) There is capital formation in the country
(d) The country's exports are increasing

C

Correct.

Investment in the economy means production of capital goods. 

When the economy produces all consumption goods and no capital goods (investment) then its GDP shall remain constant i.e. it will not grow. But till the time there is net production of capital goods i.e. investment in the economy, the production of goods and services (GDP) will keep on increasing. Capital formation means production of capital goods. So, if there is capital formation, it will necessarily lead to increase in GDP i.e. economic growth.

3k views

mhs11said

Capital formation in a country will necessarily lead to which of the following:

1. Increase in ICOR
2. Decreasein ICOR
3. Economic growth

Select the correct answer using the code given below:

(a) 1 & 3 only
(b) 2 & 3 only
(c) 3 only
(d) None of the above

C

Correct. 

Capital formation means production of capital goods.

Production of capital goods leads to future production of goods and services and hence economic growth. 

Production of capital goods increases the capital stock in the economy but does not tell whether there is any increase in efficiency of that capital. Efficiency is measured as how much output is produced from how much of inputs. So, we can’t say that ICOR will increase or decrease with capital formation.

3k views
Which of the following are part of capital account transaction?

1. Masala bonds
2. Purchase of capital equipment from abroad
3. NRI deposits in Indian banks

Select the correct answer using the code given below:
(a) 1 only
(b) 1 & 2 only
(c) 3 only
(d) 1 & 3 only

d

Correct. 

Masala bonds are issued outside India and money is raised in foreign currency, so it is part of our capital account. When NRIs are depositing money in Indian banks then it’s t ransaction between Indian residents (banks) and non-resident Indians and it creates a liability on Indian banks for future, hence it’s a capital receipt. Import of capital equipment are part of current account.

3k views
Capital formation in a country will necessarily lead to which of the following:

1. Increase in ICOR
2. Decreasein ICOR
3. Economic growth

Select the correct answer using the code given below:

(a) 1 & 3 only
(b) 2 & 3 only
(c) 3 only
(d) None of the above

C or D?

C- explanation above. 

2.9k views

mhs11said

If a country’s growth rate is good but there is no corresponding growth in employment, then which of the following could be the reasons:

(a) The growth is coming from better utilization of existing capacity and not because of increase in investment

(b) The growth is coming from increase in investment but not because of better utilization of existing capacity

(c) The growth is coming from both as a result of increase in investment and increase in capacity utilization

(d) None of the above

@upsc2020 what should be the answer to this?

A

Higher economic growth comes from additional investment, or increase in capacity utilization of the capital stock (factory).

When economic growth comes from new investment then generally more jobs are created but when economic growth comes from better utilization of the existing capacity (which was earlier not utilized properly) then jobs may not get created in the economy.

2.9k views
The term “Open access” in electricity means which of the following:

(a) Large consumers have access to the transmission and distribution network to obtain electricity from the suppliers of their choice

(b) Large consumers can choose the distribution company of their choice

(c) Any consumer of electricity can also generate electricity from their own resource and can feed on to the distribution network.

(d) Distribution companies can choose from which of the power producers they want to purchase power without government interference
2.8k views
Which of the following sectors have the highest allocation in “National Infrastructure Pipeline” (NIP)?

(a) Energy
(b) Road
(c) Rail
(d) Urban
2.8k views
‘Harmonized System of Nomenclature’ (HSN) was recently in the news, is related to which of the following?

(a) Classification of goods in international trade
(b) International financial transactions
(c) Space technology
(d) Classification of seeds and plants
2.8k views
Country X has not been granted "market economy status" by its group partners. Which of the following statements are true?

1. Prices of majority of products in the country X is regulated by the government
2. It is easy for the trading partners of country X to impose anti-dumping duty against X
3. Market economy status is related to IMF

Select the correct answer using the code given below:

(a) 1 & 2 only
(b) 2 & 3 only
(c) 1 & 3 only
(d) All of the above
3.5k views
The term “Open access” in electricity means which of the following:

(a) Large consumers have access to the transmission and distribution network to obtain electricity from the suppliers of their choice

(b) Large consumers can choose the distribution company of their choice

(c) Any consumer of electricity can also generate electricity from their own resource and can feed on to the distribution network.

(d) Distribution companies can choose from which of the power producers they want to purchase power without government interference

A?

Correct. 

Open access is the non-discriminatory use of transmission and distribution infrastructure of the licensees by consumers with demand greater than or equal to 1MW for procuring electricity from the source of their choice.

3.3k views
Country X has not been granted "market economy status" by its group partners. Which of the following statements are true?

1. Prices of majority of products in the country X is regulated by the government
2. It is easy for the trading partners of country X to impose anti-dumping duty against X
3. Market economy status is related to IMF

Select the correct answer using the code given below:

(a) 1 & 2 only
(b) 2 & 3 only
(c) 1 & 3 only
(d) All of the above

a


Correct.

3.3k views

mhs11said

‘Harmonized System of Nomenclature’ (HSN) was recently in the news, is related to which of the following?

(a) Classification of goods in international trade
(b) International financial transactions
(c) Space technology
(d) Classification of seeds and plants

A

Correct

3.3k views
Which of the following sectors have the highest allocation in “National Infrastructure Pipeline” (NIP)?

(a) Energy
(b) Road
(c) Rail
(d) Urban

a


Correct.

3.3k views
Write your comment…